Вы находитесь на странице: 1из 251

Gynecology & Obstetrics

State exam 2015/2016


Contents

Triplet 1 ................................................................................................................................................... 1
1.A Pelvic inflammatory disease .................................................................................................... 1
1.B Diagnosis and differentia diagnosis of Ectopic pregnancy ...................................................... 2
1.C Postpartum haemorrhage ....................................................................................................... 3
Triplet 2 ................................................................................................................................................... 5
2.A Puberty – physiology, pathology ............................................................................................. 5
2.B Standard examination methods in prenatal care.................................................................... 9
2.C Malpresentation of the vertex .............................................................................................. 12
Triplet 3 ................................................................................................................................................. 15
3.A Menstruation cycle ................................................................................................................ 15
3.B Anatomy of the female genitalia ........................................................................................... 18
........................................................................................................................................................... 19
3.C Positio alta occipitalis anterior .............................................................................................. 20
Triplet 4 ................................................................................................................................................. 20
4.A Gynecological examination of women (medical history, physical examination, digital
examination) ..................................................................................................................................... 20
4.B Hyperprolactinemia and galactorrhea .................................................................................. 24
4.C Development of the placenta and abnormality of placental development .......................... 26
Triplet 5 ................................................................................................................................................. 30
5.A Colpitis and vaginosis ............................................................................................................ 30
5.B Life Cycle of a Woman ........................................................................................................... 31
5.C Birth shock ............................................................................................................................. 32
Triplet 6 ................................................................................................................................................. 38
6.A Cancer screening in gynecology ............................................................................................ 38
6.B Amenorrhea .......................................................................................................................... 38
6.C Preeclampsia ......................................................................................................................... 43
Triplet 7 ................................................................................................................................................. 44
7.A Treatment of endometriosis – associated infertility ............................................................. 44
7.B Fertilization and fetal development ...................................................................................... 45
7.C Puerperal infections .............................................................................................................. 46
Triplet 8 ................................................................................................................................................. 47
8.A Menstruation and menstrual disorders ................................................................................ 47
8.B Ethical issues in gynecology, obstetrics and assisted reproduction...................................... 49
8.C Intrauterine hypoxia, causes and diagnosis .......................................................................... 50
Triplet 9 ................................................................................................................................................. 53
9.A Pelvic pain syndrome............................................................................................................. 53
9.B Legislation on assisted reproduction practice in Europe ...................................................... 54
9.C Placenta Praevia .................................................................................................................... 55
Triplet 10 ............................................................................................................................................... 57
10.A Regulation of the menstrual cycle ......................................................................................... 57
10.B Evaluation of fetoplacental function ..................................................................................... 58
10.C Kidney and urinary tract diseases in pregnancy .................................................................... 59
Triplet 11 ............................................................................................................................................... 61
11.A Dysmenorrhoea and dyspareunia ......................................................................................... 61
11.B First stage delivery................................................................................................................. 62
11.C Eclampsia ............................................................................................................................... 63
Triplet 12 ............................................................................................................................................... 64
12.A Chlamydial, Mycoplasmal and Ureaplasmal infections......................................................... 64
12.B Second stage delivery ............................................................................................................ 66
12.C Endocrine disorders in pregnancy ......................................................................................... 67
Triplet 13 ............................................................................................................................................... 71
13.A PCOS syndrome ..................................................................................................................... 71
13.B Third stage delivery ............................................................................................................... 71
13.C Preterm birth ......................................................................................................................... 73
Triplet 14 ............................................................................................................................................... 75
14.A Vulvar dystrophies and vulvitis ............................................................................................. 75
14.B Ectopic pregnancy ................................................................................................................. 76
14.C Diabetes mellitus and pregnancy .......................................................................................... 77
Triplet 15 ............................................................................................................................................... 79
15.A Endometriosis ........................................................................................................................ 79
15.B Endocrine disorders and female reproduction ..................................................................... 80
15.C HELLP syndrome .................................................................................................................... 81
Triplet 16 ............................................................................................................................................... 83
16.A Evalutation and Management of the Infertile Couple .......................................................... 83
16.B USG in pregnancy .................................................................................................................. 88
16.C Forceps and vacuum delivery ................................................................................................ 89
Triplet 17 ............................................................................................................................................... 91
17.A Influence of male factors on treatment of fertility/sterility.................................................. 91
17.B Spontaneous abortion ........................................................................................................... 92
17.C Anaemia in pregnancy ........................................................................................................... 93
Triplet 18 ............................................................................................................................................... 94
18.A Anovulation Treatment and Management............................................................................ 94
18.B Thrombophilias in Pregnancy ................................................................................................ 95
18.C Breech presentation and delivery ......................................................................................... 97
Triplet 19 ............................................................................................................................................... 99
19.A Assisted reproductive techniques ......................................................................................... 99
19.B Changes in the reproductive system /uterus and ovaries/ during pregnancy .................... 100
19.C Asynclitic birth ..................................................................................................................... 101
Triplet 20 ............................................................................................................................................. 102
20.A Cervicitis and endometritis.................................................................................................. 102
20.B Gestational trophoblastic disease ....................................................................................... 103
20.C Rhesus incompability........................................................................................................... 105
Triplet 21 ............................................................................................................................................. 106
21.A Adnexitis and parametritis .................................................................................................. 106
21.B Fetoplacentar unit function................................................................................................. 108
21.C Failure of uterine contractions during labour ..................................................................... 109
Triplet 22 ............................................................................................................................................. 111
22.A Preservation of fertility in cancer patients, cryopreservation of gametes and embryos ... 111
22.B Central nervous and respiratory system development ....................................................... 113
22.C Pregnancy and delivery in cardiac disease .......................................................................... 116
Triplet 23 ............................................................................................................................................. 117
23.A Pelviperitonitis and peritonitis diffusa ................................................................................ 117
23.B The development of the cardiovascular system and fetal blond circulation ...................... 118
23.C Abruptio placentae praecox ................................................................................................ 119
Triplet 24 ............................................................................................................................................. 120
24.A Benign lesions of the vulva, vagina and cervix uteri ........................................................... 120
24.B Termination of pregnancy ................................................................................................... 122
24.C Hypertensive disorders of pregnancy ................................................................................. 123
Triplet 25 ............................................................................................................................................. 125
25.A Ovarian cancer..................................................................................................................... 125
25.B Prenatal care ....................................................................................................................... 126
25.C The transverse and oblique fetal presentation ................................................................... 127
Triplet 26 ............................................................................................................................................. 128
26.A Benign tumors of the uterus ............................................................................................... 128
26.B Physiological changes during Pregnancy ............................................................................. 129
26.C HIV/AIDS during pregnancy ................................................................................................. 131
Triplet 27 ............................................................................................................................................. 133
27.A Malignant tumors of uterus ................................................................................................ 133
27.B Sexual transmission diseases .............................................................................................. 134
27.C Intrauterine Fetal Death – causes, diagnosis, treatment .................................................... 137
Triplet 28 ............................................................................................................................................. 139
28.A Medical Definition of Climacterium .................................................................................... 139
28.B Management of normal labor ............................................................................................. 140
28.C Coagulation disorders in pregnancy .................................................................................... 142
Triplet 29 ............................................................................................................................................. 144
29.A Uterine descensus and prolapse ......................................................................................... 144
29.B Implantation of the embryo and implant failure ................................................................ 146
29.C Post-term pregnancy and induced labour........................................................................... 147
Triplet 30 ............................................................................................................................................. 150
30.A Prevention and early diagnosis of malignant tumors in gynecology .................................. 150
30.B Secondary amenorrhea ....................................................................................................... 151
30.C Premature rupture of membranes ...................................................................................... 152
Triplet 31 ............................................................................................................................................. 154
31.A Urinary incontinence in women .......................................................................................... 154
31.B Differential diagnosis of ectopic pregnancy ........................................................................ 155
31.C Genetic birth defects ........................................................................................................... 157
Triplet 32 ............................................................................................................................................. 159
32.A Cervical cancer..................................................................................................................... 159
32.B Cardiotocograph plus ST analysis of fetal electrocardiogram and pulse oximetry ............. 162
32.C Multiple pregnancy ............................................................................................................. 165
Triplet 33 ............................................................................................................................................. 167
33.A Precancerous lesions of the vulva, vagina and cervix uteri................................................. 167
33.B Screening for congenital birth defects in pregnancy .......................................................... 169
33.C Bleeding in pregnancy – causes, diagnosis, therapy ........................................................... 171
Triplet 34 ............................................................................................................................................. 175
34.A Vaginal and vulvar cancer.................................................................................................... 175
34.B The Newborn Infant – postpartum treatment and examination ........................................ 175
34.C Complications of 3rd stage of labour ................................................................................... 177
Triplet 35 ............................................................................................................................................. 179
35.A Breast cancer ....................................................................................................................... 179
35.B Latest news on hormone replacement therapy .................................................................. 180
35.C Uterine rupture ................................................................................................................... 181
Triplet 36 ............................................................................................................................................. 182
36.A Non-epithelial ovarian cancer ............................................................................................. 182
36.B Planned parenthood and contraception ............................................................................. 183
36.C Umbilical Cord Pathology /Abnormalities ........................................................................... 185
Triplet 37 ............................................................................................................................................. 188
37.A Benign tumors of the ovary ................................................................................................. 188
37.B Obstetrical analgesia and anaesthesia ................................................................................ 189
37.C Premature infant ................................................................................................................. 190
Triplet 38 ............................................................................................................................................. 192
38.A Benign breast diseases ........................................................................................................ 192
38.B Abnormal puerperium ......................................................................................................... 198
38.C Sectio Cesarea ..................................................................................................................... 204
Triplet 39 ............................................................................................................................................. 207
39.A Hormonal treatment in gynecology .................................................................................... 207
39.B Imaging modalities/methods in obstetrics and gynecology ............................................... 209
39.C Injuries of fetus and newborn ............................................................................................. 210
Triplet 40 ............................................................................................................................................. 212
40.A Hormonal contraception ..................................................................................................... 212
40.B Development of the urogenital system and malformations of female genitalia ................ 214
40.C Maternal death.................................................................................................................... 216
HIV/AIDS: ..................................................................................................................................... 216
Triplet 41 ............................................................................................................................................. 216
41.A Hormonally active ovarian tumors ...................................................................................... 217
41.B Infectious diseases in pregnancy ......................................................................................... 221
41.C IUGR ..................................................................................................................................... 225
Triplet 42 ............................................................................................................................................. 227
42.A Diagnosis of amenorrhoea .................................................................................................. 227
42.B Anthropozoonosis and pregnancy....................................................................................... 231
42.C Abnormal puerperium ......................................................................................................... 233
Triplet 43 ............................................................................................................................................. 234
43.A Gynecology Preventive Care ............................................................................................... 234
43.B Normal puerperium ............................................................................................................. 235
43.C Abnormalities of pelvis and birth canal ............................................................................... 236
Triplet 44 ............................................................................................................................................. 239
44.A Pediatric and Adolescent Gynecology ................................................................................. 239
44.B Hormones of the Placenta ................................................................................................... 240
44.C Critical conditions in obstetrics ........................................................................................... 240

Note:

This document features functional list of content as well as hyperlinks in specific topics which refer
to other ones. In order to use just click on the topic or link and you will be moved to the referred
content.

Disclaimer:

Content of this document serves for educational purposes only. Authors of this document do not
own the rights to any information/picture/table/figure herein. All materials were taken from
various medical books as well as medical websites about the given topics and serves only as a
summary.
Triplet 1

1.A Pelvic inflammatory disease


Definition – When infections ascend from cervix or vagina into upper genital tract.

Endometritis, salpingitis, tubo-ovarian abscess & pelvic peritonitis.

Main causes: Chlamydia. Trachomatis, Neisseria. Gonorrhoeae (Mycoplasma & anaerobes).

Sometimes in laparoscopically proven PID, no bacterial cause is found.

Incidence unknown (~ 2/3 asymptomatic).

Clinical presentation:

None to very severe.


Onset of symptoms usually in first part of menstrual cycle.
Chlamydial PID is usually milder than gonococcal PID.
Lower abdominal pain is the most common symptom.
Increased vaginal discharge, irregular bleeding, postcoital bleeding, dyspareunia (pain
during sex), cervicitis.
Adnexal & cervical tenderness on examination.
May be Pyrexia (fever) ?
Palpable adnexal mass. ?

Diagnosis:

No specific symptoms, signs or lab tests.


Made by clinical findings.

Differential diagnoses: appendicitis & ectopic pregnancy.

Pregnancy test.
Laparoscopy + microbiology of upper & lower genital tract ("gold standard").
Not always available/appropriate in mild cases.

Treatment:

Do not delay (no waiting for test result).


Outpatient, Oral antibiotics for mild to moderate cases.
I.V therapy in severe cases.
Hospitalization if diagnostic uncertainty, severe case or failure to respond to oral therapy.
Antibiotic combination: Cefoxitin + Doxycycline + Metronidazole + Ofloxacin +
Ceftriaxone????
Analgesia.
No sex until both partners, completed treatment (in case of STD).
In moderate (severe) cases - review 2–3 days to ensure improvement.
Lack of response, requires further investigation, I.V therapy, surgery.

All patients should be seen after treatment to check clinical response & medication completed.

1
Complications:

- Main complications from PID are due to tubal damage.


- Tubal infertility.
- Ectopic pregnancy (x6-10).
- Chronic abdominal or pelvic pain.
- Hysterectomy.
- Repeats in 1/3.
- Infection of liver capsule (perihepatitis).

1.B Diagnosis and differentia diagnosis of Ectopic pregnancy


Definition:

Non-intrauterine pregnancy (ovarian, cervical or intra-abdominal).


Usually tubal.
Major cause of mortality!

Clinical presentation:

Pain, amenorrhea (abnormal absence of menstruation) & vaginal bleeding.


Pelvic tenderness and/or cervical excitation and/or shoulder tip pain due to diaphragmatic
irritation from haemoperitoneum.

Diagnosis:

The date of the last menstrual period, date of pregnancy test & symptoms suggesting pelvic
infections.
Pelvic examination should be gentle to avoid tubal rupture.
hCG which does not increase > 66% in 48 hours increases the likelihood of ectopic pregnancy.

Treatment:

Resuscitation & blood transfusion may be required.


Management depends on the overall clinical picture, the scan result and the serum level of
hCG.
Tubal pregnancy can be managed by:
o Laparotomy
o Laparoscopy
o medically or by observation alone.

If shocked (hypotensive...) - immediate pregnancy test (ectopic pregnancy) & consider urgent
laparotomy if positive.
If hemodynamically stable, a laparoscopic approach to the surgical management of tubal
pregnancy is preferable to an open approach.
hCG should be rechecked in 48 hours. If levels are not doubled, steady or only slightly
reduced, consider a laparoscopy.
In a well woman with a positive UPT and an empty uterus on transvaginal ultrasound, a
serum hCG level is performed. If the level is over 1500 IU/l, consider a laparoscopy.

2
Methotrexate is an option for ectopic pregnancy with minimal symptoms, clinically stable &
hCG level < 3000 IU/l.
Laparoscopic salpingectomy or salpingotomy is appropriate.
Postoperative tracking of serum hCG is necessary following salpingotomy, to identify cases
complicated by persistent trophoblast.
Expectant management is an option for clinically stable asymptomatic women with an
ultrasound diagnosis of ectopic pregnancy and a decreasing serum hCG, initially less than
1000 IU/l.

When serum hCG levels are below the discriminatory zone (<1000 IU/l) and there is no
pregnancy (intra-or extra-uterine) visible on transvaginal ultrasound scan, the pregnancy can
be described as being of unknown location (pregnancy of unknown location or PUL).

If women are managed expectantly, serial serum hCG measurements should be performed
until hCG levels are less than 15 IU/l.

Non-sensitized Rh- woman with a confirmed or suspected ectopic pregnancy should receive
anti-D immunoglobulin.

Differential diagnosis:

Miscarriage, continuing intrauterine pregnancy, PID, Appendicitis, Salpingitis


Ruptured corpus luteum cyst or ovarian follicle.
Spontaneous abortion or threatened abortion, Ovarian torsion.
Cervical cancer, dysmenorrhea, placenta previa.
UTI.

Complications:

Bleeding, shock, death.


Infertility.
Infections.

1.C Postpartum haemorrhage


Definition – Bleeding after delivery.

Primary postpartum haemorrhage:


Blood loss > 500 ml < 24h of delivery.
Risk factors: multiparity, multiple pregnancy, fibroids, polyhydramnios, placenta previa &
long labour.

Secondary postpartum hemorrhage:

Any significant loss 24h - 6 weeks from delivery.


Usually due to infection and/or retained products of conception.
More likely in women with a past history of postpartum hemorrhage.
Important to treat anaemia in the antenatal period.
Oxytocin (after delivery).

3
Main causes of PPH:
Atony: retained placenta (90%). Normally, contraction of the uterus in the third stage of
labour causes compression of blood vessels & bleeding stops.

Trauma: Episiotomy, a vaginal or cervical laceration or a rupture in the uterine wall.

Lacerations of genital tract after an instrumental delivery.

Coagulation problems: usually disseminated intravascular coagulation (DIC).

Clinical presentation:
Bleeding is usually obvious, but, an atonic uterus can fill up without obvious external loss &
the first real sign can be cardiovascular collapse (shock).
Easy to underestimate the real loss.
The most critical factors are the signs of shock, pallor, tachycardia & falling BP.

Palpate the abdomen to assess the size and tone of the uterus.

If uterus is atonic, a contraction can be stimulated by abdominal massage.

I.V access should be established with 2 wide-bore cannulas & blood taken for hemoglobin,
hematocrit, platelets, clotting & crossmatch.

Oxytocin i.v. should be given to further contract the uterus, followed by a oxytocin infusion.
Crystalloid and/or colloid should be rapidly infused to maintain the circulating volume.
A urinary catheter should be inserted to aid compression of the uterus & to measure urine
output.
If placenta has not been delivered, gently attempt at controlled cord traction.
In Hemorrhage + placenta accreta consider a hysterectomy.

Treat DIC with fresh frozen plasma or cryoprecipitate.

Techniques to stop hemorrhage are aimed at either maintaining compression of the uterus or
applying pressure on the placental bed.
o By suture, intrauterine Balloons & surgical packs.
o Hysterectomy indicated, especially in uterine rupture or placenta accreta.
o Internal iliac artery ligation (only for atony).
o Radiologically guided internal iliac artery embolization.

The decision is usually between conservative management with antibiotics, or arranging for an
evacuation of retained products with antibiotic cover under anesthesia.

In the first week the evacuation can often be carried out digitally.

However, in the presence of persistent bleeding, USG to observe the spontaneous resolution of
intrauterine hematoma & identify retained products.

4
Triplet 2

2.A Puberty – physiology, pathology


Definition:
Puberty should transform a girl into a fertile woman.

Puberty encompasses: an adolescent growth spurt the acquisition of secondary sexual


characteristics the onset of menstruation (menarche) & the establishment of ovulatory function.

Endocrine changes: reactivation of the hypothalamic-pituitary–ovarian axis.

⇨ Gonadotrophins (GnRH - FSH & LH) release, lead to production of ovarian estrogen, which
initiates the physical changes of puberty.

⇨ External signs of puberty usually occur in a specific order.

⇨ Adolescent growth spurt is an early feature.

⇨ This acceleration in growth is dependent on growth hormone as well as gonadal steroids.

⇨ Menarche (first menstruation) is a late feature.

⇨ Breast development, which is primarily under the control of ovarian estrogens, is described
in 5 stages (Tanner stages of breast development).

⇨ The appearance of the breast bud is followed by pubic & axillary hair, mainly under the
influence of ovarian & adrenal androgens (Tanner stages of pubic hairdevelopment).

⇨ The age of menarche is ~ 13.

⇨ This has been attributed to improvement in socioeconomic conditions, nutrition & general
health.

Influence of body weight: attainment of a critical body weight (48 kg).

⇨ Body fat increases to ~ 22% of body weight.

⇨ Delay of a critical body weight may delay menarche (Malnutrition, Slow growth before &
after birth, Twins, Athletic training& Eating disorders).

⇨ Anorexia nervosa can cause both primary & secondary amenorrhoea & a halt in pubertal
progress.

95% of normal girls attain stage 2 breast development by age 13.

5
50% will complete all stages of puberty by age 16.

The bone age, which is an index of physiological maturation, correlates closely with the menarche.

It can be measured by an X-ray of the hand.

80% of girls begin to menstruate at a bone age of ~13.

⇨ Menstrual cycles tend to be irregular as ovulation is initially infrequent.

⇨ Most girls take several months / a year to establish a regular cycle.

Pathologic puberty:

➢ Delayed puberty: absence of physical manifestations of puberty by age 13.

➢ Primary amenorrhoea: no menstruation by the age of 14 accompanied by failure to develop


secondary sexual characteristics OR no menstruation by age 16 in the presence of normal
sexual development.

➢ Arrested puberty: puberty starts without normal progression.

➢ Constitutional delay: normal but inherently late in entering puberty.

o Although these individuals are usually of short stature & have usually been shorter
than their peers for years, their height is generally appropriate for their bone age.

o All stages of development are delayed.

o They may be considered to be physiologically immature, with a functional deficiency


of gonadotrophin-releasing hormone (GnRH) for their chronological age, but not for
their stage of physiological development.

o On attaining a bone age of 11–13 years they can be expected to enter puberty.

o Associated with:

▪ Conditions affecting weight.

▪ Systemic disease.

▪ Malnutrition.

▪ Anorexia nervosa.

o Restoration of weight usually results in spontaneous onset of puberty.

6
CNS tumors: interference with GnRH synthesis / secretion / stimulation of the pituitary gland
(craniopharyngioma).

Primary (peripheral) gonadal failure (hyper-gonadotrophic hypogonadism):

o Hypogonadism due to an impaired response of the gonads to the gonadotropins


(FSH&LH) resulting in a lack of sex steroid production (estrogen & progesteron)&
elevated gonadotropin levels (as an attempt of compensation by the body).

o May present as either congenital (usually)/ acquired.

o There are many different etiologies of HH.

1) Congenital causes:

Chromosomal abnormalities (resulting in gonadal dysgenesis) - Turner's syndrome,


Klinefelter's syndrome, Swyer's syndrome, XX gonadal dysgenesis, & mosaicism.???????

Defects in the enzymes involved in the gonadal biosynthesis of the sex hormones.

Gonadotropin resistance: FSH insensitivity &LH + FSH resistance due to mutations in the
GNAS gene.

2) Acquired causes (due to damage / dysfunction of the ovaries):

Ovarian torsion.

Premature ovarian failure.

Ovarian resistance syndrome.

Trauma, surgery, autoimmunity, chemotherapy, radiation, infections (STD), toxins & drugs
(anti androgens, opioids, alcohol).

Diagnosis:

➢ Hormone panel: FSH, LH, estradiol, prolactin & thyroid.

➢ History & physical examination.

➢ Pelvic USG.

➢ Karyotype.

➢ X-ray for bone age.

7
➢ Pelvic CT / MRI.

Treatment (cause dependent):

➢ They will require hormone replacement therapy, until age 50.

➢ The first stage in treatment is to achieve apparently normal puberty by estradiol.

➢ Commonly, ongoing estrogen replacement is with the combined oral contraceptive pill
(COCP).

Precocious puberty:

➢ Signs of sexual maturation < 8 years.

➢ The growth spurt is a striking feature.

➢ Menstruation usually brings the girl to medical checkup.

Intracranial lesions:

➢ This is the next most likely cause, particularly in younger girls.

➢ An intracranial lesion resulting from encephalitis, meningitis, hydrocephaly / a small space-


occupying lesion, may trigger premature reactivation of the hypothalamic-pituitary–ovarian
axis.

➢ Feminizing tumors of the ovary / adrenal may give rise to vaginal bleeding without signs of
pubertal development.

Diagnosis of precocious puberty:

➢ Hormone panel: FSH, LH, estradiol & thyroid.

➢ X-ray of the hand to determine bone age.

➢ USG of abdomen & pelvis.

➢ Radiological skeletal survey of the long bones.

➢ Cranial CT / MRI.

8
The goal is To arrest or regress of the physical signs of puberty&in particular menstruation.

To avert the rapid advance in bone age, as premature fusion of the epiphyses would compromise the
final height of the child.

The introduction of GnRH agonists, which suppress gonadotrophin secretion for constitutional &
cerebral precocious puberty.

Sexual abuse:

This is the involvement of dependent sexually immature children & adolescents in sexual activity
they do not truly comprehend, to which they are unable to give informed consent & which violates
social taboos / family roles.

Particular attention should be paid to bleeding, bruising / any other area of injury.

2.B Standard examination methods in prenatal care


Antenatal care is a screening program aimed at identifying problems at an early stage to minimize
the risks to mothers & their babies.
For health promotion, preparation for labour, parenthood & surveillance of risk.
The initial visit ideally 8 - 10w, every 4 weeks until 30w, every 2w until 32w & then every
week.

Past obstetric history: previous pregnancies & labours, gestation at delivery & induced or
spontaneous labour.
The duration of labour, mode of delivery, birth weight, sex, neonatal outcome & postnatal
complications.

Medical & surgical history: Previous operations, particularly gynaecological procedures (cone biopsy
may predispose to cervical incompetence) & blood transfusions received?
Hypertension, diabetes, heart disease, renal disease, epilepsy, asthma, thyroid dysfunction.

Family history: potential inherited conditions, thalassemia, cystic fibrosis, sickle cell anaemia,
chromosomal disorders & structural abnormalities.

History of present pregnancy: The date of the first day of the last menstrual period & details of the
menstrual cycle before conception.
Correlation with early pregnancy, USG dating is important.

Social & drug history: drugs taken, during the pregnancy. Alcohol, smoking & drug abuse.
Evidence of socioeconomic deprivation.
Identification of matters relating to child protection necessitates (social work department).

9
Examination: pulse rate, BP, weight & height.

A BMI > 30 is potential for complications.


Abdominal examination: approximate indication of the uterine size, identify abnormal
masses & other abnormalities.
There is no indication for a routine vaginal examination, perform cervical cytology if overdue.
USG: fetal viability?, gestational age, identifies multiple pregnancy.
Measure nuchal translucency & diagnose fetal anomalies (anencephaly).
Urine analysis: protein & glucose.
Blood test: CBC (anaemia & thrombocytopenia). Blood group ABO + Rh + presence of red cell
antibodies.
Rubella status: check mothers immunity.
Hemoglobin electrophoresis (sickle cell anaemia or thalassemia).
Hepatitis B status.
Serology for syphilis.
HIV.

Discuss screening options for chromosomal & structural abnormalities.

Mothers at the extremes of reproductive age are at increased risk of obstetric complications,
particularly hypertensive disorders & perinatal mortality.

The incidence of proteinuria pre-eclampsia in a second pregnancy is x10–15 greater if there was pre-
eclampsia in the first pregnancy.

Those who have had a previous instrumental delivery usually have a normal delivery next time.

In general, those with a previous C section for a non-recurrent indication, e.g. breech, fetal distress
or relative cephalopelvic disproportion (CPD) secondary to fetal malposition should be offered
vaginal birth, although repeat elective C section may be recommended in certain circumstances.

Smoking is associated with low-birth-weight babies.

Although there is no evidence to support an association with fetal abnormality, long-term


follow-up has demonstrated intellectual & emotional impairment.
Increased risk of placental abruption, preterm labour, intrauterine fetal demise & sudden
infant death syndrome (SIDS).

Alcohol & drug abuse carry significant fetal risks. Avoid in pregnancy!
May continue working providing she is not 2 tired.

Moderate exercise is likely to be of benefit and should be encouraged.

Antenatal surveillance: used to identify obstetric complications. Gestational hypertension & pre-
eclampsia.
BP & urinalysis, checked every visit.

‘Small for gestational age’ (SGA): birth weight < 10th percentile.

‘fetal growth restriction’ (FGR): ‘a fetus which fails to reach its genetic growth potential’.
Carries a significant risk of antenatal & intrapartum asphyxia, intrauterine death, neonatal
hypoglycemia, long-term neurological impairment & perinatal death.

10
USG will identify, most small babies.

Impaired glucose tolerance test & diabetes: If family history of diabetes, previous large-for-
gestational-age baby or persistent glycosuria.

Hemolytic disease: Maternal IgG antibodies to fetal red cell antigens cross the placenta & may lead
to fetal hemolysis, anaemia & "hydrops fetalis".

Initial sensitization usually occurs in previous delivery, but may occur with vaginal bleeding at
any stage, amniocentesis, external cephalic version or an unrecognized event (silent
fetomaternal transfusion).
The most significant antibody is Rh antigen (Rh- mothers may develop against Rh+ fetal cells).
All women should be screened for anti-red cell antibodies at booking & again in the 3rd
trimester.
Those with antibodies require further investigation.
o Rh- women without sensitization are recommended to receive 2 doses of anti D.

Breech presentation: associated with multiple pregnancy, bicornuate uterus, fibroids, placenta
praevia, polyhydramnios & oligohydramnios.
Confirmed by USG.
Planned C section at term is associated with less perinatal mortality & less serious neonatal
morbidity.

Anaemia As there is a physiological fall in hemoglobin (Hb) as pregnancy advances.


Oral FeSO4 if Hb <10 g/dl or if the mean corpuscular volume (MCV) is low (<80 fl).
Check folate, B12 & ferritin before deciding on therapy.
Parenteral iron should never be given in thalassemia!

Polyhydramnios (Excess of liquid) In the 2nd & 3rd trimesters, liquor is produced by fetal kidneys and
is swallowed by the fetus.
Diagnosed by USG (a single pool >8 cm in depth, and/or an amniotic fluid index > 90th
percentile).

Caused by Increased production from high urine output Macrosomia, diabetes, recipient of
twin–twin transfusion, "hydrops fetalis", Gastrointestinal obstruction, Esophageal atresia,
duodenal atresia, bowel obstruction or Hirschsprung’s disease Poor swallowing because of
neuromuscular problems or mechanical obstruction Anencephaly, myotonic dystrophy,
maternal myasthenia, facial tumor, macroglossia or micrognathia.
Associated with: placental abruption, malpresentation, cord prolapse, carrying a large-for-
gestational-age infant requiring a caesarean section, perinatal death.
Antibody titers should be checked to exclude alloimmune haemolytic disease.

Increased antenatal fetal surveillance + increased awareness of the risks of intrapartum


complications.

Prolonged pregnancy (> 42 weeks) Is associated with an increased perinatal mortality due to
‘unexplained’ intrauterine death, intrapartum hypoxia and meconium aspiration syndrome.

Sweeping the membranes By vaginal examination and inserting a finger through the internal os to
separate the membranes from the uterine wall, thus releasing endogenous prostaglandins.

11
Induction of labour after 41 weeks reduces the incidence of fetal distress & meconium staining &
reduced C section rate.

Fetal monitoring by fetal movement charts, cardiotocography (CTG), biophysical profile scoring &
Doppler flow velocity studies.

Fetal cardiotocography (CTG): indication of fetal well-being at a particular moment but has little
longer-term predictive value.

Fetal biophysical profile (BPP): five parameters are assessed.?????

Doppler flow velocity: of umbilical arteries indicates placental vascular resistance.


Useful in pregnancies considered at risk of hypoxia due to impaired placental function.
In severe compromise, diastolic flow may stop altogether or may even reverse.
Abnormal waveforms (absent or reduced - end diastolic flow) are associated with an
increased risk of structural & chromosomal abnormalities.

2.C Malpresentation of the vertex


Definition:

➢ "presentation": the part of the fetus presenting to the pelvic inlet.

➢ Normal presentation is vertex.

‘Malpresentation’ describes any non-vertex presentation (face, brow, breech, other part of the
body).

Clinical presentation:

➢ 'Malposition’ is when the head, coming vertex first, does not rotate to occipitoanterior
(persistent occipitotransverse / occipitoposterior).

➢ The presenting diameter is dependent on the degree of flexion / extension of the head
(deflexed & brow presentations have a wide diameter).

12
➢ Fetal neurocranium: occipital, sphenoid, parietal, temporal & ethmoid bones joined by
frontal, sagittal, lambdoid & coronal sutures.

➢ ‘Moulding’: change in shape of the skull during labour due to the birth canal shape.

➢ Face presentation: associated with anencephaly (rare), edema & bruising. Usually only
recognized after the onset of labour.

o If the face is swollen it is easy to confuse with a breech presentation.

➢ Brow presentation: least favorable.

o The head may flex to become a vertex presentation / extend to a face presentation.

Treatment:

➢ C section may be required.

Complications:

➢ Increased risk for injuries to the uterus / birth canal (bleeding) & abnormal labor ("failure to
progress").

➢ Fetal distress: tachycardia / bradycardia.

➢ Caput: Edema of fetal head.

➢ Fetal neurocranium: occipital, sphenoid, parietal, temporal & ethmoid bones joined by
frontal, sagittal, lambdoid & coronal sutures.

➢ ‘Moulding’: change in shape of the skull during labour due to the birth canal shape.

➢ Face presentation: associated with anencephaly (rare), edema & bruising. Usually only

Definition:

➢ "presentation": the part of the fetus presenting to the pelvic inlet.

➢ Normal presentation is vertex.

‘Malpresentation’ describes any non-vertex presentation (face, brow, breech, other part of the
body).

Clinical presentation:

➢ 'Malposition’ is when the head, coming vertex first, does not rotate to occipitoanterior
(persistent occipitotransverse / occipitoposterior).

13
➢ The presenting diameter is dependent on the degree of flexion / extension of the head
(deflexed & brow presentations have a wide diameter).

recognized after the onset of labour.

o If the face is swollen it is easy to confuse with a breech presentation.

➢ Brow presentation: least favorable.

o The head may flex to become a vertex presentation / extend to a face presentation.

Treatment:

➢ C section may be required.

Complications:

Increased risk for injuries to the uterus / birth canal (bleeding) & abnormal labor ("failure to
progress").
Fetal distress: tachycardia / bradycardia.
Caput: Edema of fetal head.

14
Triplet 3

3.A Menstruation cycle


Definition:

Monthly series of changes a woman's body goes through in preparation for the possibility of
pregnancy.

Each month, one of the ovaries releases an egg (ovulation) & hormonal changes prepare the
uterus for pregnancy.

The menstrual cycle can be described by changes in uterus or ovary.

The endometrial cycle results from the growth & shedding of the uterine lining.

At the end of the menstrual phase, the endometrium thickens again (proliferative phase).

After ovulation, endometrial growth stops, the glands become more active & full of
secretions (secretory phase).

Endometrial changes are controlled by the ovarian cycle.

The average duration of the ovarian cycle is 28 days (follicular, ovulation & post ovulatory
or luteal phase).

If the cycle is prolonged, the follicular phase lengthens (longer time to ovulation) but the
luteal phase remains constant at 14 days.

Fundamental to normal cycle: hypothalamic-pituitary–ovarian axis + responsive follicles in


ovaries + a functional uterus.

15
The hypothalamus controls the cycle & influenced
by higher centers (anxiety or stress can change
the cycle).

Hypothalamus --> pituitary by


gonadotrophin-releasing hormone
(GnRH).

GnRH stimulates --> follicle-stimulating


hormone (FSH) & luteinizing hormone
(LH).

FSH stimulates growth of follicles


(follicular phase) & sex hormone
secretion (estradiol).

LH stimulates sex hormone production


(testosterone, converted by FSH to
estradiol).

The mid-cycle surge of LH which triggers rupture of the mature follicle (ovulation).

Post-ovulatory production of progesterone by the corpus luteum is also under the influence
of LH.

The ovarian cycle:

I. Follicular phase Days 1–8: FSH + LH rise in response to the fall of estrogen & progesterone
at menstruation.

This stimulates development of 10–20 follicles. With growth of the dominant follicle,
estrogen levels increase.

II. Days 9–14: The primary follicle transforms into a Graafian follicle (oocyte inside).

Follicular maturation is associated with increase in the production of estradiol by the


developing follicle.
As the estrogen level rises, the release of both gonadotrophins is suppressed (negative
feedback).

III. Ovulation (Day 14): rapid enlargement of the follicle, followed by protrusion from the
surface of the ovarian cortex & rupture of the follicle with extrusion of the oocyte
(ovulation).

Some women can identify the time of ovulation because they experience a short-lived pain in
iliac fossa.

The final rise in estradiol concentration is thought to be responsible for the subsequent mid-
cycle surge of LH & FSH (positive feedback).

16
Immediately before ovulation there is a precipitous fall in estradiol & increase in
progesterone.

IV. Luteal phase (Days 15–28): The Graafian follicle forms the corpus luteum.

This is the major source of sex hormones (estrogen & progesterone) secreted by the ovary in
the postovulatory phase.

Progesterone levels peak 1 week after ovulation (day 21 of 28-day cycle).

During the luteal phase gonadotrophin levels remain low until the regression of the corpus
luteum (days 26–28).

If conception & implantation occur, the corpus luteum does not regress, because it is
maintained by hCG secreted by the trophoblast.

The detection of the presence of hCG in a sample of urine forms the basis of pregnancy testing.

If conception & implantation do not occurr, the corpus luteum regresses, progesterone levels
fall & menstruation starts.

The fall of sex hormones (estrogen & progesterone) allows the FSH + LH levels to rise & start
the next cycle.

The uterine cycle:

The cyclical production of sex hormones (estrogen & progesterone) by the ovary induces
changes in the uterus (endometrium & cervical mucus).

The endometrium: has a superficial layer which is shed during menstruation & a basal layer
which regenerates the superficial layer during the subsequent cycle.

I. Proliferative phase: During the follicular phase in the ovary, the endometrium is exposed to
estradiol.

After menstruation, the secretion of estradiol from the ovary brings about repair &
regeneration of the endometrium + growth & proliferation of glands, blood vessels.

II. Secretory phase: After ovulation, progesterone production prepares the endometrium for
implantation.

III. Menstrual phase: Normally, the luteal phase of the ovary lasts for 14 days, at the end of which
regression of the corpus luteum is associated with a decline in ovarian sex hormones (estradiol
& progesterone).

This fall (estrogen & progesterone) is followed by intense spasmodic contraction of the
endometrial arterioles & ischemic necrosis (shedding the superficial layer & bleeding).

The vasospasm & uterine contractions at the time of the menstrual flow is by prostaglandins.

17
Basal body temperature: rises ~ 0.5°C following ovulation & sustained until menstruation
(progesterone's effect on hypothalamus).
Should conception occur, this higher temperature is maintained throughout pregnancy.

Breast changes: very sensitive to sex hormones (estrogen & progesterone).


During the normal cycle, breast swelling occurs in the luteal phase in response to increasing
progesterone levels.

Psychological changes: changes in mood with increased emotional lability in late luteal phase (falling
levels of progesterone).

For menstrual cycle regulation see question 10/A

3.B Anatomy of the female genitalia


External & internal genitalia:

Blood supply:
o Perineum from internal pudendal artery.
o Uterine, Vesical & vaginal arteries from internal iliac artery.
o Ovarian vessels from abdominal aorta (inside suspensory ligament).

Innervation:
o Perineum from pudendal nerve (Sacral plexus (S2, S3, S4)).
o Uterus & Vagina from hypogastric nerves & Sacral plexus.
o Ovary from ovarian plexus.

18
19
3.C Positio alta occipitalis anterior
Definition:

Malpresentation, the head is high above the pelvic inlet during the first stage of labor.

Head position is with the sagittal suture in the "anterior/posterior" axis.

There is a delayed labor (non progressing) & C section is usually the way to deliver safely.

Triplet 4

4.A Gynecological examination of women (medical history, physical


examination, digital examination)
Gynecological history:
Usual history-taking with questions about the presenting complaint, its history & associated
problems.
Past medical history, drugs & allergies.
Social status, family history & general systemic enquiry.
Menstrual cycle, fertility, pelvic pain, urogynaecological & obstetric - histories.
Menstrual history: The pattern of bleeding?
o The simple phrase ‘tell me about your periods’...
o The bleeding pattern of the menstrual cycle is expressed as a fraction. A cycle of 4/28
means the woman bleeds 4 days every 28 days.

Bleeding too little (Amenorrhoea): is the absence of periods.

Primary amenorrhoea: no menstruation by age 16.

Secondary amenorrhoea: no menstruation > 6 months.

Oligomenorrhoea: infrequent periods (> 42 days cycle).

Climacteric: peri-menopausal time when periods become less regular & increasing
menopausal symptoms.

Menopause: time after last ever period (assessed retrospectively).

Bleeding too much (menorrhagia): Loss > 80 ml during regular menstruation.


It is very difficult to accurately assess, how heavy the periods are.

Blood clots are not normal!


Symptoms of anaemia may also be present.

A history of the menstrual cycle since menarche (the first period) can reveal changes in the bleeding
pattern.

20
Bleeding at the wrong time It is important to ask specifically about bleeding, brown, or bloody
discharge between periods (inter-menstrual bleeding), or after intercourse (postcoital bleeding). Can
point to abnormalities of cervix or uterine cavity.

Postmenopausal bleeding (PMB): bleeding > 1 year after the last period.

Undiagnosed abnormal bleeding requires further investigation!

Fertility history:

Last menstrual period (LMP) This question is vital & should be followed with whether that
period came at the expected time (normal character?).
Contraception: sexually active? contraception?
o Problems with chosen contraceptives & why they were stopped.
Postmenopausal: hormone replacement therapy? any symptoms of menopause?

Cervical smears: Women age's 20 - 64 are invited for cervical screening every 3 - 5 years.
Any previous abnormalities should be noted & colposcopic investigation or treatment?.
If > 50, it may be relevant to discuss breast screening.

Pelvic pain history: Painful periods (Dysmenorrhoea) is a common problem & its effects on lifestyle
is important.

The cramping pain of primary dysmenorrhoea is most intense just before & during the early
stages of a period.
Young women are particularly affected & the pain has usually been present from the time of
the first period.
Usually not associated with structural abnormalities and may improve with age or after a
pregnancy.

Secondary dysmenorrhoea: menstruation was not painful in the past & more likely indicates pelvic
pathology.

Progressive dysmenorrhoea: the intensity of the pain increases throughout menstruation, may
suggest endometriosis.

Pain before or during periods is likely to be of gynecological origin.

It is vital to take a urinary & lower gastrointestinal history as urinary tract infection or
irritable bowel syndrome may present with pelvic pain.

Any pain is likely to be worse if the person is anxious, stressed or depressed.

Chronic pelvic pain is particularly affected by psychosomatic factors.

Pain on intercourse (dyspareunia) has two main types (superficial & deep).

Deep dyspareunia: associated with pelvic pathology that restrict uterine mobility (scarring,
adhesions, endometriosis, masses).

Superficial dyspareunia: from local abnormalities at the vaginal orifice or 2 low lubrication.

21
It can also be due to a voluntary or involuntary contraction of the muscles of the pelvic floor
referred to as ‘vaginismus’.

Vaginal discharge: normal or associated with cervical ectopy & if offensive or irritant, can indicate
infection.
It can also suggest neoplasia of the cervix or endometrium.
Enquire about the duration, amount, color, smell & relationship to cycle.

Urogynaecological history: Urinary incontinence? Incontinence after exercise, coughing, laughing or


straining can suggest stress incontinence.

Other urinary symptoms: frequency & nocturia? dysuria or haematuria may suggest bladder
infection or pathology.

Prolapse: vaginal discomfort, dragging sensation, feeling of something ‘coming down’.

The Uterus & vaginal walls (anterior & posterior) can prolapse.

I. Gynaecological examination:

Signs of gynecological disease are not limited to the pelvis (anaemia, pleural effusions,
visual field defects, lymphadenopathy).
Done by speculum, taking a cervical smear & bimanual pelvic examination.
A female chaperone should always be present!
The examination requires full explanation & verbal consent.

II. Inspection: hair distribution & vulvar skin.

The vulva can be a site of chronic skin conditions (eczema, psoriasis, warts, cysts of the
Bartholin’s glands & cancers).
Ulceration may imply herpes, syphilis, trauma or malignancy.
Look at the perineum & gently part the labia to inspect the vaginal orifice.
Perineal scars are usually secondary to tears or episiotomy during childbirth.
Asking the woman to cough may reveal stress incontinence or the bulge of a prolapse.

III. Physical examination:

1. Speculum examination:

Inspect the vagina for atrophic vaginitis & discharge. A creamy or mucous discharge
is normal.

A yellow-greenish frothy discharge is seen with Trichomonas vaginalis.

A grey-green fishy discharge suggests bacterial vaginosis.

Purulent cervical discharge with gonorrhoea.

Increased mucous discharge with chlamydial cervicitis.

22
2. Vaginal Swabs: should be taken from the vaginal fornices or the cervical canal (endocervical).

The cervical os is small & round in the nulliparous, bigger & more slit-like in parous
women.

Inspect cervical mucosa for any changes...

Threads from an IUCD may be present.

3. Taking a cervical smear: in mid - late follicular phase (not during menstruation).

The most commonly used technique involves using liquid-based cytology and a
broom-type sampling device (endocervical + ectocervix).

4. Check for prolapse:

Coughing will show a bulge of the posterior wall if a rectocele is present.

When the posterior wall is held back, coughing will demonstrate the bulge of a
cystocele and/or uterine descent.

23
Digital examination:

1. Pelvic examination:
Apply lubricating gel to the gloved fingers of the right hand. Part the labia with the index and middle
fingers of the left hand. Gently slip the right index finger into the vagina. If comfortable, slip the
middle finger in below the index finger, making room posteriorly to avoid the sensitive urethra.

The cervix feels like the tip of a nose and protrudes into the top of the vagina.

2. Digital pelvic examination: palpate the cervix & record irregularities or discomfort.

3. ‘Cervical excitation’ is when touching the cervix causes intense pain & implies active pelvic
inflammation.

Assess the position of the uterus. It is usually anteverted (cervix is posterior &
uterine body anterior).

Retroverted uterus (cervix is anterior & uterine body posterior).

The fingers should be manipulated behind the cervix to lift the uterus. With the
left hand above the umbilicus, feel through the abdomen for the moving uterus.

Bimanual examination: Examination of the uterus & adnexa.


Assess mobility, regularity & size of uterus.
The adhesions of endometriosis, infection, surgery or malignancy fix the uterus & make the
bimanual examination more uncomfortable.
Asymmetry of the uterus may imply fibroids.
Feel for adnexal masses in the vaginal fornices lateral to the cervix on each side.

4.B Hyperprolactinemia and galactorrhea


Hyperprolactinemia Definition:

Presence of abnormally high levels of prolactin in the blood.

Normal < 500 mIU/L [20 ng/mL].

Prolactin is a hormone produced by the pituitary gland that is primarily associated with
lactation & plays a vital role in breast development during pregnancy.

Clinical presentation:

> Typical history of oligomenorrhea, amenorrhea or infertility, which generally results from
prolactin suppression of gonadotropin-releasing hormone (GnRH) & galactorrhea
(spontaneous flow of breast milk).

May be a part of normal body changes during pregnancy & breastfeeding.

May be caused by diseases affecting the hypothalamus & pituitary gland, drugs, medicinal
herbs, heavy metals, diseases of liver, kidneys, ovaries & thyroid.

24
Nonpuerperal hyperprolactinemia: pituitary adenomas produce prolactin (prolactinomas).

Diagnosis:

Secretion is pulsatile: it increases with sleep, stress, pregnancy & chest wall stimulation or
trauma.

Must be drawn after fasting.

The presence of a pituitary tumor may cause visual-field defects or headache.

Treatment:

Asymptomatic Patients (idiopathic or microprolactinoma) can be monitored without


treatment.

Depending on etiology...
o Hypothyroidism should be given thyroid hormone replacement therapy.
o Surgery/radiotherapy for tumors.
o Estrogen replacement.
o Change drug causing the problem.

Bromocriptine (dopamine agonist).

Complications:
Persistent associated hypogonadism can lead to osteoporosis, blindness, hemorrhage &
infertility.

Galactorrhea definition:
A milky nipple discharge unrelated to the normal milk production, also in those who have
never had children or after menopause.

Not a disease, but it could be a sign of an underlying problem.

Excessive breast stimulation, medication side effects or disorders of the pituitary gland all
may contribute to galactorrhea.

Often from hyperprolactinemia.

May be idiopathic.

Clinical presentation:
Persistent or intermittent milky nipple discharge (no blood).

Nipple discharge involving multiple milk ducts.

Spontaneously leaked or manually expressed nipple discharge.

One or both breasts affected.

25
Absent or irregular menstrual periods.

Headaches or vision problems (pituitary tumor).

Diagnosis:
A physical exam for breast lumps & suspicious areas.
Analysis of fluid discharge (fat droplets present help confirm the diagnosis of galactorrhea).
A blood test (prolactin & TSH levels).
A pregnancy test.
Mammography & USG.
Head CT/MRI (pituitary gland).

Treatment:
Cause dependent.

Complications:
Cause dependent.

4.C Development of the placenta and abnormality of placental


development
The interface between mother & fetus to fulfill critical roles such as preventing allograft rejection of
fetus, fetal nutrient supply, respiratory gas exchange & transfer of toxic metabolic waste into the
maternal blood circulation for elimination.

The placenta also functions as an endocrine organ producing steroid and protein hormones for the
protection of pregnancy.

26
1. Placental abruption:

Premature separation of placenta from uterus (> 20w - before birth).

A significant cause of third-trimester bleeding associated with fetal + maternal morbidity &
mortality.

Patients with this condition typically present with bleeding, uterine contractions & fetal
distress.

Classification (class 0-III):

Based on extent of separation (partial vs. complete) & location of separation (marginal vs.
central).

Etiology unknown!

Risk factors:
- Maternal hypertension (most common).
- Maternal trauma.
- Smoking.
- Drug abuse.
- Sudden decompression of uterus (premature rupture of membranes, birth of the first
twin...).
- Chorioamnionitis.
- Previous placental abruption.
- Subchorionic hematoma.

27
Presentation:

One or more of the following (may be asymptomatic):


- Vaginal bleeding.
- Contractions.
- Abdominal tenderness (even uterine tetany).
- Decreased fetal movement.
- Idiopathic premature labor.
- Fetal death.

Diagnosis:

No digital examination on a bleeding pregnant patient!!!

Pelvic examination in placenta previa can produce profuse vaginal bleeding.

History of hypertension or any other risk factor.

Shocked (hypotensive), pregnant & obvious vaginal bleeding... may be placental abruption.

USG will show fetal movements, vital signs, location of placenta, bleeding.

External fetal monitor.

Blood test (CBC, clotting profile, BUN).

Management: not in emergency settings.

Continuous external fetal monitoring.


Fluid resuscitation (crystalloids or blood).
Vaginal delivery (preferred if fetus died).
C section: often necessary for fetal or maternal stabilization.
Hysterectomy: uncontrolled bleeding.. last step after correction of coagulopathy, ligation of
the uterine artery, administration of uterotonics...

Complications:

Hemorrhage into the decidua basalis, usually followed by vaginal bleeding.


Compromised fetal blood supply (due to compression).
Penetration of blood through the thickness of the uterine wall.
Uterine rupture.
Miscarriage.
Maternal hypovolemic shock.

Prognosis:

Class 0 - I good prognosis.


Class III fetal & maternal death may follow.

2. Placenta Praevia

28
A rare, obstetric complication in which the placenta is inserted partially / wholly in the lower
uterine segment. Because of placental migration during pregnancy the diagnosis is made by
USG, only > 34w.

A leading cause of antepartum hemorrhage (vaginal bleeding).

Classification:

Minor: in lower uterine segment, but does not cover the internal os.

Major: In lower uterine segment& covers the internal os.

Risk factors: uterine scars (previous C section, surgery, trauma, uterine cancer, infection), drug
abuse, previous placenta previa, idiopathic.

Clinical presentation: often present with painless, bright red vaginal bleeding (usually ~32w). This
bleeding often starts mildly & may increase as the area of placental separation increases.

Should be suspected if there is bleeding > 24w.

Diagnosis: USG (> 34w).

Treatment: depends on presentation and progression.

Any danger to fetus / mother... Emergency C section.

If no acute danger & bleeding stops... Continue as a risk pregnancy until term & then: In minor -
vaginal delivery possible & in major - elective C section.

Complications:

Maternal: Antepartum hemorrhage, Malpresentation, Abnormal placentation&Postpartum


hemorrhage.

Increased risk of puerperal sepsis &postpartum hemorrhage because the lower segment, contracts
less well postpartum.

Fetal: IUGR, Premature delivery&Death.

3. Placenta accrete/incretta/percretta See Question 13/B

29
Triplet 5

5.A Colpitis and vaginosis


Colpitis (Vaginitis): Inflammation of the vagina.

➢ May result in abnormal discharge, itching, pain, often associated with an irritation / infection
of the vulva, dyspareunia (pain during sex).

➢ Usually due to infection.

1) Bacterial vaginosis (BV): infection & Inflammation of the vagina.

➢ Disturbance between normal flora & pathogens.

➢ Risk factor for getting an STD & HIV.

➢ Douching (washing the vagina), Sex & multiple sex partners increases the risk of BV.

Etiology unknown!

Clinical presentation: may by asymptomatic. Abnormal vaginal discharge, fish like odor,
itching, burning during urination.

Diagnosis: clinical presentation + smear test.

Treatment: depending on test results / antibiotics (metronidazole / Clindamycin).

Complications: other superinfection / STD, premature labor, PID & infertility.

2) Yeast infections: usually by Candida albicans. Same as above.

Clinical manifestation: mainly itching & white, thick discharge that looks like cottage cheese.

Treatment: anti fungal ("azole" family), miconazole / fluconazole.

3) Trichomoniasis (parasite, STD): same as above.

Treatment: metronidazole.

4) Vaginal atrophy (atrophic (non infectious) vaginitis): same as above.

Vaginal sprays, douches, perfumed soaps, scented detergents & spermicidal products may
cause an allergic reaction or irritate vulvar & vaginal tissues. Thinning of the vaginal lining —
a result of decreased hormone levels (estrogen) following menopause / surgical removal of
your ovaries — can also cause vaginal itching & burning.

Treatment: topical estrogen & avoid irritating factors.

30
5.B Life Cycle of a Woman
The 7 ages of women:

Intrauterine.
Neonatal (< 28 day).
Childhood (< puberty).
Adolescent (<18).
Fertile (< 45).
Climacterium, menopause (45 - 55).
Senium (> 55).

Intrauterine age:

Normal vs. abnormal (congenital abnormalities).

Genetic disorders – Down, Edward, Patau, Triploidy, XO, XXY, XXX...).

Structural disorders – congenital heart disease, NTD, abdominal wall defects, genitourinary
abnormalities, lung disorders...

Congenital infections – Toxoplasmosis, Rubella, CMV, HSV, HIV, HCV, HBV, syphlilis....

Prenatal diagnosis: Serological screening (triple test-AFP, HCG, estriol), USG (nuchal translucency),
amniocentesis, chorionic villus sampling.

Neonatal age:

After birth hypoxia, Apgar score (max 10) – skin color, muscle tone, response to stimulation.

Physical birth injury – brachial plexus, facial nerve, skeletal, soft tissue, caput succedaneum,
cephalic hematoma.

RDS (respiratory distress syndrome) by a deficiency of surfactant in lungs (premature).

Infections.

Surgical correction of fetal abnormalities.

Hyperbilirubinemia.

Diagnosis of metabolic disorders - Phenylketonuria, Congenital Hypothyroidism, Cystic


Fibrosis (CF).

Childhood:

Development of skeleton, body, endocrinology.

31
Early diagnosis of genetic disorders & abnormal genital tract development.

Genitalia trauma.

Alien body in vagina.

Infections of vulva & vagina.

Violence (abuse) – somatic & psychical.

Normal puberty (telarche, pubarche, menarche).

Adolescent growth spurt.

Acquisition of secondary sexual characteristics.

Onset of menstruation (menarche).

Establishment of ovulatory function.

Pathological puberty:

Precocious puberty – signs of sexual maturation < 8 years.

Delayed puberty – absence of physical manifestations of puberty by 13 years.

5.C Birth shock


In the mother:
Postpartum Shock state due to complications.

In the newborn:
Transient depression of muscle tone & deep tendon reflexes in newborns after birth.
Usually lasts < 24 - 48 hours.
May happen in vaginal or cesarean deliveries.

Worldwide, 1 woman dies every minute, every day from a complication of pregnancy!

In developed countries, maternal death is uncommon.

Major causes of mortality:

1. Unexpected collapse
2. Amniotic fluid embolism.
3. Retained placenta.
4. Uterine inversion.
5. Uterine rupture.
6. Hemorrhage.

32
7. Eclampsia.
8. Pulmonary embolism (PE).

Remember that there are often two lives at stake & in most emergencies minutes or even seconds
count.

‘Resuscitate the mother & you will resuscitate the fetus’.

An obstetric emergency can cause profound lifelong psychological problems for both the mother &
partner. This can manifest itself as postnatal depression, post-traumatic stress syndrome & fear of
becoming pregnant.

The aim is to resuscitate the mother & then (and only then) to consider the welfare of the fetus.

4 “H”:
1. Hypoxia,
2. Hypovolemia,
3. Hypo/hyperkalaemia,
4. Hypothermia.

4 “T”:
1. Thromboembolism,
2. Toxic (including anesthesia),
3. Tamponade,
4. Tension pneumothorax.

Eclampsia (including magnesium toxicity).

Amniotic fluid embolus.

The decision for peri-mortem C section should be made by 4 minutes if there is no


response to active resuscitation & the delivery by 5 minutes (the ‘4-minute rule’).

Amniotic fluid embolism:

One of the most catastrophic conditions that can occur in pregnancy (rare).

Etiology unknown:

Breakdown occurs in the barrier separating mother & fetus, allowing a bolus of amniotic
fluid to enter the maternal circulation, moves to the pulmonary circulation & produces
massive perfusion failure, bronchospasm & shock.

May be an anaphylactoid reaction to fetal antigens entering the maternal circulation.

Can occur at any time in pregnancy but it mostly occurs in labour, after vaginal delivery &
following C section.

Risk factors:
- Multi-parity,
- placental abruption,

33
- intrauterine death,
- precipitate labour,
- termination of pregnancy,
- abdominal trauma,
- amniocentesis.

Clinical presentation:

o Usually develops almost instantaneously & the diagnosis must be considered in all
collapsed obstetric patients.
o Some or all signs & symptoms.
o Classically a woman in late stages of labour or immediately postpartum starts to
gasp for air, starts fitting & may have a cardiac arrest.
o There is often a profound disseminated intravascular coagulopathy (DIC) with
massive hemorrhage, coma & death.
o There are inevitably signs of fetal compromise.
o Chills, Shivering, Sweating, Anxiety, Coughing, Cyanosis, Hypotension,
Bronchospasm Tachypnoea, Tachycardia, Arrhythmias, Myocardial infarction,
Seizures, DIC.

Diagnosis:

Autopsy.
In a surviving patient by finding fetal squames in washings from the bronchus or in a
sample of blood from the right ventricle.
In the acute situation, as there is no single clinical or laboratory finding which can diagnose
or exclude. The diagnosis is made clinically by exclusion.

Management:

This is primarily supportive & should be aggressive.

There is, however, no evidence that any specific type of intervention significantly improves
maternal prognosis.
Initial therapy to support cardiac output & management of DIC.

If the woman is undelivered, an immediate C section, providing the mother can be


stabilized.

A chest X-ray often show pulmonary edema & increase in right atrial + ventricular size

The ECG demonstrates right ventricular strain & there is metabolic acidosis.

Aggressive fluid replacement, maintenance of cardiac output with a dopamine infusion,


treatment of anaphylaxis with adrenaline (epinephrine), salbutamol, aminophylline and
hydrocortisone.

Treatment of DIC with fresh frozen plasma & cryoprecipitate treatment of hemorrhage
after delivery with Syntocinon, ergometrine, carboprost (Haemabate) or misoprostol &
uterine massage.

34
Prognosis:

Neonate: very poor, mortality rate ~ 60% & survivors usually suffer neurological
impairment.

Maternal: outcome in mothers who have suffered a cardiac arrest is complicated by the
fact that many are left with serious neurological impairment.

Venous thromboembolic disease:

In pregnancy the balance of the clotting system is altered towards clot formation.

Increased levels clotting factors & reduced levels of anticoagulants.

The gravid uterus causes a degree of mechanical obstruction to the venous system & leads
to peripheral venous stasis in the lower limbs.

Venous thromboembolic disease is the commonest direct cause of maternal mortality in


the UK (> 50% of deaths occur antenatally, mostly 1st trimester).

Thromboembolism may be asymptomatic but usually presents with symptoms of calf


tenderness, cough & chest pain.

Essential to make a definitive diagnosis, not just for the current pregnancy but also for next
pregnancies.

Hematological testing (blood test) for D-dimers is not helpful in pregnancy!

Doppler USG is particularly useful for identifying femoral veins.

It is safe and should be the first-line investigation.

X-ray venography is more specific (radiation exposure!).

Pregnancy is not a contraindication to chest X-ray and/or a ventilation–perfusion scan –any


radiation risks are outweighed by the benefits of accurate diagnosis.

A normal scan virtually excludes the diagnosis of pulmonary embolism.

A CT pulmonary arteriogram (CTPA) may also be appropriate, especially if a large


pulmonary embolism is suspected.

Treatment:

DVT or pulmonary embolism (PE) in pregnancy is with intravenous (i.v.) or subcutaneous


(s.c.) heparin, continued into labour.

Continuing anticoagulation for 6–12 days.

Once anticoagulants are stopped, women should be screened for thrombophilias.

35
Thromboembolic risk factors in pregnancy & postpartum:

Age > 35 years, Obesity (> 80 kg), 4th pregnancy or more, Gross varicose veins, Current
infection, Pre-eclampsia, Immobility prior to surgery (> 4 days), Major current illness (heart
or lung disease), cancer, inflammatory bowel disease, nephrotic syndrome, Caesarean
section, particularly emergency caesarean section, Extended major pelvic or abdominal
surgery (hysterectomy), Women with a personal or family history of deep vein thrombosis
(DVT), pulmonary embolism (PE) or thrombophilia, paralysis of lower limbs,
antiphospholipid antibody.

Breathlessness and syncopal episodes are present in 90% of normal pregnancies, atrial ectopic
beats are common, and up to 96% of normal women may have an audible ejection systolic
murmur.

Potential hemodynamic problems? A very close follow-up by a multidisciplinary team is mandatory


and a careful plan should be made for delivery. Serious consideration of pregnancy termination is
advisable in women with Eisenmenger’s syndrome, primary pulmonary hypertension, or
pulmonary veno-occlusive disease.

With atrial fibrillation, anticoagulation is required to prevent atrial clot forming and subsequent
embolic problems.

Severe cardiac disease can cause problems at delivery, particularly in those with prosthetic valves,
aortic stenosis, severe mitral stenosis, left ventricular dysfunction, or pulmonary hypertension.

Cardiac disease and delivery Labour should be conducted in a high-dependency or intensive care
unit, possibly with central venous catheter monitoring, aiming for a vaginal delivery.

Particular care is required in the immediate postpartum period as the increased circulating volume
following uterine retraction may lead to fluid overload and congestive failure.

Myocardial infarction (MI) is rare in pregnancy but is the commonest cardiac cause of maternal
mortality.

Peripartum cardiomyopathy (rare), associated with hypertension in pregnancy, multiple


pregnancy, high multi-parity & increased age.

Presents with sudden onset of heart failure and on chest radiology or echocardiography - grossly
dilated heart.

Retained placenta:

Failure to deliver the placenta within 30 minutes of delivery.

Increases risk of postpartum hemorrhage x10 due to inability of the uterus to contract.

May be severe & life-threatening, particularly in partial separation.

More likely with preterm gestations, after a previous caesarean section & a morbidly
adherent placenta (accreta).

36
Failure of the placenta to deliver may occur because of an unusually adherent un-
separated placenta, or because the placenta has separated successfully but is retained
within the uterus by a partially closed cervix.

Risk factors for placenta accreta:

Previous retained placenta, multi-parity, Advanced maternal age, Placenta praevia,


Previous caesarean section, History of dilatation & curettage or suction termination of
pregnancy, Previous postpartum, endometritis.

Morbidly adherent placenta has three options: placenta accreta (superficial myometrium),
placenta increta (deep myometrium) and placenta percreta (into pelvis & organs).

Partial separation or iatrogenic effort at separation may lead to profound hemorrhage.

Treatment:

Bleeding heavily?
o A retained placenta is an obstetric emergency & treatment must be immediate.

o Aside from the initial resuscitation measures (above) the patient should be
transferred to theatre for a manual removal of placenta.

No bleeding?
o Conservative approach. I.V access should be established & crossmatch arranged in
case bleeding begins, and it is reasonable to wait an hour or so for spontaneous
expulsion of the placenta to occur.

o In the interim the use of Syntocinon, the ‘rubbing-up’ of a contraction, or


breastfeeding, with its resultant physiological release of oxytocin, may help to aid
expulsion.

o If the placenta is still retained after 1 hour, go to operation theatre.

o The placenta can then be gently stripped off the uterine wall and delivered.

o Once it is out, a contraction should be ‘rubbed-up’ and a bolus of Syntocinon given


i.v. to reduce the risk of postpartum hemorrhage due to an atonic uterus.

o Give antibiotics as there is a significant association between manual removal of the


placenta and postpartum endometritis.

o Removal impossible or dangerous (uterine rupture)?

o The clinical diagnosis is ‘placenta accreta’.

Management depends on degree of hemorrhage.

Persistent uncontrollable hemorrhage? Hysterectomy is required.

37
No active hemorrhage? Suction curettage or conservative management are options.

Conservative management: Placenta is left in situ to be absorbed over time, with a


significant incidence of major complications from infection & bleeding.

Triplet 6

6.A Cancer screening in gynecology


Cervical smears (PAP test):

Women of 20 - 64 years are invited for cervical screening every 3 - 5 years.

The date of the woman’s last smear should be noted & when recommended for next smear.

Any previous abnormalities should also be noted & whether she has had any colposcopic
investigation or treatment.

If she is over 50, it may be relevant to discuss breast screening.

HPV (human papilloma virus ) test: woman > 30 together with PAP test.

During scheduled gynecological examinations the inspection (palpation) of the cervix and
external genitalia is done.

Genital warts are a sign of HPV.

6.B Amenorrhea
Definition:

Failure of menstruation to occur at the expected time.

May be Primary & secondary.

Clinical presentation:

I. Primary amenorrhoea:

When menstruation has never occurred (no menstruation by age 16).


The likely cause of primary amenorrhoea depends on whether secondary sexual
characteristics are present or not.

If secondary sexual characteristics are absent, then the cause is most likely delayed puberty.

In normal pubertal development, anatomical causes should be suspected.

The main ‘anatomical’ causes:

38
Congenital absence of the uterus.

Imperforate hymen: Menstrual blood is retained within the vagina causing cyclical lower
abdominal pain each month at the time of menstruation (cryptomenorrhoea).
o Treatment by incision, usually under anaesthesia.

Physiological delay: Development is normal but delayed onset of menstruation. Often


positive family history.

A progestogen challenge test is useful to identify constitutional menstrual delay.

A progestogen (e.g. norethisterone) is given orally for 5 days, and withdrawal should lead to
a vaginal bleed. If such a bleed occurs, it is reasonable to offer reassurance that spontaneous
menstruation is likely to occur.

An abdominal ultrasound may be reassuring to confirm that the uterus and ovaries are
normal.

Low body weight and excessive exercise are also associated with primary amenorrhoea.
The other causes listed below are rare.

39
➢ A progestogen challenge test is useful to identify constitutional menstrual delay.

➢ A progestogen is given orally for 5 days & withdrawal should lead to a vaginal bleed. If such
a bleeding occurs, spontaneous menstruation is likely to occur.

➢ Abdominal USG: confirm that the uterus & ovaries are normal.

➢ Low body weight & excessive exercise are also associated with primary amenorrhoea.

Treatment of primary amenorrhoea:

Depends on the etiology.

Complications of primary amenorrhoea:

II. Secondary amenorrhoea:


Established menstruation ceases > 6 months in absence of pregnancy.

III. Physiological amenorrhoea:

Pregnancy and lactation.

o Pregnancy should be excluded.

40
The high postpartum level of prolactin associated with breast feeding suppresses ovulation &
the gonadotrophin-releasing hormone (GnRH).

IV. Hypothalamic amenorrhoea (‘hypogonadotrophic hypogonadism’):

Frequently associated with stress & usually resolves spontaneously.

Physical stress in the form of athletic training can also result in suppression of the
hypothalamo-pituitary–ovarian axis. There are low levels of pituitary gonadotrophins in
association with low levels of prolactin and oestrogen.

If pregnancy is not desired, oestrogen replacement therapy is advisable (oral contraceptive


pill).

If the woman wishes to become pregnant, ovulation may be induced by GnRH therapy or
exogenous gonadotrophins.

41
V. Anorexia nervosa:

Should be considered. Restoration of the body weight results in the return of ovulatory
function, although there may be a significant time interval between them.

Ovulation induction therapy is not recommended before restoration of body weight, as


pregnancy, carries the risk of growth restriction of the fetus and increased perinatal
mortality.

VI. Pituitary amenorrhoea:

Prolactin stimulates breast development and subsequent lactation.

High levels of prolactin, which may be either physiological (during lactation) or pathological
in turn suppress ovarian activity & gonadotrophin's.

Mildly elevated prolactin levels are common and may be due to stress.

Sustained higher levels can result in amenorrhoea and galactorrhea.

Causes of hyperprolactinaemia: Pituitary adenoma, Micro & Macroadenomas.


Secondary: Primary hypothyroidism, Chronic renal failure, Pituitary stalk compression, PCOS
(polycystic ovary syndrome) Drugs (phenothiazines, haloperidol, metoclopramide,
cimetidine, methyldopa, antihistamines & morphine).

If prolactin > 1000 mU/l head CT or MRI should be done.

Visual fields should be checked as optic chiasma compression may lead to bitemporal
hemianopia.

All patients should have pituitary imaging before treatment.

This treatment is usually with a dopamine agonist (bromocriptine or cabergoline) that


suppresses prolactin level and induces regression of the prolactinoma.

Microsurgical excision of an adenoma (rare).

VII. Ovarian amenorrhoea:

Premature ovarian failure: Menopause (50 years). The term ‘premature ovarian failure’ is
used to describe cessation of ovarian function < 40 years.

As in the natural menopause, failure is usually due to depletion of primordial follicles in the
ovaries.

~ 1% of women and may be due to surgery, viral infections (mumps), drugs, radiotherapy,
idiopathic, chromosomal abnormality (XXX) & autoimmune disorders.

Pregnancy by in vitro fertilization (IVF) with donor oocytes may be possible.

42
Hormone replacement therapy is required to relieve postmenopausal symptoms and
minimize the risk of osteoporosis.

VIII. Polycystic ovary syndrome (PCOS):

Associated with menstrual disturbance & most common form of anovulatory infertility.

Characterized by the presence of at least two out of the following three criteria: oligo-or
amenorrhoea, ultrasound appearance of large-volume ovaries & multiple small follicles.

clinical evidence of excess androgens (acne, hirsutism) or biochemical evidence (raised


testosterone).

Associated with the prevalent insulin resistance, there is a characteristic dyslipidaemia and a
predisposition to non-insulin-dependent diabetes & cardiovascular diseases.

PCOS may be considered a systemic metabolic condition rather than primarily of


gynecological origin.

Treatment depends on whether the presenting problem has been menstrual irregularity,
hirsutism or infertility.

The combined oral contraceptive pill has been used to regulate the menses & hirsutism.

Complications: depending on the etiology (infertility, postmenopausal symptoms, osteopenia,


osteoporosis).

6.C Preeclampsia
Definition:

Hypertension associated with proteinuria & edema, occurring primarily in first pregnancy &
after 20th GWs.

A multi system disorder.

Hypertension is usually the first manifestation, followed by proteinuria.

Clinical presentation:

BP > 140/90 (In 2 measurements at least 4 hours apart).

> 300 mg protein in 24 hour urine collection.

> 20th week gestation.

Renal insufficiency- serum/plasma creatinine >0.09 mmol/L or oliguria.

43
Liver - raised serum transaminases and/or severe epigastric/right upper quadrant pain.

Neurological problems – hyperreflexia with clonus; severe headaches with hyperreflexia;


persistent visual diaturbances (scotomata).

Haematological disturbances – thrombocytopenia, DIC.

In a previously normotensive women & resolving completely after 2 months.

Treatment:
Magnesium.
The only real treatment for pre-eclampsia & eclampsia is delivery.

Complications:
Eclampsia and its complications.
A prolonged fit, can lead directly to maternal death.
Cerebral hemorrhage, renal or hepatic failure, respiratory failure, coagulopathy or HELLP
syndrome.

Triplet 7

7.A Treatment of endometriosis – associated infertility


Endometriosis: endometrial tissue that grows outside the uterus.

Infertility: the inability to conceive & unstable to carry a pregnancy to full term.

Diagnosis of endometriosis is by history, physical examination, USG & laparoscopy (visual & biopsy).

In younger women with unfulfilled reproductive potential, surgical treatment removes endometrial
tissue & preserves the ovaries (without damaging normal tissue).

An infertile woman may be treated expectantly after surgery, with fertility medication / IVF.

Ablation of endometriosis has shown high rate of short-term recurrence after the procedure.

The best surgical procedure with much less rate of short-term recurrence is to cut & remove the
lesions completely.

Surgery:

➢ Conservative treatment consists of the excision of the endometrium, adhesions, resection of


endometriomas, & restoration of normal pelvic anatomy.

➢ 55% - 100% of women develop adhesions following pelvic surgery which can result in
infertility, chronic abdominal, pelvic pain, & difficult reoperative surgery.

➢ Temporary ovarian suspension, for a week after surgery may be used to reduce the incidence
of adhesions after endometriosis surgery.

44
➢ A hysterectomy is used to treat endometriosis in women who do not wish to conceive.
However, this should only be done when combined with removal of the endometriosis by
excision, as if endometriosis is not also removed at the time of hysterectomy, pain may
persist.

Hormonal treatment:

➢ Progesterone / Progestins: Progesterone counteracts estrogen that inhibits the growth of the
endometrium.

➢ Such therapy can reduce or eliminate menstruation in a controlled & reversible fashion.

➢ Gonadotropin-releasing hormone (GnRH) agonists: thought to work by decreasing hormone


levels. More effective for pain relief in endometriosis than no treatment / placebo.

➢ An ideal therapy would diminish inflammation & underlying symptoms without being
contraceptive.

➢ Surgery is more effective than medicinal intervention for addressing infertility associated
with endometriosis.

➢ In-vitro fertilization (IVF) procedures are effective in improving fertility in many women with
endometriosis.

➢ Vaginal childbirth decreases recurrence of endometriosis.

Complications:

➢ Internal scarring, anatomical distortions, adhesions, pelvic cysts, chocolate cyst of ovaries,
ruptured cysts, bowel & ureteral obstruction resulting from pelvic adhesions.

7.B Fertilization and fetal development


The sperm has only three essential roles in embryogenesis:

1. stimulates a change that prevents further sperms from entering the cell.

2. Deliver a paternally haploid genome.

3. Stimulate the second meiotic division in the egg.

Imprinting:

The male and female gamete both contain 23 chromosomes, one copy of each autosomal
chromosome & one sex chromosome (X in egg & X / Y in sperm).

45
Fertilization occurs in the fallopian tube.

The embryo is transported along the fallopian tube & implants in the uterine wall ~ 6 gestational day
(GD).

At implantation the trophoblast buries itself in the endometrium & gains access to the maternal
circulation, enables a very rapid period of growth.

The amniotic cavity begins to form by 9 GD.

Organogenesis (19 – 56+ GD):

1. Neural tube & brain (19 GD - 8 years).


Forebrain, midbrain, hindbrain, spinal cord.

2. Gut tube & derivatives.


Thyroid & pituitary.
Lungs.
Pancreas & biliary system.
Urogenital system.
Sex can be determined (18w).

3. Heart & liver.


First heart beat (6w).

4. Craniofacial structures.

5. Limbs & skeletal muscle.

7.C Puerperal infections


Definition:

Also known as postpartum infections, puerperal fever or childbed fever, is any bacterial
infection of the female reproductive tract following childbirth or miscarriage.

Clinical presentation:
Usually: fever > 38.0 °C, chills, lower abdominal pain & bad-smelling vaginal discharge.

Usually occurs between 24H - 10 days, from delivery.

The most common infection is that of the uterus & surrounding tissues.

Risk factors:
Cesarean section, group B streptococcus in vagina, premature rupture of membranes
(PROM), & prolonged labour.

Diagnosis:
By clinical symptoms.

46
Differential Diagnosis: breast engorgement, UTI, infections of the abdominal incision or episiotomy
& atelectasis.

Treatment:
Risk factors must be included.
Ampicillin + gentamicin (vaginal delivery) or clindamycin + gentamicin (C-section).

Complications:
Like any other infection, related to the site & etiologic factors.
Spreading of the infection, Sepsis, shock, bleeding...

Triplet 8

8.A Menstruation and menstrual disorders


Definition:

The endometrium: has a superficial layer, shed during menstruation & a basal layer,
regenerates the superficial layer.

Menstrual phase: Normally, the luteal phase of the ovary lasts for 14 days, at the end of
which regression of the corpus luteum is associated with a decline in ovarian sex hormones
(estradiol & progesterone).

This fall (estrogen & progesterone) is followed by intense spasmodic contraction of the
endometrial arterioles & ischemic necrosis (shedding the superficial layer and bleeding).

The vasospasm & uterine contractions at the time of the menstrual flow is due to
prostaglandins.

Menstrual disorders:

Abnormal condition in a woman's menstrual cycle.

Categories:

- Disorders of ovulation.
- Disorders of cycle length.
- Disorders of flow.
- Dysmenorrhea.

Disorders of ovulation:

Oligoovulation: infrequent or irregular ovulation (cycles ≥36 days).

Anovulation: absence of ovulation when it would be normally expected.

47
Manifests as irregularity of menstrual periods, that is, unpredictable variability of intervals, duration,
or bleeding.

May cause stop of periods (secondary amenorrhea) or excessive bleeding (dysfunctional uterine
bleeding).

Etiology:

Hypothalamic-pituitary-ovarian axis disorders, Polycystic ovarian syndrome (PCOS),


hyperprolactinemia. Excessive exercise, stress. Very High/low BMI. Thyroid dysfunctions,
ovarian trauma, Idiopathic.

Diagnosis:

According to the clinical picture & infertility.


Blood tests for hormone levels.
Basal body temp charts.
Imaging methods: head MRI, pelvic/thyroid ultrasound.

Treatment:

According to the etiological factor...

Surgery/chemotherapy/radiation (CNS & thyroid tumors).

Hormone replacement therapy (hypothyroidism) or after thyroidectomy (hyperthyroidism).

Ovulation induction: selective estrogen-receptor modulator (clomifene citrate) &


gonadotropins (hCG, LH, FSH, GnRH).

Bromocriptine for hyperprolactinemia.

Corticosteroids in overproduction of male hormones by adrenal glands (reduce


testosterone).

Lifestyle changes: lose/gain weight (normal BMI), stress management, moderate exercise.

Disorders of cycle length:

Polymenorrhea: cycles with intervals < 21 days.


o Etiology:????????

Irregular menstruation (periods): variation in menstrual cycle length > 8 days.


o Etiology:????????

Metrorrhagia: irregular menstruation that occurs between the expected menstrual periods.

o Etiology:

48
May be a sign of an underlying disorder, hormone imbalance, endometriosis,
uterine fibroids, cancer of reproductive organs.
Due to repeated & heavy bleeding, it can cause significant iron deficiency
anemia.

Oligomenorrhea: infrequent, often light menstrual periods (intervals > 35 days).


o Etiology:
Prolactinomas (pituitary adenomas), thyrotoxicosis, hormonal changes in
perimenopause, Prader–Willi syndrome & Graves disease. Endurance sports
(long runs, cycling...).

Amenorrhea: no menstruation in a woman of reproductive age. (see question 6b).

8.B Ethical issues in gynecology, obstetrics and assisted reproduction


Violence against women.
Sex selection for non-medical purposes.
The ethical aspects of sexual and reproductive rights.
Conscientious objection.
Adolescent youth and reproductive health care & confidentiality.
Human cloning.
Patenting human genes.
Embryo research.
Sale of gametes and embryos.
Altering genes in humans.
Donation of genetic material for human reproduction.
Embryonic or fetal tissue for therapeutic clinical applications.
Ethical guidelines on multiple pregnancy.
Ethical aspects of gamete donation from known donors.
Surrogacy.
Oocyte and ovarian cryopreservation.
Brain death and pregnancy.
Caesarean delivery for non-medical reasons.
Definition of pregnancy (life?).
Management of severe congenital anomalies.
Termination of pregnancy following prenatal diagnosis.
Anencephaly & organ transplantation.
Pregnancy and HIV-positive patients.
Planned home birth.
Cord blood banking.
Female sterilization.
Ethical aspects of induced abortion for non-medical reasons.
Physicians’ relationships with industry.

49
8.C Intrauterine hypoxia, causes and diagnosis
Definition:

The fetus is deprived of an adequate supply of oxygen.

Etiology (mother, placenta, fetus):

• Maternal smoking.
• Placental infarction.
• Placental abruption.
• Umbilical cord prolapse or occlusion.

Intrauterine hypoxia can cause cellular damage to CNS.

Increased mortality rate & risk of sudden infant death syndrome (SIDS).

Primary or a secondary risk factor in disorders of CNS (epilepsy, ADHD, eating disorders & cerebral
palsy).

Origin of Hypoxia
Pre-placental Utero-placental Post-placental
Diminished uterine artery flow
Hypoxic environment Pre-eclampsia (mechanical compression,
thrombotic occlusion)
Pre-exciting maternal cardiac
Progressive fetal cardiac failure
disease
Maternal hematological
disorders – iron deficiency
anemia, Sickle cell disease and
Thalassemia

1. Maternal smoking (preventable).

A Wide variety of deleterious effects on the developing fetus.


Carbon monoxide (CO) induced tissue hypoxia & placental insufficiency reduce blood flow
from the uterus to the placenta, reducing availability of oxygenated blood to the fetus.
Placental insufficiency as a result of smoking has a causal effect in pre-eclampsia.
Smokers x5 more likely to develop pre-eclampsia.
Nicotine is a teratogen of autonomic nervous system, leading to increased susceptibility to
hypoxia-induced brain damage.
Smoking causes a decrease in maternal nucleated red blood cells (NRBC), reducing oxygen
transport options.

2. Placental infarction:

Results from interruption of blood supply to a part of the placenta, causing its cells to die.
Diagnosed by USG.
Small placental infarcts are normal at term.
Large infarcts are associated with vascular abnormalities.

50
Very large infarcts lead to placental insufficiency & may result in fetal death.
Risk factors: maternal hypertension, diabetic microangiopathy, coagolupathies.
Treatment: stop risk factors (smoking...), anti coagulants?, test for coagolupathies...

3. Placental abruption:

When the placental lining has separated from the uterus before delivery.
The most common cause of late pregnancy bleeding.
A significant contributor to maternal mortality worldwide.
Treatment depends on how serious the abruption is & week of pregnancy.
The effects on fetus depend on both its severity & gestational age.
The heart rate of the fetus can be associated with the severity.
Signs & symptoms:
o in early stages, may be no symptoms.
o sudden-onset abdominal pain.
o contractions that don't stop.
o pain in the uterus.
o tenderness in the abdomen.
o vaginal bleeding.
o pallor
o non reassuring fetal status (decreased movement, worrisome heart rate).
Risk factors:
o Pre-eclampsia.
o Chronic hypertension.
o Smoking
o Maternal trauma (accidents, assaults, falls)
o nosocomial infection.
o Short umbilical cord.
o Prolonged rupture of membranes (>24 hours).
o Thrombophilia.
o Retroplacental fibromyoma.
o Multiparity.
o Multiple pregnancy.
o Age < 20 or > 35.
o History of placental abruption/C section.
o cocaine abuse.
Women may present with vaginal bleeding (or not - concealed), abdominal or back pain,
abnormal or premature contractions, fetal distress or death.

Diagnosis:
o The diagnosis is one of exclusion, other possible sources of vaginal bleeding or
abdominal pain have to be ruled out in order to diagnose (USG).

Treatment:
o Depends on blood loss & the status of the fetus.

o If < 36 weeks & neither mother or fetus is in any distress, monitor in hospital until
change in condition or fetal maturity.

o Immediate delivery (c section) if fetus/mother are in distress.

51
o Vaginal birth is usually preferred if no fetal/maternal distress.

o Blood volume replacement to maintain BP & blood plasma replacement?

4. Umbilical cord prolapse or occlusion:

‘Cord presentation’: umbilical cord between the presenting part & membranes, before
membrane rupture.

‘Prolapsed umbilical cord’: same situation after membrane rupture (remain inside the vagina
(occult prolapse) or can prolapse outside).

More common in rupture of membranes, malpresentation, polyhydramnion, premature or


small fetus.

The danger is that pressure on the cord (from fetus) will cause cord compression that
compromises blood flow to the fetus.

The first sign is usually a sudden decrease in fetal heart rate that is severe & does not
immediately resolve.

On a fetal heart tracing, moderate to severe variable decelerations.

Whenever there is a sudden decrease in fetal heart rate or abnormal fetal heart tracing,
umbilical cord prolapse should be considered.

The possibility for fetal death & other complications, make it an obstetric emergency!

Guidelines focus on quick delivery (cesarean section)

Treatment:

Depends on the etiology.

Treatment of infants suffering birth asphyxia is by lowering the core body


temperature.

High concentrations of oxygen lead to generation of oxygen free radicals leading


to reperfusion injury.

Use normal air instead of 100% oxygen.

52
Triplet 9

9.A Pelvic pain syndrome


Possible causes (differential diagnosis):
- Appendicitis.
- Ectopic pregnancy.
- Miscarriage.
- Pelvic inflammatory disease.
- Broken pelvis.
- Endometriosis (see question 7a).
- Ovulation.
- Menstrual cramps.
- Ovarian cysts & other ovarian disorders.
- Fibroids.
- Uterine/Cervical cancer.
- Bladder disorders.
- Sexually transmitted diseases.
- Kidney infection or kidney stones.
- Intestinal disorders.
- Nerve conditions.
- Hernia.
- Pelvis disorder.
- Psychogenic pain.

Clinical presentation:
Worsening of menstrual cramps.
Menstrual pain,
Vaginal bleeding, spotting or discharge.
Painful or difficult urination.
Constipation or diarrhea.
Bloating or gas.
Bloody stool.
Pain during intercourse (dyspareunia).
Fever or chills.
Pain in the hip/groin area.

Diagnosis:
History.
Physical examination.
Blood & urine tests.
Pregnancy test.
Vaginal cultures (STD).
Abdominal & pelvic X-rays
Bone density screening.
Diagnostic laparoscopy.
Hysteroscopy.
Stool culture.
Colonoscopy.
USG.
CT/MRI (abdomen & pelvis).

53
Treatment:
According to the etiology.

Complications:
According to the etiology.

9.B Legislation on assisted reproduction practice in Europe


Definition:

Assisted reproductive technology (ART) is the technology used to achieve pregnancy in


procedures such as fertility medication, artificial insemination, in vitro fertilization (IVF) &
surrogacy.

Used primarily for infertility treatments.

Artificial insemination with donor semen (AID) is indicated in cases of male infertility or when
the husband is a carrier of serious inherited diseases or abnormality (Schenker, 1995b).

A single woman (or lesbian couple) is entitled to have children, even though these children may
have no legal father (European Convention, 1978).

In Poland & France the donor must be married.

In Israel the donor must be single.

French legislation states that the donor is only refunded for expenses (Lansac and Le Lenaou,
1994).

The donor has no rights, obligations or interest with regard to the child born as result of AID,
and the child has no rights of ligation or interest toward the donor in any current system.

Surrogate motherhood may be utilized in cases of uterine infertility or in cases of severe


maternal disease during pregnancy (Schenker, 1992).

In the UK, surrogacy may be practiced on a non-commercial basis, & only to benefit women for
whom a surrogacy agreement represents the only chance to have a child.

In order to avoid inheritance of age-related genetic disorders, donors should be young


(Benshushan and Schenker, 1993a).

Most international ethics committees state that gamete donors should not be reimbursed for
the donation.

Some countries have already regulations that limit the number of embryos transferred into the
uterus (e.g. the UK).

54
9.C Placenta Praevia
Definition:
An obstetric complication in which the placenta is inserted partially or wholly in the lower
uterine segment.
A leading cause of antepartum haemorrhage (vaginal bleeding).

Clinical presentation:

Painless, bright red vaginal bleeding.

Commonly occurs around 32w.

This bleeding often starts mildly and may increase as the area of placental separation
increases.

Suspected if there is bleeding after 24w.

May present as failure of engagement of fetal head.

Etiology Unknown.

Risk factors:
- Previous placenta previa, caesarean delivery, myomectomy or endometrium damage caused
by dilatation & curettage (D&C).
- Previous, large number of closely spaced pregnancies, are at higher risk due to uterine
damage
- Smoking during pregnancy.
- Cocaine use during pregnancy.
- Women < 20 & > 35 (increasing risk with age).
- Women with a large placentae from twins or erythroblastosis.
- Placental pathology
o Vellamentous insertion
o succinturiate lobes
o bipartite
o bilobed placenta
- Placenta previa is a risk factor of placenta accreta.

Diagnosis:
History may reveal antepartum hemorrhage.
Abdominal examination (uterus non-tender, soft & relaxed)
Fetus is in an oblique or breech position or lying transverse as a result of the abnormal
position of the placenta.
Malpresentation in ~ 35% cases.
No digital vaginal examination!
Confirmed by USG.
Transvaginal USG has superior accuracy over transabdominal.
If the woman is not bleeding severely she can be managed non-operatively until the 36th
week.

55
Treatment:
An initial assessment to determine the status of the mother and fetus is required.
Immediate delivery of the fetus may be indicated if the fetus is mature or if the fetus or
mother are in distress.
Blood volume replacement (to maintain BP) & blood plasma replacement (to maintain
fibrinogen levels) if necessary.
The corticosteroids are indicated at 24–34 weeks gestation if the patient has bleeding, given
the higher risk of premature birth.

Mode of delivery:
Determined by clinical state of the mother/fetus & USG.
RCOG recommends that the placenta should be at least 2 cm away from internal os for an
attempted vaginal delivery.
In cases of fetal distress and major degrees, C section is indicated.
C section is contraindicated in DIC.
Hysterectomy (rare).

Complications:

I. Maternal:
- Antepartum hemorrhage.
- Malpresentation,
- Abnormal placentation.
- Postpartum hemorrhage.
- Placenta praevia increases the risk of puerperal sepsis & postpartum hemorrhage because
the lower segment to which the placenta was attached contracts less well post-delivery.

II. Fetal:
- IUGR.
- Premature delivery.
- Death.

56
Triplet 10

10.A Regulation of the menstrual cycle

For Menstrual Cycle details see question 3/A

Follicular phase:

Continuous pulsatile release of GnRH from hypothalamus, reinitiated at puberty.


GnRH release + end of negative ovarian feedback (=drop in estrogen, progesterone, inhibin)
rise in FSH, LH (Gonadotropins)
Gonadotropins ovaries folliculogenesis
Follicular and luteal cells in follicles release steroid hormones (estradiol, progesterone)
Steroid hormones negative feedback on GnRH and gonadotropin secretion
LH surge for 48 hrs (due to sudden increase in estradiol – not entirely understood)
ovulation

Luteal phase:

Remaining follicular cells become corpus luteum: steroid hormone and Inhibin production,
progesterone dominant phase.
Inhibin B peaks (negative feedback on pituitary FSH), then falls until end of follicular phase
No pregnancy: corpus luteum corpus albicans: drop in steroid hormones and inhibin
negative feedback lifted
Pregnancy: production of hCG “rescues” corpus luteum continued production of steroids

57
10.B Evaluation of fetoplacental function
Placenta provides fetus with O2 and nutrients and receives CO2 and waste products – Placental
insufficiency occurs when the uteroplacental unit is compromised.

Consequences:

Fetal hypoxia
Shunting of blood flow to brain
Late decelerations of FHR (Fetal heart rate)
Anaerobic glycolysis
Metabolic acidosis
Causes:

- DM
- Chronic HTN
- Coagulopathies (Placental infarcts)
- Anemia
- Placental abruption
- Tobacco use, Drugs, Medications

Evaluation:

Doppler Ultrasound of Umbilical Artery – Used in high-risk pregnancies, determine need for
intervention (CS)

Assess resistance to blood flow in placenta


Based on characteristics of systolic and diastolic blood flow
Index systolic/diastolic ratio: When peripheral resistance (in placenta) increases, diastolic
flow (forward flow from fetus’ heart to placenta) decreases (and can become reversed)
increased s/d ratio.
Seen in severe cases of IUGR secondary to placental insufficiency impeding fetal demise

Placental volume and villous tree vascular flow patterns:

Improved USG methods can estimate placental volume and blood flow
Reduced size and abnormal flow patterns: adverse outcomes (preeclampsia, IUGR)

Maternal serum analyte (not routinely used, not accurate enough)

hCG
Pregnancy-associated plasma protein A (PAPP-A)
ADAM12
Angiogenic factors

Indirect methods:

Symphysis fundal height:

58
Sequential measurement: reveal changes in fetal growth
Suspicion for IUGR USG

USG assessment of fetal growth:


Head (Bi-parietal) circumference
Abdominal circumference
Amniotic fluid volume
Femur length

Middle cerebral artery Doppler: pulsatility index


Work up of IUGR, twin to twin transfusion sy,

Asymmetric IUGR due to shunting of blood to brain


Usually indicates severe compromise and requires intervention

10.C Kidney and urinary tract diseases in pregnancy


Urinary Tract Infections (UTI):

Dilatation of upper renal tract + urinary stasis predisposes to cystitis and pyelonephritis
Cystitis:
o Symptoms: frequency, urgency, dysuria, haematuria, proteinuria
o T.: oral ATB to prevent complications
Pyelonephritis:
o Symptoms: fever, rigors, vomiting, loin and abd pain
o T.: IV ATB, IV fluids, antipyretics
Preferred ATBs: Amoxicillin, Cephalosporins. Contraindicated: Ciprofloxacin, Tetracyclines

Acute renal failure:

Oliguria, Urea and Creatinine, hyperkalemia, metabolic acidosis

Pregnancy-related causes:

Pre-renal:
o Haemorrhage
Antepartum (abruption, placenta praevia)
PPH
o Hyperemsis
o Septic shock
Intrinsic
o Pre-eclampsia
o HELLP sy
o Sepsis
o Drug reaction

59
o Amniotic fluid embolus
Post-renal
o Obstruction

Management: Treat underlying disease, correct hyperkalemia, coagulopathies; dialysis.

Chronic kidney disease:

Commonest causes: Reflux nephropathy, DM, Lupus nephritis, PCKD, chronic GN

Risk factors associated w/ CKD:

Maternal
o Possibly permanent deterioration in renal function
o HTN, Proteinuria (PU)
o Pre-eclampsia
o UTIs
o Venous thromboembolism
Fetal
o Abortion
o IUGR
o Spontaneous and iatrogenic preterm delivery
o Fetal Heath

Management:

Investigations ideally before conception: Urea and electrolytes, CBC, 24 h protein, Crea
clearance
Early + regular antenatal care BP, renal function, PU, fetal growth + well-being, + Doppler
detect: anemia, UTI, pre-eclampsia, IUGR
Stop ACE-I
Implications on pregnancy:

Mild impairment: successful outcome in 90%


Moderate: decline in renal function, IUGR, preterm up to 50%
Severe impairment (crea > 250 μmol/L):higher risk for complications than chance of
successful pregnancy, advise against.

Pregnancy after renal transplant:

1 yr after tp, stable renal function, immunosuppression at maintenance levels


Frequent antenatal care (cave: infection, dehydration, pre-eclampsia, rejection!)
BP and anaemia!
Immunosuppressive Tx is continued
CS only for obstetric reasons

60
Triplet 11

11.A Dysmenorrhoea and dyspareunia

Dysmenorrhea

Definition: Difficult menstrual flow or painful menstruation (which interferes with daily
activities). Cyclic menstrual pain is common and usually is described as cramping and is often
accompanied by low back pain, nausea and vomiting, headache or diarrhea.

Always consider Intra Uterine Device (IUD) as the cause of uterine cramping.

Primary dysmenorrhea describes cyclic menstrual pain without an identifiable associated


pathology commonly starts shortly after menarche considered to be diagnosis of
exclusion
Secondary dysmenorrhea frequently complicates endometriosis, leiomyomas,
adenomyosis, endometrial polyps and outlet obstruction.

May be associated with other gynaecological complaints, such as dyspareunia (pain during sex)
treatment for this type focuses on eliminating the cause.

Risk factors include: long menstrual periods, smoking, increased BMI

Pathophysiology: Endometrial sloughing release of PG’s myometrial contractions and


ischemia. In severe cases there are high levels of PG’s in menstrual fluid, especially in 1st two days of
menstruation.

Diagnosis: Primary dysmenorrhea is diagnosis of exclusion.

Menstrual cramps with no other symptoms no additional evaluation empirical therapy


Culture for STD’s if risk for Pelvic Inflammatory Disease is present
Transvaginal USG to exclude structural pelvic pathology

Treatment:

NSAID’s because PG’s are an important factor in the pathogenesis of dysmenorrhea


Hormonal contraception believed to lower PG’s production
GnRH estrogen lowering effect leads to endometrial atrophy and diminished PG
production (not commonly used)
Hysterectomy done if pregnancy is not planned in the future
LUNA (laparoscopic uterine nerve ablation) or presacral neurectomy pain relief

61
Dyspareunia

Definition: Painful intercourse, which may be associated with vulvar, visceral, musculoskeletal,
neurogenic or psychosomatic disorders. Frequently associated with Chronic Pelvic Pain thus
diagnostic follow up overlap. Classified as:

insertional (pain in vaginal entry) – can be due to: vulvodynia, vulvitis, poor lubrication
deep (associated with deep thrusting) – can be due to: endometriosis, pelvic adhesions, bulky
leiomyomas

or

primary
secondary – pain after intercourse

Diagnosis:

- History ask for associated symptoms (vaginal discharge, vulvar pain, dysmenorrhea,
Chronic Pelvic Pain, poor lubrication); previous surgery, breast feeding, psychosocial
background
- Physical exam erythema, episiotomy scars, atrophy, skin irritants; Deep: cystitis,
congenital anomalies, reconstruction surgery
- pH testing
- culture

Treatment: Treat the cause.

- vaginismus – structure desensitization


- poor lubrication education about adequate arousal techniques and/or use of lubricants

surgery – for structural pathologies; may include: ablation of endometriosis, lysis of adhesion,
restoration of normal anatomy

11.B First stage delivery


First stage of labor begins with regular uterine contractions and ends with complete cervical
dilatation at 10 cm. It is divided into a latent and an active phase.

1. Latent phase
Begins with mild, irregular uterine contractions that soften and shorten the cervix
Contractions become progressively more rhythmic and stronger
It is the longest part and can range from 12 to 20 hours (in first pregnancy)
Contractions last 30-70 seconds and feel like backache or menstrual cramps
The rupture of amniotic membranes can occur spontaneously in stage one or later in the
process
No need yet to go to hospital

62
2. Active phase
Usually begins at about 3-4 cm of cervical dilation and is characterized by rapid cervical
dilation and descent of the presenting fetal part
Contractions are stronger and more painful, lasting approx. 45-60 seconds with 3
minutes interval
When cervix dilates to 8-10 cm (“transition stage”) contractions come aoorix every 2-3
min and last for 1 minute
Nausea and back pain sensation is possible

11.C Eclampsia
Definition: Onset of convulsions in a woman with preeclampsia not attributed to other causes.
Acute complication of pregnancy.
.
The seizures are generalized (tonic – clonic) and may appear before, during, or after labour. Together
with preeclampsia this condition is known as hypertensive disorder of pregnancy or toxemia of
pregnancy and it affects 3-5% of pregnancies. Etiology is unknown, but most significant risk factor is
preeclampsia.

Risk factors:

- Previous eclampsia
- First pregnancy
- Pre-existing vascular disease (hypertension, DM, nephropathy)
- Thrombophilic disorder factor V Leiden
- Multiple gestation
- Genetic predisposition
- Age: <20 or >40 y.o.

Pathophysiology:

Occurs only in the presence of a placenta and is resolved by its removal.

Placental hypoperfusion is a key feature which is accompanied by increased sensitivity of


maternal vasculature to pressor agents. Loss of plasma from vascular tree results in edema
which additionally compromises the situation.

Symptoms of toxemia: hypertension, renal, pulmonary, hepatic & cerebral dysfunction.

Signs and symptoms: The key symptom is convulsions and seizures. The rest is more or less similar
to preeclampsia, but vary depending on organ affected.

Hypertension
Proteinuria
Cerebral signs nausea, vomiting, headache
HELLP syndrome Hemolysis, Elevated Liver enzymes, Low Platelets

63
End organ failure abdominal pain, liver failure, pulmonary edema, oliguria, altered level of
consciousness
Fetal distress or fetal bradycardia

Tonic-clonic convulsions are usually self-limited (1-2 min). Secure airway, prevent injury & aspiration
of gastric contents.

Give Diazepam / Lorazepam (only if convulsions continue).

Nearly all seizures are accompanied by a prolonged fetal heart rate deceleration, resolve after
seizure.

If possible, 10-20 minute period of in utero resuscitation should be done prior to delivery.???????

C section only if vaginal delivery is not possible within reasonable time.

Prevention:

Detection & management of preeclampsia is critical to reduce risk of eclampsia (magnesium


sulphate).

Treatment:

Prevent further convulsions, control BP & immediate delivery if possible.

Supportive: monitoring, airway support, oxygen.


Antihypertensives: labetolol (beta blocker) & hydralazine.

Deliver even if immature, as the eclamptic condition is unsafe for both baby & mother.

Regional anesthesia (epidural / spinal) for C section is contraindicated if coagulopathy has


developed.

Triplet 12

12.A Chlamydial, Mycoplasmal and Ureaplasmal infections

Chlamydial infections

Chlamydial infection is second most prevalent of the STD. Chlamydiae are small gram-negative
obligate intracellular microorganisms that preferentially infect squamocolumnar epithelial cells.
Chlamydia trachomatis infection affects the cervix, urethra, salpinges, uterus, nasopharynx, and
epididymis. Leading cause of infertility in women.
C trachomatis infection causes other diseases as well, including conjunctivitis, pneumonia or
pneumonitis, afebrile pneumonia syndrome, Lymphogranuloma venerum, Fitz-Hugh-Curtis syndrome
and trachoma.

64
Etiology
Nonwhite race
Multiple sexual partners or a new sexual partner
Age 15-24 years
Poor socioeconomic conditions
Exchange of sex for drugs or money
Intercourse without a barrier contraceptive
History of a previous STD or current coinfection with another STD
Certain cytokine
Certain variants in Toll-like receptor 1 and 4

Diagnosis: Treatment:
Physical examination & symptoms: Primary: Aziyhtomycin/Doxycycline
Painful and frequent urinating Secondary: Eryhtromycine
Bleeding between periods or heavy Sexuall abstinence untill a woman and
periods partner are treated
Painful/bleeding intercourse
Pain the lower abodminal parts with Prognosis:
nausea Antibiotic treatment is 95% effective for first-
Swollen skin around genitals time therapy. The prognosis is excellent if
treatment is initiated early and the entire course
of antibiotics is completed.

Prevention: Patient's education.

Mycoplasmal (genitalium) infections

Clinical presentation: generally produces severe clinical symptom / a combination of symptoms (may
be asymptomatic).
Urethritis.
Mucopurulent discharge.
Burning while urinating.

Risk factor for:


Cervicitis & pelvic inflammatory diseases (PID).
Endometritis & salpingitis.

Unlike other Mycoplasma, the infection is not associated with bacterial vaginosis.

It is highly associated with the intensity of HIV infection.

Treatment: ATB's - Azithromycin (macrolide) / Doxycycline (broad spectrum tetracycline - during


pregnancy?).

Complications:

Preterm birth, spontaneous abortion, cervicitis, pelvic inflammatory disease (PID) & Infertility.

65
Ureaplasmal infections

Infection of newborn - during delivery / by direct invasion of the fetus in utero.

Diagnosis: Microscopically.

Treatment: Erythromycin (macrolide) / Doxycycline (broad spectrum tetracycline - during


pregnancy?).

Complications: Congenital pneumonia, bacteremia, meningitis, & death in infants with very
low birth weight. May cause placental inflammation & invade the amniotic sac, causing
persistent infection & adverse pregnancy outcomes, including premature birth

12.B Second stage delivery


Begins with complete cervical dilatation (10cm) & ends
with delivery of fetus.

Mechanism of labor (cardinal movements):

Changes in the position of the fetus head during its


passage:

➢ Engagement.
➢ Descent.
➢ Flexion.
➢ Internal rotation.
➢ Extension.
➢ Restitution & external rotation.
➢ Expulsion.

Fetal heart rate should be monitored continuously (CTG).

Prolonged duration of this stage does not mandate operative delivery if progress is being made.

Management of second-stage arrest (non progressing):

➢ Continuing observation / expectant management.


➢ Instrumental delivery (Forceps / vacuum).
➢ Positioning of mother.

66
Delivery maneuvers:

➢ The head is held in mid position until it is delivered, followed by suctioning of the oropharynx
& nostrils.

➢ Fetus neck wrapped by umbilical cord? Unwrap if possible.

➢ If the cord is wrapped too tightly, may be double clamped & cut.

➢ Anterior shoulder is delivered first with downward traction on head & chin.

➢ Upward pressure in the opposite direction for posterior shoulder.

➢ Fetus should now be easily delivered with gentle traction away from the mother.

➢ Cord is clamped & cut.

➢ The baby is vigorously stimulated + dried & transferred to pediatrician / placed on mother's
abdomen.

Pain control:
➢ Systemic:
Meperidine.
Fentanyl.
Nalbuphine.
Butorphanol.
Morphine.

Or, regional:
Epidural.
Spinal.
Combined (spinal-epidural).

12.C Endocrine disorders in pregnancy


Endocrine + metabolic changes as a consequence of physiological alterations By fetoplacental
boundary (connection between mother & fetus).

Changes in maternal hormones & their binding proteins complicate assessment of the normal level of
most hormones during gestation.

Endocrine disorders can have a serious outcomes for the developing fetus.

Major endocrine disorders:

Gestational Diabetes Mellitus (GDM), pre-existing type 1 DM, thyroid & adrenal disorders.

Maternal obesity on pregnancy outcomes & infant obesity predisposition.

67
1. Gestational Diabetes Mellitus

Gestational DM – most common endocrine complication in pregnancy. It is defined as any degree of


carbohydrate intolerance with onset or first recognition during pregnancy. It can develop in
overweight, hyperinsulinemic, insulin-resistant or –deficient women. GDM increases fetal and
maternal morbidity and mortality.

Uncontrolled can risk in spontaneous abortion and congenital anomalies. DKA is an immediate
threat to maternal and fetal life. GDM increases the risk of fetal macrosmia (>4000 g), which is
associated with secondary complications such as operative delivery, shoulder dystocia and birth
trauma. There are also neonatal complication such as hypoglycaemia, respiratory distress syndrome
(RDS), hypocalcemia and hyperbilirubinemia.

Risk factors:
- Family history of DM
- Previous delivery of a newborn >4000 g
- Obesity
- Previous GDM
- Glycosuria
- Hypertension
- Preeclampsia in previous pregnancies
- Repeated abortions
- Age >30 y/o

Maternal effects: Fetal effects:


1. Hypertension 1. Macrosmia – leads to LGA, but with excessive fat
2. Preeclampsia deposition on the shoulders and trunk lead to
3. DM 2 shoulder dystocia
4. Polyhydramnion 2. Hypoglycemia – predisposes the newborn to
5. Placental abruption hypocalcemia within minutes after birth – life
6. Post-partum uterine atony threatening complication
3. Congenital anomalies
i. Cardiac – ASD, VSD, TOF, Coarctation
ii. CNS – neural tube defects, anencephaly
iii. Renal – hydronephrosis, renal agenesis, ureteral
duplications
iv. GIT – duodenal atresia, anorectal atresia
4. Other: RDS, polycythenia, DM 2, risk for obesity

Diagnosis:

75 g oral glucose load.


Clinical findings that suggest maternal hyperglycemia: fetal weight 70% or greater for
gestational age or polyhydramnios.

Management:

Conservative physical activity, multiple small meals

68
Intensive glycemic control
Insulin usually rapid insulim + NPH additive if conservative methods are not sufficient
Metformin
C-section if the baby is > 4000 g

2. Thyroid disorders

Causes

Hyperthyroidism – Grave’s disease


Hypothyroidism – Hashimoto’s thyroiditis

Hyperthyroidism

Clinical feature

Mimic normal pregnancy: increased HR, sensitivity to hot temperatures and fatigue
Irregular heartbeat
Nervousness
Severe nausea or vomiting
Slight tremor
Trouble sleeping
Weight loss

Hypothyroidism

Clinical feature

Extreme fatigue
Weight gain
Constipation
Difficulty concentrating and memory problems
Sensitivity to cold temperatures
Muscle cramps

Diagnosis of hypo-/hyperthyroidism

Based on symptoms
Physical exam
Blood test: TSH, T3, T4

Treatment

Hyperthyroidism
o Antithyroid drugs – propulthiouracil (1st trimester) and methimazole (2nd & 3rd
trimester)
Hypothyroidism
o Synthetic hormone – Levothyroxine – similar to T4

69
3. Adrenal disorders

Congenital adrenal hyperplasia, Cushing syndrome, Pheochromocytoma & primary


hyperaldosteronism.

All can reduce fertility & severely impact maternal + fetal health.

If treated properly, usually successful gestation & delivery.

A. Congenital Adrenal Hyperplasia


Genetic disorder which gene mutation affects encoding of adrenal steroidogenic enzyme
(essential for cortisol synthesis)
Decreased fertility
Treated with Glucocorticoids (prednisone, hydrocortisone, dexamethasone)
Management include:
o Gestational: adrenal steroid replacement and adrenal androgen suppression
o Labor and delivery: stress-dose glucocorticoid Tx
C-section in patients who have had reconstructive genital surgery

B. Addison Disease
AKA: chronic adrenal insufficiency cortisol deficiency
Conception, fetal development and delivery usually is not problematic
Nausea and vomiting possible in 1st trimester
Diagnosed by endrocine tests
Treated with Glucocorticoids and Mineralocorticoids

C. Cushing syndrome
Rare in pregnant woman because hypercortisolism results in ovulatory disturbances and sub-
fertility
Weight gain, abdominal striae, HTN, glucose intolerance
Usually caused by adrenal adenoma
Lab: cortisol levels, corticotrophin levels
Imaging: USG, MRI
Treatment – treat the cause; surgery (adrenalectomy)
Due to poor wound healing C-section is recommended

D. Pheochromocytoma
Rare tumor, which can be dangerous for both mother and fetus
Main sign: Hypertension
Arrhythmia, headache, perspiration, palpitation and tachycardia, postural hypotension,
chest/abdominal pain
Diagnosis – early dx is critical for mortality based on symptoms and MRI
Treatment:
o Conservative – alpha-adrenergic blockade, alpha&beta blockers
o Surgery – removal of the tumor

70
Triplet 13

13.A PCOS syndrome


Polycystic ovary syndrome is a problem in which a woman's hormones are out of balance. It can
cause problems with periods and make it difficult to get pregnant. PCOS also may cause unwanted
changes in the appearnce. If it isn't treated, over time it can lead to serious health problems, such as
diabetes and heart disease.

Overview:
PCOS is the most common endocrine disorder in women responsible for 80% of all cases of
anovulatory subfertility.
Irregular or absent ovulations (cycle >42 days).
• Polycystic ovaries on pelvic ≥ 12 antral follicles on one ovary.
• Ovarian volume >10mL.
Etiology
Not fully known.
hypersecretion of LH ,Elevated LH:FSH ratio
Genetic (familial clustering).
Insulin resistance with compensatory hyperinsulinaemia (defect on insulin receptor).
Hyperandrogenism (elevated ovarian androgen secretion).
Obesity:
Diagnosis
Basal (day 2–5): hormones check up
Exclusion other causes of amenorrhoea.
Examination:
BMI.
Signs of endocrinopathy, hirsutism, acne, alopecia, acanthosis nigricans.

Treatment
Aims at Improving menstrual regularity
Weight loss (exercise+diet)
Metformin
Combined Oral Contraceptive Pills (COCP)
Laparoscopic ovarian diathermic
Prognosis:
Increased risk of cardiovascular and cerebrovascular disease
Increased risk of hyperplasia and carcinoma
Increased risk of diabetes melitus

13.B Third stage delivery


Delivery of the placenta.

3 classic signs indicate that the placenta has separated from the uterus:
1. Uterus contracts & rises.
2. Cord suddenly lengthens.
3. A gush of blood occurs.

Usually 5-10min - 30min after delivery of the fetus.

71
Excessive traction should not be applied to the cord to avoid inverting the uterus, which can cause
severe postpartum hemorrhage (obstetric emergency!).

The placenta can also be manually separated by passing a hand between the placenta & uterine wall.

After the placenta is delivered, inspect it for completeness & for presence of 1 umbilical vein + 2
umbilical arteries.

Oxytocin can be administered throughout the third stage to facilitate placental separation by
inducing uterine contractions & decreases bleeding.

Complications:

➢ Postpartum hemorrhage (PPH):

Most common complication of 3rd stage.

Active management of 3rd stage reduces the frequency of this complication.

➢ Retained placenta:

Retention of the placenta in utero > 30min.

Bleeding may be visible / may manifest only by the increasing size of the uterus.

In absence of evidence of placental detachment, consider placenta accreta / a variant.

Injections into the cord vein: normal saline, oxytocin + saline, prostaglandin + saline, misoprostol +
saline & dextran should help deliver.???????????

Manual removal if the above maneuvers failed / if significant bleeding occurs.

Perform manual removal with a level of analgesia that matches the clinical urgency of the situation.
USG may be useful in some cases.

➢ Uterine inversion:

Very rare. The risk is increased in abnormalities of placentation (accreta) & is more likely with fundal
cord insertions & any condition that predisposes patients to uterine atony & prolapse.

Cord traction should never be done without counter traction / in absence of uterine contraction.

Leave the placenta attached & focus management on maternal resuscitation & rapid return of the
uterus to the abdominal cavity.

72
Placenta Accreta

All or part of the placenta attaches abnormally to the myometrium:

I. Accreta – chorionic villi attach to the myometrium


II. Increta – chorionic villi invade into the myometrium
III. Percreta – chorionic villi invade through the
myometrium

Diagnosis is done mainly by USG, but MRI can be helpful if USG is


inconclusive. Treatment include planned caesarean section and abdominal hysterectomy.

13.C Preterm birth


Birth before the organs are mature enough to allow normal, postnatal survival.

Risk for short & long term complications, including disabilities, impediments in growth & mental
development.

The 3 primary causes of mortality (infants < 1000gr): respiratory failure, infection & congenital
malformations.

Infection of the amniotic fluid leading to pneumonia is the major cause of mortality.

In Infants < 500gr: immaturity is the cause of mortality.

Etiology:
Preterm labor & preterm premature rupture of the membranes (PPROM) / due to maternal
indications (pregnancy-induced hypertension).
➢ Choramnionitis → Amniocentesis that demonstrates bacteria.
➢ Low socioeconomic status.
➢ Maternal diabetes → complicated by diabetes & poor glycemic control.
➢ Multiple gestation pregnancy → pregnancies related to infertility treatment have
dramatically increased.
➢ Early / advanced maternal age.
➢ Tobacco use.
➢ Emergency Induction: preeclampia, distress, placental abruption, placenta previa, Rh disease.

Clinical presentation:

Neurologic criteria: muscle tone of the trunk, extremities & joint mobility.

In contrast to false labor, true labor is accompanied by cervical dilatation & effacement.

Vaginal bleeding in the 3rd trimester, heavy pressure in the pelvis, abdominal / back pain could be
indicators that a preterm birth is about to occur.

73
A watery discharge from the vagina may indicate premature rupture of the membranes (PROM) &
but may not be followed by labor, usually delivery is indicated as infection (chorioamnionitis) is a
serious threat to both fetus & mother.

Diagnosis:

➢ Fetal fibronectin has become the most important biomarker—secretion indicates chorion-
decidua disruption.???????
➢ PV → cervix dilation of > • Cramps, vaginal discharge (amnion), pelvic pressure.???
➢ CTG - Heart Rate & contractions.

Treatment:

➢ Reduction of spontaneous preterm birth may be achieved by reduction in maternal activity.


➢ Corticosteroids → bethametazone / dexamethazone for lung maturation & surfactant
production.
➢ Tocolysis (anti contractions): nifedipine, Atosiban (oxytocin antagonist), I.V magnesium
sulfate.
➢ Antibiotics for bacterial vaginosis.
➢ Monitor at the Neonatal intensive care unit.

Complications:

➢ The shorter the preterm, the greater the risks of mortality & morbidity.

➢ Hypothermia: insufficient fat storage & disregulation.

➢ Neurological: apnea of prematurity, hypoxic-ischemic encephalopathy, retinopathy of


prematurity, developmental disability, cerebral palsy & intraventricular hemorrhage.

➢ PDA (Pathological demand avoidance).?????

➢ Respiratory → RDS (hyaline membrane disease), chronic lung disease (bronchopulmonary


dysplasia).

➢ GIT & metabolic → feeding difficulties, hypoglycemia, rickets, hypocalcemia.

➢ Infections → sepsis, pneumonia & UTI.

74
Triplet 14

14.A Vulvar dystrophies and vulvitis


Vulvar dystrophies refers to skin changes of the vulva, also known as Dermatoses. There are four
types:

Squamous cell hyperplasia – pink red vulva with irregular white or gray patch of the skin of
the vulva (keratanized) that is slightly raised (thickened) and usually minimal malignancy
potential. Associated to chronic irritation regardless of the aetiology.
Lichen simplex chronicus – thickened, white and itchy area, usually secondary to scratching
or rubbing and Atopic Dermatitis. Erythema is seen in early stages.
Lichen sclerosis – area of thin skin, which can happen at any age, including puberty, but
usually seen in menopause. Low malignancy potential.
Lichen planus – chronic idiopathic skin disease, which cause symptoms on the skin or in the
mouth and sometimes in the vulvar or vaginal area. In severe cases the genital area can
cause painful itchy red areas and sores – treated with corticosteroids, antihistamines and UV
light. Presents with rash and itching and has no established cure. Usually its mild and
asymptomatic thus does not need a treatment.

Symptoms: Diagnosis:
Itching Hx + Physical exam
Burning Labs:
Pain during intercourse o Elevated IgE (in Lichen Simplex
White or gray patches of thickened skin Chronicus)
on the vulva, sometimes with scaring, o Patch test – to exclude allergic
cracking, bleeding or wrinkling contact dermatitis
Redness or sores Biopsy
Stinging and irritation

Treatment:
1. Squamous cell hyperplasia / lichen
simplex chronicus corticosteroids
such as Betamethasone (topical), if
antihistamines added there is an
antipruritic effect
2. Lichen planus corticosteroids (cream
or pills)
3. ATBs if infection is suspected

75
Vulvitis

Vulvitis refers to the inflammation of the soft folds of the vulvar skin (clitoris, labia and vaginal
vestibulum) which is caused by either infection, allergic reaction or injury and can affect at any age.

Symptoms: Diagnosis:
- Extreme constant itching - Hx + physical exam
- Burning sensation in the vulvar area - Caginal discharge testing for infections
- Vaginal discharge (if present)
- Small cracks of the vulvar skin Treatment:
- Erythema and swelling - Cease contact with irritating agent
- Blisters (vulva) - Cortisone ointment to reduce irritation
- Scaly, thick whitish patches and itching
- Bathing
- Topical estrogen cream

14.B Ectopic pregnancy


Definition: Also known as extrauterine pregnancy. Blastocyst implants anywhere other than the
endometrial lining of the uterine cavity. Accounts for 1-2% of all pregnancies. Used to be more life-
threatening in the past but still remains as an important cause of morbidity and mortality. Nearly
95% occur in Fallopian tube.

Ectopic pregnancy remains the leading cause of early pregnancy related death and mortality is
strictly related to severe hemorrhage from tubal rupture. Taking risk factors into account may lead to
faster diagnosis. Although there were singular cases of live deliveries (by laparotomy) ectopic
pregnancy results in death of the fetus usually within first trimester. Fortunately in most of the cases
women who had ectopic pregnancy can expect normal pregnancy in the future.

Classification: Risk factors:


Tubal pregnancy - Prior ectopic pregnancy
Nontubal ectopic pregnancy - Prior tubal surgery
Heterotopic pregnancy - Smoking
Persistent ectopic pregnancy - Prior STD with confirmed PID
Pregnancy of unknown location - 3 or more miscarriages
- Age >40
- Prior medical/surgical abortion

Clinical features: Differential diagnosis:


- May by asymptomatic - Abortion
- Amenorrhea - Endomyometritis
- Vaginal bleeding - Cervicitis
- Abdominal pain - Endometriosis
- Breast tenderness - Salpingitis
- Nausea - Tubo-ovarian abscess
- If rupture is present: - Adnexal torsion

76
o Abdominal distension - Appendicitis
o Tenderness - Diverticulitis
o Peritonism
o Hypovolemic shock
o Pain
- Pelvic mass (20%) posterolateral to the
uterus

Diagnosis: Management:
Beta human Chorionic Gonadotropin (b- Drugs – preferably in asymptomatic
hCG) – increased woman, contraindicated in
Serum progesterone level (less than 20 hemodynamic instability and sensitivity
nmol/l) - supportive to Methotrexate itself
USG (transvaginal) – we can see an o Methotrexate
adnexal mass that moves separately o Prostaglandins
from the ovary o Progesterone antagonist
Culdocentesis – extraction of fluid from Surgery
rectouterine pouch o Laparotomy
Laparoscopy/laparotomy – gold o Laparoscopy
standard for confirmation o Salpingectomy
Dilation and curettage o Salpingostomy

14.C Diabetes mellitus and pregnancy


Gestational DM: Most common complication of pregnancy.

Any degree of carbohydrate intolerance with onset / first recognition during pregnancy. It can
develop in overweight, hyperinsulinemic, insulin-resistant / deficient women.

GDM increases fetal, maternal morbidity & mortality.

Uncontrolled DM is a risk for spontaneous abortion & congenital anomalies.

DKA is an immediate threat to maternal & fetal life. GDM increases the risk of fetal macrosomia
(>4000gr) risk for operative delivery, shoulder dystocia, birth trauma & neonatal complication such
as hypoglycaemia, respiratory distress syndrome (RDS), hypocalcemia & hyperbilirubinemia.

The effects are different for women with preconception DM & those with GDM, as glucose levels are
toxic to the developing fetus & high level of glucose in first 8w may lead to serious congenital
anomalies & even stillbirth.

77
Risk factors:

- Family history of DM
- Previous delivery of a newborn >4000 g
- Obesity
- Previous GDM
- Glycosuria
- Hypertension
- Preeclampsia in previous pregnancies
- Repeated abortions
- Age >30 y/o

Maternal effects: Fetal effects:


7. Hypertension 1. Macrosmia – leads to LGA, but with excessive fat
8. Preeclampsia deposition on the shoulders and trunk lead to
9. DM 2 shoulder dystocia
10. Polyhydramnion 2. Hypoglycemia – predisposes the newborn to
11. Placental abruption hypocalcemia within minutes after birth – life
12. Post-partum uterine atony threatening complication
3. Congenital anomalies
i. Cardiac – ASD, VSD, TOF, Coarctation
ii. CNS – neural tube defects, anencephaly
iii. Renal – hydronephrosis, renal agenesis, ureteral
duplications
iv. GIT – duodenal atresia, anorectal atresia
4. Other: RDS, polycythenia, DM 2, risk for obesity

Diagnosis:

Screening is the best diagnostic method, which in Europe features 1 step approach using 75 g oral
glucose load. Clinical findings that suggest maternal hyperglycemia: fetal weight 70% or greater for
gestational age or polyhydramnios.

Normal glucose tolerance test values

Less than or equal to 100 milligrams per deciliter (mg/dL) or 5.6


Fasting:
millimoles per liter (mmol/L)
75 g of
glucose 1-hour: Less than 184 mg/dL or less than 10.2 mmol/L

2-hour: Less than 140 mg/dL or less than 7.7 mmol/L


Management:

Conservative physical activity, multiple small meals


Intensive glycemic control
Insulin usually rapid insulim + NPH additive if conservative methods are not sufficient
Metformin

C-section if the baby is > 4000 g

78
Triplet 15

15.A Endometriosis
A common benign, gynecologic disorder defined as the presence of endometrial tissue out of its
normal location (uterus).

Hormonally dependent disease (Estrogen) mainly found in reproductive-age woman, may be


asymptomatic, subfertile / suffer varying pelvic pain.

Staged from minimal (I) to severe (IV).

Etiology / Pathogenesis unknown!

Sites of endometriosis:

A. External endometriosis
a. Ovaries, uterine ligaments (broad & round ligament) & pouch of Douglas – most
common sites
b. Fallopian tubes, intestines, urinary bladder etc.
c. Brain, liver, lungs (probably due to vascular spread)
B. Internal endometriosis
o AKA: Adenomyosis
o Endometrial tissue is confined to myometrium
o Diagnosed by histology after hysterectomy

Signs & symptoms:

I. Signs – at bimanual examination fixed, retroverted uterus; enlarged, tender ovaries;


enlarged uterus (adenomyosis)
II. Symptoms:
a. Chronic pain (!!!)
b. Dysmenorrhea
c. Dyspareunia
d. Pain or bleeding at defecation
e. Infertility
f. Endometriosis in lungs may cause cyclic hemoptysis
g. Endometriosis in brain may cause cyclic seizures

Diagnosis:

- USG – hypoechoic areas, especially in ovaries


- Laparoscopy – gold standard – foci are seen as blue-black spots
- Biopsy
- MRI – for deep tissues

Treatment:

- Medical therapy
o Progesterone – inhibits the growth of endometrium

79
o Oral contraceptives
o Antigonadotropin – suppresses growth of endometrial tissue
o GnRH agonists – desensitize the GnRH receprors lower Lh/FSH lower estrogen
- Surgical therapy
o Laparoscopic or open
o Conservative surgery – excision of endometriomas (to maintain fertility), thermal
coagulation
o Semi-conservative – hysterectomy but ovarian function is retained
o Radical surgery – removal of uterus & ovaries

15.B Endocrine disorders and female reproduction


The endocrine system has a number of disorders affecting the female reproductive system.

Obesity.

Thyroid disorders (resulting from missing glands, cancer, genetic disorders).

Adrenal hyperplasia.

Tumors in the pituitary gland.

Clinical presentation:

Sub-fertility / infertility.

Growth delay.

Short statue.

Delayed sexual development.

Irregular periods.

Amenorrhea – absent menstrual periods more than 3 months.

Primary Amenorrhea – occurs before first menstrual cycles, experience of abnormal changes
through puberty and features poorly developed reproduction organs
Secondary Amenorrhea – occurs when a woman experiencing normal menstrual cycles stios
menstruating for a period of six months or more. This include also abnormal/irregular
periods. Common causes include: obesity, excessive exercise, body fat percentage under 15%
and sudden weight loss.

Polycytosis – see Q13.A – AKA: PCOS; one of the most common reproduction and endocrine disorder
affecting fertility. It is a leading cause of infertility and occurs due to an imbalance in the female sex
hormones. Features infertility, changes in menstrual cycle, skin changes, ovarian cysts.

Hirsuitism – defined by excessive hairness in children and women in areas where terminal hair does
not normally grow. Often caused by pituitary/adrenal gland disorders.

80
Turner syndrome – a 45,X (gonadal dysgenesis) genetic endocrine disorder affecting female
reproduction system. Features normal body growth but puberty is inhibited. Other symptoms include
heart or renal problems which vary according to the case. This disorder causes sub-fertility but with a
proper treatment it can be corrected and pregnancy is possible.

Premature ovarian failure – AKA: ovarian hypofunction. It relates to the reduced function of the
ovaries, including decreased production of hormones. Usually caused by genetic endocrine disorders
and factors such as chromosome abnormalities, or it may occur in some autoimmune disorders. Also
women undergoing chemotherapy and radiation therapy may experience premature ovarian failure.

15.C HELLP syndrome


HELLP syndrome is a life-threatening obstetric complication which is thought to be a variant of pre-
eclampsia, which occurs in late stages of pregnancy and even after childbirth. HELLP is an
abbreviation for: Hemolysis, Elevated Liver enzymes, Low Platelet count. Risk factors include: age >
34 y.o, multiparity and white race/European.

Pathophysiology: unknown!

Characterized by thrombocytopenia, hemolytic anemia & liver dysfunction.


Main process is in the liver: cells suffer ischemia & necrosis.
Leads to a form of DIC & paradoxical bleeding (emergency!).

Clinical features:

- Occur typically between 27’ week gestation and delivery (mean age 25)
- Systemic symtoms: malaise, nausea, vomiting, weight gain and other non-specific Sx
- Epigastric and right upper quadrant pain tenderness
- Headache, visual changes and jaundice
- Edema with secondary weight gain
- Dyspnea (if pulmonary edema present) crackles
- Fatigue or weakness
- Signs in Head, ears, eyses, nose and throat:
o Signs of dehydration sunken eyes
o Edema puffy eyes
o Dry mucous membranes

Complications:

Maternal
o Hematologic – DIC, bleeding, hematoma
o Cardiac – cardiac arrest, myocardial ischemia
o Pulmonary – pulmonary edema, respiratory failure, pulmonary embolism, ARDS
o CNS – haemorrhage/stroke, cerebral edema, central venous thrombosis, seizures
o Hepatic – hepatic hematoma, ascites, nephrogenic diabetes insipidus
o Infection

81
Neonatal
o Prematurity
o Intrauterine growth retardation
o Thrombocytopenia

Diagnosis:

- Blood test: CBC, coagulation panel, liver enzymes (AST/ALT, LDH), electrolytes, BUN,
bilirubin, D-dimer
- Renal function studies
- Fibrin degradation product (FDP) – elevated
- Histology – (liver) parenchymal necrosis with hyaline deposits of fibrin-like material
- CT/MRI – large vessel vasculopathy
- USG – may reveal increased echogenicity in irregular, well demarcated liver areas

Staging:

Two distinct classification systems has been developed: Tennesee classification and Missisipi
classification.

Missisipi classification – Class 1-3 – based on platelet count, AST/ALT levels and LDH levels.
Tennesee classification – complete or partial; complete (platelet count, AST/ALT and LDH
levels), partial severe preeclampsia + 1-2 features of HELLP

Treatment:

- Immediate delivery of the baby


- Magnesium sulphate – anticonvulsant – decreases risk for seizures
- Fresh Frozen Plasma – to treat DIC
- Blood transfusion – to treat anaemia
- Corticosteroids – Dexamethasone - prevent further hepatocyte death and platelet activation
- Antihypertensives – Labetalol or Hydralazine
- Postpartum curettage – lowers mean arterial pressure and improves oliguria and
thrombocytopenia

Bed rest – to minimize risk of trauma (if hepatic hematoma is present)

82
Triplet 16

16.A Evalutation and Management of the Infertile Couple


Definition: inability to conceive after 1 year of unprotected intercourse of reasonable frequency.

primary infertility: no prior pregnancies

secondary infertility: infertility following at least one prior conception

Epidemiology: 10 to 15 % of reproductive-aged couples; most couples are more correctly considered


to be subfertile.

In general, infertility can be attributed to the female partner one third of the time, the male partner
one third of the time, and both partners in the remaining one third.

Today, 85% of infertile couples who undergo appropriate treatment can expect to have a child.

Approximately 15% of infertile couples are considered to have unexplained infertility

Female gynaecological and medical history:

Focus on:

menstruation (frequency, duration, recent change in interval or duration, hot flashes, and
dysmenorrhea)
prior contraceptive use
duration of infertility
History of: recurrent ovarian cysts, endometriosis, leiomyomas, sexually transmitted
diseases, or PID
Pregnancy complications such as miscarriage, preterm delivery, retained placenta,
postpartum dilatation and curettage, chorioamnionitis, or fetal anomalies should be noted
Interventions such as cervical conisation
Hyperprolactinemia, thyroid disease, PCOS, CAH
Prior Chemotherapy (Ovarian Failure)

Male History

Questions to be asked: pubertal development and difficulties with sexual function, erectile
dysfunction, particularly in conjunction with decreased beard growth ( may suggest decreased
testosterone levels)

History of:

STIs: may result in obstruction of the vas deferens


Mumps
Cryptorchidism, testicular torsion, or testicular trauma ( may suggest abnormal
spermatogenesis)

83
Illness with high fevers
Prior treatment with chemotherapy or local radiation therapy
Use of anabolic steroids also decreases sperm production

Evaluation for specific causes:

1. ovulation
2. normal female reproductive tract anatomy
3. normal semen characteristics

Etiology of Infertility in the Female

I. Ovulatory Dysfunction (Anovulation/Oligo-ovulation)


Hypothalamus (acquired disorders include those due to lifestyle, for example, excessive
exercise, eating disorders, or stress)
Anterior pituitary
Ovaries (PCOS)
Thyroid disease and hyperprolactinemia may also contribute to menstrual disturbances

Menstrual Pattern. regular ovulation?

Basal Body Temperature. ovulation?

Ovulation Predictor Kits. measure the concentration of urinary luteinizing hormone

Serum Progesterone. on cycle day number 21 following the first day of menstrual bleeding, or 7
days following ovulation.

Endometrial Biopsy.

Sonography. Observation of development of a mature antral follicle. Dg of PCOS

II. Female Aging and Ovulatory Dysfunction

Loss of viable oocytes

Starting at age 35, fertility testing should be strongly considered after failure to conceive for 1 year,
or perhaps even after six months, in all patients desiring conception!

Follicle-Stimulating Hormone. Measurement of serum FSH levels in the early follicular phase is a
simple and sensitive predictor of ovarian reserve

Other tests for ovarian reserve: Estradiol, Antral Follicle Count, Antimüllerian Hormone,
Clomiphene Citrate Challenge Test.

84
III. Tubal and Pelvic Factors

Pelvic adhesions, Tubal obstruction

Essential for ovum, sperm, embryo transport and fertilization!

Signs and symptoms suggesting tubal/pelvic factors: Dysmenorrhea, Chronic Pelvic pain
Hx: PID, endometriosis, prior pelvic surgery, appendicitis,
o Sequelae of these include fallopian tube scarring & obstruction
Salpingitis isthmica nodosa

Hysterosalpingogram. detects approximately 70% of anatomic abnormalities

Hysteroscopy. evaluates the endometrium and the architecture of the uterine cavity

Laparoscopy. assesses pelvic structures, including the uterus, ovaries, and fallopian tubes as well
as the pelvic peritoneum. Laparoscopy also allows treatment of pelvic abnormalities, such as
adhesions and endometriosis.

IV. Uterine Abnormalities


Congenital Anomalies. uterine septum, bicornuate uterus, unicornuate uterus, and uterine
didelphys.
Acquired Abnormalities. Endometrial Polyps. Leiomyomas. Asherman Syndrome
(intrauterine adhesions, also called synechiae, history of uterine dilation and curettage)

Investigations: (1) hysterosalpingography, (2) transvaginal sonography with or without saline


instillation, (3) 3-D transvaginal sonography, (4) hysteroscopy, (5) laparoscopy, and (6) pelvic imaging
by magnetic resonance (MR) imaging.

V. Cervical Factors

Affected mucus production. Observed in


women who underwent: cryosurgery, cervical
conization, or a loop electrosurgical excision
procedure (LEEP) for treatment of an abnormal
Pap smear.

PostcoitalTest. sample of the cervical


mucus is obtained from the cervical os within
few hours of intercourse.

Etiology of Infertility in the Male

1. abnormalities of sperm production


2. abnormalities of sperm function
3. obstruction of the ductal outflow tract

Semen Analysis.

85
The semen specimen is usually obtained by masturbation after 2 to 3 days of abstinence; analysis
should be performed within 1 hour of ejaculation. Sperm production and development takes
approximately 70 days. Therefore, abnormal results of the semen analysis reflect events that
occurred more than 2 months before the specimen collection

86
Hormonal Evaluation of the Male

Defect in hypothalamic-pituitary function


Defects within the testes

Serum FSH and testosterone levels

Low FSH and low testosterone levels: suggest hypothalamic dysfunction, such as idiopathic
hypogonadotropic hypogonadism (Kallmann syndrome)
Elevated FSH and low testosterone levels: testicular failure, and most men with oligospermia
fall into this category
Elevated serum prolactin levels and thyroid dysfunction impact spermatogenesis and are the
most likely endocrinopathies to be detected

Genetic Testing of the Male

Genetic testing is indicated in men with azoospermia (no sperm) and severe oligospermia.

Most common mutations found:

cystic fibrosis transmembrane conductase regulator (CFTR)


somatic and sex chromosome abnormalities (Klinefelter)
microdeletions of the Y chromosome

Testicular Biopsy

Testicular failure, cryopreservation

Management:

Ovarian Stimulation

Ovulation induction is indicated in women with anovulation or oligo-ovulation. Underlying


conditions (hyperprolactinemia, PCOS, and high levels of stress (including psychologic stress,
intense exercise, and eating disorders) should be treated first.

- Clomiphene citrate: selective estrogen receptor modulator, by negative feedback: FSH &
LH release by
- COH (Controlled Ovarian Hyperstimulation ): exogenous gonadotropins.

Intrauterine Insemination

Suitable for infertility due to: cervical factors, mild and moderate male factors, and unexplained
infertility.

Motile and morphologically normal sperm are washed and place into the uterine cavity.

IUI with anonymous sperm donation is an alternative.

87
Assisted reproductive technologies

Involve ovarian stimulation to produce multiple follicles, retrieval of the oocytes from the
ovaries, oocyte fertilization in vitro in the laboratory, embryo incubation in the laboratory, and
transfer of embryos into a woman’s uterus through the cervix.

16.B USG in pregnancy

Weeks 10 – 20 – 30 – 36!

USG in First trimester

- Best timing for first-trimester examination: 18-20 wks GA


- Transvaginal or Transabdominal
- Abdominal ultrasound: visible at 5 to 6 wks GA
- Transvaginal ultrasound: visible 3 to 4 wks GA

Rule out an ectopic pregnancy (confirm IU implantation)

Estimate GA

Evaluate multiple gestations

Confirm cardiac activity

Pelvic masses, uterine abnormalities?

Nuchal translucency test (NT): measuring lucent area behind head. Measuring NT detecting
Down sy, trisomy 18, trisomy 13, Turner sy, Cardiac defects.

Standard USG in second/third trimester


- Evaluation of fetal presentation
- Amniotic fluid volume
- Cardiac activity
- Placental position
- Fetal biometry
- Anatomic survey

Limited USG (emergency, eg in a bleeding patient): Heart rate, placental abruption


Specialized USG
- Fetal Doppler
- Biophysical Profile (heart rate, muscle tone, movement, breathing, amniotic fluid)
- Fetal Echo
- Additional biometric studies

88
Evaluation of placental and cervical abnormalities
o Placental abruption
o Color-flow Doppler ultrasound assessment is used to identify placenta accrete
o Transvaginal USG
cervix: short cervix = risk for preterm delivery (at 24-30 GA)
rule out placenta previa
Fetal Growth assessment
o Biparietal parameter and head circumference
o Abdominal circumference: single most parameter in assessing fetal size and growth
o Femur length

16.C Forceps and vacuum delivery


AKA: operative vaginal delivery. 10-15%. Associated with less morbidity to the mother in the 2nd
stage of labour than CS. It depends on the operator’s skill and experience which one to use.

Indication for operative vaginal delivery

Maternal
o Exhaustion
o Prolonged 2nd stage
> 1h of active pushing in multiparous women
> 2h in primiparous women
o Medical indication for avoiding Valsalva manouevre (cardiac disease, hypertensive
crisis, uncorrected cerebral vascular formation)
Fetal
o Fetal compromise
o Control the after-coming head of breech (forceps)
Classification

Outlet operative vaginal delivery (mostly performed today)


Low operative vaginal delivery
Midpelvis operative vaginal delivery (sudden onset of severe maternal or fetal compromise,
simultaneous preparation of CS in case vaginal delivery fails)
Forceps

Curved blades positioned around fetal head


Used to apply traction
Control (slow down) rate of head in breech delivery
Complications
o Significant maternal genital tract trauma
o Injuries to brain and spine
o Musculoskeletal injury
o Corneal abrasion
o Shoulder dystocia

89
Greater risk for mother, smaller risk for fetus than vacuum extraction

Vaccum extraction

Vacuum cup is placed (between contractions) at fetal head and suction is applied (negative
pressure)
Should not be used before 34 wks GA
Complications
o Intracranial hemorrhage
o Subgaleal hematomas
o Scalp lacerations
o Retinal hemorrhage
Safer for mother
Criteria necessary for operative delivery

90
Triplet 17

17.A Influence of male factors on treatment of fertility/sterility


Fertility is dependent on both men & women.

It is crucial to look into male factor in treatment of fertility. Investigation should start in primary care
after 1st year / earlier if history of genital surgery, cancer treatment / previous sub-fertility.

Increasing ‘testicular dysgenesis syndrome’ with an increase in cryptorchidism, testicular cancer &
hypospadias (urethra on the underside of penis).

Normal male fertility is dependent on normal spermatogenesis, erectile function & ejaculation.

Etiology:
Semen abormality (eg, testis cancer drugs, OATS, varicocele)
Azoospermia
Oligozoospermia
Genetic issues
Disease like Cystic fibrosis
Immnulogical
Coital dysfunction

Diagnosis:
Semen check:
• Volume >1.5mL.
• Concentration >15 × 106 / mL.
• Progressive motility >32%.
• Total motility >40%.
FSH: elevated in testicular failure.
Karyotype: exclude 47XXY.
Cystic fibrosis screen: congenital bilateral absence of the vas deferens

Treatment:
- Treat any underlying medical conditions.
- Change lifestyle issues (alcohol, stop smoking).
- Review medications:
• antispermatogenic (alcohol, anabolic steroids, sulfasalazine)
• antiandrogenic (cimetidine, spironolactone)
• errectile/ejaculatory dysfunction (α or β blockers, antidepressants,
diuretics, metoclopramide).
- Medical treatments:
• gonadotrophins in hypogonadotrophic hypogonadism
• sympathomimetics (e.g. imipramine) in retrograde ejaculation.
- Surgical:
• relieve obstruction
• vasectomy reversal.
1 Surgical treatment of varicocele does not improve pregnancy rate and
is therefore not indicated.
- Sperm retrieval:
• from postorgasmic urine in retrograde ejaculation
• surgical sperm retrieval from testis with 50% chance of obtaining

91
sperm (greater if FSH is normal).
• Assisted reproduction: intrauterine insemmination, in vitro

See also Question 16.A

17.B Spontaneous abortion


Most abortion occur in first trimester & ~50% are due to chromosomal abnormalities. After the first
trimester, abortion rate & incidence of chromosomal abnormalities decrease.

Hemorrhage into the decidua basalis & adjacent tissue necrosis, usually accompanies early
miscarriage. In these cases, the ovum detaches, this stimulates uterine contractions that result in
expulsion.

Risk factors:

Fetal factors :
Developmental abnormality of the zygote, embryo,fetus, placenta.
Chromosomal errors
Maternal factors:
Infections
Endocrine disorders → hypothyroidism, DM (major anomalies)
Drugs
Smoking, alcohol
IUD (increase the risk for septic abortion)
Toxins (arsenic, lead, formaldehyde, benzene)
Immunological factors ( autoimmune diseases)
Thrombophilia
Trauma
Uterine defects ( leiomyomas, Asherman sy., fusion defects)
Incompetent cervix (painless cervical dilation)
Other - CVS, amniocentesis, progesterone deficiency (or more precisely decreased
activity of progesterone receptors), multiple pregnancy,

Clinical classification:

➢ Threatened abortion: presumed when a bloody vaginal discharge / bleeding appears through
a closed cervical os during the first half of pregnancy. With miscarriage, bleeding usually
begins first & cramping abdominal pain follows, hours to days later.

➢ No effective therapy!

➢ Inevitable abortion: Gross rupture of membranes - leaking amnionic fluid + cervical


dilatation, signals almost certain abortion.

➢ Incomplete abortion: Bleeding after placenta (whole / part) detaches from the uterus. The
internal cervical os opens & allows passage of blood. The fetus & placenta may remain
entirely in utero / may partially extrude through the dilated os.

92
➢ Missed abortion: previously confirmed pregnancy but then non-viable fetus in the uterus (in
USG), without abortion like features.

➢ Septic abortion: Endomyometritis is the most common manifestation of postabortal


infection, but parametritis, peritonitis, septicemia & even endocarditis occasionally develop.

➢ Treatment: prompt I.V broad-spectrum antibiotics followed by uterine evacuation.

➢ Recurrent miscarriage: > 3, consecutive pregnancy losses < 20w / weight < 500gr.

Diagnosis:
Physical examination → pelvic examination, abdominal examination, etc.
Lab → CBC with differential, beta-hCG, blood type, coagulation profile, urinalysis.
USG → usually transvaginal, always rule out ectopic pregnancy, retained products of
conception, hematoma, etc.
Culdocentesis → if fluid (or blood) is present in the cul-de-sac.
Diagnostic D&C → if normal pregnancy has been excluded.
Diagnostic criteria for nonviability:
The absence of an embryo with heartbeat ≥ 11 days following a scan that showed

Treatment:
Complete abortion - no futher treatment needed
Missed/Incomplete/Inevitable abortion before 13week's gestation-misoprstol, surgical
suction, dilation and curretage.
Ectopic pregnancy- medically (methotrexatte) or surgically (laparoscopy, laparotomy)
Septic abortion- Broad spectrum antibiotics
Unlcear diagnosis -suction dilation and curretage with diagnostic laparoscopy

17.C Anaemia in pregnancy


RBC's increases but plasma volume increases more, resulting in a relative anemia (hemodilution) -
physiologically lowered hemoglobin (Hb), hematocrit (Hct) & RBC count, but no effect on the mean
corpuscular volume (MCV).

Anemia: hemoglobin < 11 g/dL (1st & 3rd trimester), < 10.5 g/dL (second trimester).

Etiology/Risk factors:
multiple pregnancy
insufficeint minerals and nutrients in diet
smoking
alkohol
chronic diseases of gastrointestinal tract
taking epilepsy drugs
congenital blood diseases of mother
strong vomiting (also due to pregnancy)

Cl. Features & diagnosis:


Symptoms:
chronic tireness
tachcardia

93
hypotension
dyspnea
headache, dizzyness, nausea
weak nails and falling off hairs
swallowing problems
paleness
disturbance in sight and feeling
stomachache and jaundice
limbs numbing
Laboratory check up of the blood.

Treatment:
Diet rich in iron
Supplementation with vitamin B12 and iron and vitamin C
In some cases erythropoeitin

Triplet 18

18.A Anovulation Treatment and Management


Failure of ovary to release an egg (ova) > 3 months + infertility.

Normal functioning ovary releases one ovum every ~28 days.

Controlled by hypothalamic-pituitary-ovarian axis. Any condition that alters this axis leads to
anovulation.

Cardinal signs are oligomenorrhea / amenorrhea.

The most common cause is Polycystic Ovarian Syndrome (POCS).

Other causes: obesity, hyperprolactinemia, premature ovarian failure, thyroid dysfunction (hypo /
hyperthyroidism), anorexia nervosa, stress, tumors & drugs (phenothiazines, TCA, antipsychotics,
morphine, alpha-methyldopa, verapamil, cimetidine).

Treatment:

First estimate patient’s goals about future pregnancy as therapeutical approach differ.

Therapy focuses on ovulation induction & treating the cause.

It is crucial to rule out causes that pose serious danger to the patient.

⇨ Clomiphene citrate – selective estrogen receptor modulator – first-line for ovulation


induction, increases ovulation by increasing GnRH release.

⇨ Human chorionic gonadotropin (hCG) – gonadotropin similar to LH secreted by pituitary just


before ovulation. In combination with Clomiphene citrate has increased effectiveness.

⇨ Other medications:

94
o Levothyroxine – thyroid hormone replacement therapy (hypothyroidism).

o Oral contraceptives – hormone replacement.

o Antiandrogens – inhibit androgen feedback on pituitary gland.

o Glucocorticoids – corrects adrenal insufficiency.

o Progestins – used for endometrial stabilization & organization of basal layer in


chronic anovulation.

o Estrogens – used to build endometrial lining in acute / chronic anovulation.

o Aromatase inhibitors – reduces serum estrogen level.

⇨ Surgery – indicated to resolve the underlying cause, especially when medical therapy failed.

⇨ Indication: pituitary adenoma, benign neoplasm of ovarian / adrenal origin (exploratory


laparotomy, resection & staging are indicated).

⇨ Diet – usually applied in cases of endocrinological & metabolic disorders, observed in PCOS
(low-carbohydrate / low cholesterol).

⇨ Activity – weight-bearing exercise is recommended for hypoestrogenic states, such as


premature ovarian failure, when estrogen replacement is contraindicated.

⇨ Consultations:

o Neurosurgery – macro-adenoma.

o Psychiatry – anorexia nervosa, bulimia, dysmorphic disorder.

o Endocrinology – Cushing syndrome, Addison disease, DM 2, panhypotituitarism,


refractory thyroid disease.

o Gynecologic oncology / surgery – adnexal / adrenal mass of benign / malignant


origin.

18.B Thrombophilias in Pregnancy


Pregnancy is hyper-coagulable state – 4-5 times higher risk of venous thromboembolism.

Severe pregnancy complications (preeclampsia, IUGR, placental abruption, Recurrent miscarriage &
stillbirth) are associated with thrombophilias.

Pregnancy is a state characterized by the Virchow triad:

- Hypercoagulability
- Venous stasis
- Endothelial injury

95
Higher pro-coagulation factors (I, II, VII, IX, X) & Resistance to anticoagulants.

Protein C is increased, while Protein S (C-protein cofactor) decreases.

Also included are inherited thrombophilias (genetic mutations).

Etiology:

- Prior personal history of VTE


- Inherited thrombophilias (low-risk)
o Heterozygous factor V Leiden
o Heterozygous prothrombin G20210A mutation
o Protein S deficiency – vit. K dependent anticoagulant protein activated by C-protein
o Protein C deficiency – vit. K dependent anticoagulant protein degrading factor V and
VII
- Inherited thrombophilias (high-risk)
o Homozygous factor V Leiden
o Homozygous prothrombin G20210A mutation
o Compound heterozygous factor V Leiden with prothrombin mutation
o Antithrombin deficiency – inactivates thrombin and factors IX, X, XI and XII

- Acquired thrombophilias
o Antiphospholipid antibody syndrome – disorder which is diagnosed with either of the
following:
One or more unexplained fetal losses of a morphologically normal fetus after
10 weeks
At least one preterm birth prior to 34 weeks indicated for
preeclampsia/eclampsia
At least 3 consecutive unexplained miscarriages prior to 10 weeks
A history of vascular thrombosis
Lab:
Anticardiolipin IgG or IgM > 95th percentile
Antibeta2-glycoprotein I IgG > 99th percentile
Lupus anticoagulant
o Activated Protein C Resistance
o APCR and pregnancy loss
o Hyperhomocystanaemia

Symptoms: Thrombophilia poses a high risk for developing DVT and pulmonary embolism.

Warning signs of DVT:

Pain, swelling and tenderness in one of the legs


A heavy ache in the affected area
Warm skin in the area of the clot
Redness of the skin

96
Pulmonary embolism:

Chest or back pain


Dyspnea
Cough
Dizziness
fainting

Outcomes related to Thrombophilias in Pregnancy:

Pregnancy loss
Placental abruption – although consistent association has not yet been established, it has
been found that thrombophilia increases risk for abruption
Preeclampsia

Diagnosis:

Blood tests: CBC, coagulation panel,

Management:

Anticoagulant therapy
o Unfractioned heparin (UFH) – doesn’t cross the placenta
o Low-molecular-weight heparin (LMWH) - doesn’t cross the placenta
o Vitamin K antagonists – can cross the placenta
o Complications:
Maternal – thrombocytopenia, osteoporosis, bruising, urticarial rashes
Aspirin – crosses the placenta
o Might increase risk of gastrschitis

18.C Breech presentation and delivery


Breech presentation occurs, when the fetus lies with the buttocks or feet closest to the cervix
contrary to cephalic presentation. It is the most common malpresentation in obstetrics and accounts
for 5% of all deliveries.

Predisposing factors:

Polyhydramnios
Multiple pregnancy The fetus has additional space in the uterus
Premature labor
Placenta Praevia
Uterine malformations The fetus cannot assume vertex position
Fetal abnormalities

Types of breech positions:

Frank breech – 50-70%, hips are flexed but the knees are extended; feet lie close to the head
Complete breech – 5-10%, both knees and hips are flexed ‘Cannon ball’ position
Incomplete breech – 10-30%, one or both hips are extended; presents with foot first

97
Diagnosis:

External examination Leopold maneuvers


USG
Vaginal examination

Management/delivery:

External cephalic version


o Transabdominal manual rotation of the fetus into a cephalic (vertex) presentation
o Performed after 37th week of gestation
o Always be prepared for C-section
o Contracindications
Multiple pregnancy
Placenta Praevia
Vaginal breech delivery
o Can be performed after 37th week if complete footling is excluded (both feet down),
fetus < 3500 g
o Type 1 - Spontaneous breech delivery – no traction or manipulation is used, usually
preterm
o Type 2 – Assisted breech delivery – most common type of breech delivery; infant is
allowed to spontaneously deliver up to the umbilicus and then maneuvers are
initiated to assist for the remainder body parts
o Type 3 – Total breech extraction – the fetal feet are grasped and the entire fetus is
extracted. Should be used onlu for non-cephalic second twin
Complications
o Umbilical cord prolapsed and compression leading to neonatal hypoxia especially
with footling
o Cerebral hemorrhage
o Placental abruption
o Hip dislocation, fractures and brachial plexus injuries due to traction (Erb’s palsy)
o Asphyxia due to prolonged labor (foot or legs do not dilate the cervix properly)
Caesarian Section
o When vaginal delivery is contraindicated

Vaginal breech delivery requires an experienced obstetrician and careful counselling of the parents.
Parents must be informed about potential risks and benefits to the mother and neonate for both
vaginal breech delivery and C-section.

The risks of a C-section on subsequent pregnancies include: uterine rupture, placental attachment
abnormalities (placenta praeviam abruption, accreta).

98
Triplet 19

19.A Assisted reproductive techniques


The term assisted reproductive technologies describes clinical and laboratory techniques used to
achieve pregnancy in infertile couples for whom direct corrections of underlying causes are
not feasible. In principle, IUI meets this definition. By convention, however, ART procedures are
those that at some point require extraction and isolation of an oocyte.

1. In Vitro Fertilization
During IVF, mature oocytes from stimulated ovaries are retrieved transvaginally with sonographic
guidance.
Sperm and ova are then combined in vitro to prompt fertilization. If successful, viable embryos are
transferred transcervically into the endometrial cavity using sonographic
guidance.

Similar to Intra Uterinal Inseminnation, substantial benefit is achieved using controlled


ovarian hyperstimulation prior to egg retrieval. Many ova are genetically or functionally abnormal.
Thus, exposure of several ova to sperm results in an increased chance of a healthy
embryo. Optimally, 10 to 20 ova are harvested, and from these, one healthy embryo is ideally
transferred back to the uterus.

Unfortunately, methods to determine embryo health are imperfect. Therefore, to maximize the
probability of pregnancy, more than one embryo is typically transferred, thus resulting in increased
risk of multifetal gestation.

2. Intracytoplasmic Sperm Injection


This variation on IVF is most applicable to male factor infertility.
During the micromanipulation technique of ICSI, cumulus cells surrounding an ovum are
enzymatically digested, and a single sperm is directly injected through the zona pellucida and oocyte
cell membrane.
For azoospermic men, ICSI has made pregnancy in their partners possible. In these cases, sperm are
mechanically extracted from the testis or epididymis.

3. Gestational Carrier Surrogacy


This variation on IVF places a fertilized egg into the uterus of a surrogate, rather than into the
“intended mother.”
Indications are varied, and this approach may be appropriate for women with uncorrectable uterine
factors, for those in whom pregnancy would pose significant health risks, and for those with
repetitive unexplained miscarriage.

4. Egg Donation
Egg donation may be employed in cases of infertility associated with ovarian failure or diminished
ovarian reserve. Additionally, this technique may also be used to achieve pregnancy in fertile women
when off spring would be at risk for maternally transmitted genetic disease.
For highest succes of fresh oocytes reason requires synchronization of the recipient’s endometrium
with egg development in the donor.

To accomplish this, the egg donor completes one of the superovulation protocols.
After a donor receives hCG to allow the final stages of follicle and egg maturation, the recipient
begins progesterone to prepare her endometrium.
In the recipient, estrogen and progesterone are typically continued until late in the first trimester

99
when placental production of these hormones is deemed to be adequate.

5. Gamete Intrafallopian Transfer


This technique is similar to IVF in that egg retrieval is performed after controlled ovarian
hyperstimulation. Unlike IVF, however, fertilization and early embryo development do not take place
in the laboratory. Eggs and sperm are placed via catheter through the fimbria and deposited directly
into the oviduct. This transfer of gametes is most commonly performed at laparoscopy. Like

IUI, GIFT is most applicable for unexplained infertility and should not be considered for tubal factor
causes of infertility.
In general, GIFT is more invasive,
provides less diagnostic information, and requires transfer of more than two eggs for optimal
pregnancy chances, which increases the risk of higher-order multifetal gestation.

6. Zygote Intrafallopian Transfer


This technique is a variant of IVF with similarities to GIFT.
Zygote transfer is not performed directly into the uterine cavity, but rather into the fallopian tube at
laparoscopy. If the transfer is completed after a zygote has begun to divide, the procedure is more
accurately termed tubal embryo transfer (TET).

7. Embryo Cryopreservation
With IVF, many eggs are retrieved to yield ultimately one to three healthy embryos for transfer. This
frequently leads to extra embryos. Successful freezing and thawing of embryos has been
possible for two decades.

8. Oocyte Cryopreservation
Significant technical challenges have been encountered with cryopreservation of unfertilized eggs. At
this time, oocyte cryopreservation is still considered by most to be experimental, and long-term
outcomes are unknown. This technique, however, is proving useful in attempting to preserve the
fertility potential of women facing gonadotoxic chemotherapy.

9. In Vitro Maturation
This technique has been used to achieve pregnancy by aspirating antral follicles from unstimulated
ovaries and culturing these immature oocytes to allow resumption and completion of meiosis in
vitro.
Currently, IVM is considered experimental, and long-term outcomes are unknown.
This technique maybe useful in patients with PCOS in whom stimulation poses a significant risk of
OHSS.
This could potentially allow preservation of fertility potential for women in whom
gonadotoxic chemotherapy is required.

19.B Changes in the reproductive system /uterus and ovaries/ during


pregnancy
Uterus
After conception, the uterus provides a nutritive and protective environment in which the fetus will
grow and develop. It increases from the size of a small pear in its non-pregnant state to
accommodate a full-term baby at 40 weeks of gestation. The tissues from which the uterus is made
continue to grow for the first 20 weeks, and it increases in weight from about 50 to 1,000 grams.
After this time, it doesn’t get any heavier, but it stretches to accommodate the growing baby,

100
placenta and amniotic fluid. By the time the pregnancy has reached full term, the uterus will have
increased to about five times its normal size:
In height (top to bottom) from 7.5 to 30 cm
In width (side to side) from 5 to 23 cm
In depth (front to back) from 2.5 to 20 cm.

Ovaries
If the ovum is fertilized, it is sustained for a short time by the hormones produced by the corpus
luteum. Progesterone and estrogen, secreted by the corpus luteum, are essential for the
preservation of the pregnancy during its early months. If pregnancy does not occur, the egg
disintegrates and the corpus luteum shrinks.
Pregnancy, maintains the corpus luteum by means of the hormones produced by the young placenta.
The corpus luteum is not essential in human pregnancy after the first few weeks because of the
takeover of its functions by the placenta. At the end of pregnancy the corpus luteum has usually
regressed until it is no longer a prominent feature of the ovary.
During the first few months of pregnancy the ovary that contains the functioning corpus luteum is
considerably larger than the other ovary.
During pregnancy, both ovaries usually are studded with fluid-filled egg sacs as a result of chorionic
gonadotropin stimulation; by the end of pregnancy, most of these follicles have gradually regressed
and disappeared.

19.C Asynclitic birth

Asynclitism: This term describes the condition when the fetal head is turned in the maternal pelvis
such that one parietal bone is closer to the pelvic outlet.

Diagnosis:
The position can be determined by finding the location of the sagittal suture and its relationship to
the posterior portion of the ear, if palpable.
If the sagittal suture is in a U formation, an anterior asynclitism presentation should be
suspected. Conversely,
If the sagittal suture is in the shape of an inverted U,
this may indicate posterior asynclitism (ie, posterior
parietal bone) presentation.

Most fetuses can be delivered by forceps if they are in or can


be maneuvered (manually or by forceps) into an occiput
anterior or posterior position.
Correct determination of the position may be the most
important step prior to forceps application.

Management:
Mid-cavity rotational forceps (Keilland’s) assist in
delivery
Vaccum assist in delivery
Ceaseran section

101
Triplet 20

20.A Cervicitis and endometritis


Cervicitis

Def.: Inflammation of the uterine cervix

Etiology:

Non-infectious
o Traumatic (Tampons, contraceptive devices,…)
o Radiation
o Chemical irritation
o Systemic inflammation
o Malignancy
Infectious (more common, all are STIs)
o Chlamydia trachomatis (!)
o Neisseria gonorrhea (!)
o Others: Trichomonas vaginalis, Mycoplasma genitalum, HSV (mainly type 2), HPV
Risk factors: Multiple sex partners, age <25, low socioeconomic status, alcohol/drug use, single
marital status, urban residence

Neisseria:

Symptoms: often + vaginitis, also urethritis, and Bartholin gland infection


o Often asymptomatic (Carriers!)
o Profuse, odorless nonirritating white-yellow vaginal discharge
o Can ascend and cause URTI (endometrium, fallopian tube)
Dg.: G-, intracellular, NAAT (Nucleic acid amplification test) mostly replaced culture and tests
vaginal, endocervical and urine specimens
T.: uncomplicated: single dose IM Ceftriaxon 250 + Azithromycin 1g; Doxycycline 100 mg
twice daily for 7 days; abstinence, + test for other STIs, refer sexual partners
Chlamydia:

Symptoms (caused by invasion of epithelium)


o Mucopurulent discharge or endocervical secretions
o Edematic and hyperemic endocervical tissue
o Dysuria from concurrent urethritis
Dg.: obligate intracellular, Cx, NAAT, ELISA from endocervical specimen
T.: Primary: Azithromycin 1g once or Doxycycline 100 mg twice daily for 7 days, abstinence,
refer sexual partners
Complications:

PID infertility, chronic pelvic pain, ectopic pregnancy


Spontaneous abortion, PROM, preterm delivery
Untreated HSV during pregnancy: mental retardation, blindness, low birth weight, meningitis
HPV: linked with cervical cancer

102
Endometritis

Def.: Inflammation of endometrial lining, may involve myometrium, parametrium

Etiology: usually polymicrobial (Ureaplasma, GBS, Peptostreptococcus, Gardnerella, Chlamydia)

Pregnancy related endometritis


o Postpartum infection
o Retained products of conception (after delivery or abortion), usually chronic
Non-pregnancy related endometritis (referred to as PID)
o PID
o Invasive gynecological procedures
o Intrauterine deivces, usually chronic
Risk factors:

Pregnancy related: CS, prolonged rupture of membranes, long labour, meconium stained
amniotic fluid, manual removal of placenta
PID: IUD, young age, STIs, multiple sexual partners and unprotected sex
Presentation:

Symptoms:
o Fever, lower abdominal pain, malaise
o Abnormal vaginal discharge and bleeding
o Foul smelling lochia (vaginal discharge after birth)
o Dyspareunia
Examination
o Fever, usually 36 hrs after delivery
o Uterine and adnexal tenderness
o Tachycardia
Treatment:

Broad Spectrum ATB, prompt and aggressive to avoid infertility: cephalosporins, clindamycin,
gentamycin
Removal of retained products

20.B Gestational trophoblastic disease


Def.: Gestational trophoblastic disease = tumors originating from the placenta. Characterised by
tumor marker βhCG.

Risk Factors: High maternal age, prior unsuccessful pregnancies, prior GTD, oral contraceptive use

Gestational Trophoblastic Disease

1) Complete Hydatidiform Mole (Molar pregnancy)

Chorionic villi in placenta show trophoblastic proliferation

103
Complete diploid karyotype. Chromosomes are entirely of paternal origin. No fetal tissue or amnion
is produced.

Clinical: vaginal bleeding, higher than expected βhCG levels, uterine size, anemia; if removed late:
hyperemesis gravidarum, preeclampsia,

Nowadays: most are removed at around 12th week.

1. Incomplete (partial) Hydatidiform Mole

Decreased trophoblastic proliferation than complete. Mostly also contain (nonviable) fetal tissue and
amnion. Triploid karyotype (maternal and 2 paternal sets).

Clinical: vaginal bleeding, elevated βhCG, uterine size corresponds to GA

Dg.: usually USG, also βhCG; histopathology, ploidy determination, immunostaining

T.: Suction curettage, Rh Immune globulin given to Rh- women (partial mole)

Complications: GTN develops in 15% of complete moles postmolar surveillance (βhCG),


prophylactic chemotherapy

2. Gestational Trophoblastic Neoplasia

Gestational trophoblastic neoplasia (GTN) = subset of GTD that develops malignant sequelae

Trophoblastic cells invade endo- and myometrium.

Histologic Classification/Types

Invasive Mole (local invasion, rarely spreads)


Gestational Choriocarcinoma (extremely malignant, early metastases)
Placental Site trophoblastic tumor (infiltrate mostly within uterus, bleeding months or years
after pregnancy)
Epithelioid Trophoblastic Tumor (nodular, long after pregnancy)
Etiology: Develops with/follows some form of pregnancy, mostly follow a hydatidiform mole

Clinical: metrorrhagia, preceding pregnancy (mole, carried to term, abortion)

Dg.: persisting plateau of βhCG, correlating with USG

Evaluation: Pelvic CT, CXR, MR of the brain

T.: Chemotherapy (Methotrexate, Dactinomycin), Surgery: chemotherapy resistant disease,


emergency procedure (bleeding), adjuvant hysterectomy, primary treatment for placental site
trophoblastic tumor and epitheloid trophoblastic tumor (chemotherapy resistant)

104
20.C Rhesus incompability
Pathophysiology:

Mother is Rh-ve (d/d), fetus may be Rh +ve (father has to be Rh +ve)


Fetal cells cross into the maternal circulation in normal pregnancy ( in sensitizing events)
The foreign Ag causes immune response: isoimmunisation production of IgM (cannot
cross placenta)
Subsequent pregnancy with Rh +ve fetus : memory B cells produce IgG IgG crosses into
fetal circulation.
IgG binds to fetal red cells, which are destroyed in the reticulo- endothelial system. 

Causes a haemolytic anaemia (if erythropoesis is inadequate to compensate, severe anaemia
causes high output cardiac failure, ‘fetal hydrops,’ and, ultimately, death). 

In milder cases, haemolysis leads to neonatal anaemia or jaundice from increased bilirubin
levels. 

Potential sensitizing events: <0.1ml are sufficient!

Termination of pregnancy, evacuation of retained products of conception


Ectopic pregnancy
Vaginal bleeding > 12wks, or earlier if heavy. 

ECV (External cephalic version in breech position)
Blunt abdominal trauma
Invasive uterine procedure, e.g. amniocentesis or CVS (chorionic villous sampling)
Intrauterine death
Delivery

Fetal consequences:

Haemolytic anemia: increased fetal hematopoiesis (in liver!)


Production of other proteins therefore decreased ascites, subcutaneous edema, pleural
effusions.
Anemia
o High output cardiac failure ( effusions, ascites, edema)
o Myocardial ischemia, directly compromising cardiac function.
Fluid accumulation in at least two extravascular compartments (pericardial effusion, pleural
effusion, ascites, or subcutaneous edema) is referred to as hydrops fetalis.
Dg.: At first visit: pregnant woman tested for ABO and Rh D blood type, and ery. Ab. Repeated at
each pregnancy.

Assessment:

Ab titers: don’t reflect fetal well-being, only risk


Amniotic fluid assessment: bilirubin levels
USG:
o Measurement of peak velocity of middle cerebral artery. Gives information about
blood viscosity.

105
o Visualize hydropic changes in fetus
o Cordocentesis (Percutaneous umbilical blood sampling): Fetal Hct in severe anemia
<30%
Management:

Prenatal:

Hct <30% is indication for blood transfusion into umbilical vein. Can be performed after 18 wks.
Repeated as long as MCA flow is abnormal. Risk of fetal loss 1-3% after 26th week.

Postnatal:

Correct anaemia and possible coagulopathy.

Phototherapy to treat hyperbilirubinaemia.

Monitor Hct, Ab can persist for weeks.

Prevention: anti-D-Ig, 300μg

Within 72 hours of delivery to Rh –ve women who deliver an Rh +ve child.


At 28 wks GA to all Rh –ve women
Prophylactic does when a feto-maternal haemorrhage can occur
After large feto-maternal haemorrhages: haemorrhage can be assessed by Kleihauer test to
determine amount of haemorrhage and find appropriate anti-D-Ig dosage.

Triplet 21

21.A Adnexitis and parametritis


Adnexitis
Def.: Inflammation of the adnexa = fallopian & ovaries. Also called oophorsalpingitis. Often used
synonymously with PID, but PID lacks accurate definition (upper reproductive tract inflammation).
Pyosalpinx refers to an accumulation of pus in the fallopian tube.
Classification: acute/chronic
Etiology: usually ascending infection from lower reproductive tract. Often simultaneous with
cervicitis and endometritis. Commonly after menstruation or ovulation – cervical mucous is easy to
penetrate.
Common pathogens: C. trachomatis, Mycoplasma, E. coli, N. gonorrhea, S. aureus
Clinical presentation:

Symptoms

Acute lower abdominal pain, often post-mestrual or post-ovulatory


Vaginal discharge or bleeding when concurrent with cervicitis/endometritis
Dyspareunia
Nausea and vomiting in severe cases

106
Signs

Adnexal tenderness
Cervical motion pain
Dg.:

Examination in specuala: cervicitis


Usg: visualisation of abscesses, fluid accumulations
CBC: Leukocytosis, elevated CRP
DD.: ectopic pregnancy, rupturing ovarian cyst, appendicitis, IBD, appendicitis, diverticulitis

Complications:

Formation of Obstructions and Adhesions, Infertility


Pyosalpinx, Hydrosalpinx, Haematosalpinx
Abscess (Tuboovarian abscess, Douglas abscess)
Peritonitits, acute abdomen, Hugh-Fitz-Curtis
Sepsis
Chronis andexitis
Therapy:

Cephalosporins, Tetracyclines, Fluoroquinolones


Surgical evacuation of abscesses
Chronis adnexitis: often difficult to treat due to adhesions
Parametritis

Def.: Infection and inflammation of the parametrium (connective tissue surrounding and fixating the
uterus)
Etiology:

Pathogens: Streptococcal and Staphylococcal infection


Mostly occurs as complication surgery (SSI), after delivery, radiotherapy

Clinical

Signs and symptoms:

Abdominal pain, radiating into gluteal area


Fever
Dysuria, dyschezia
Treatment: ciprofloxacin + metronidazole, drainage of abscesses

107
21.B Fetoplacentar unit function
The placenta is an essential and unique “organ of pregnancy,”
Point of connection between the mother and fetus
Necessary for the fetus to live and grow in the uterus

Essential functions of the placenta

To anchor the fetus and establish the fetoplacental unit.


Organ for gaseous exchange.

Endocrine organ to bring the needed changes in pregnancy.
Transfer of substances to and from the fetus
Barrier against infection

Placenta as endocrine organ:

The production of human chorionic gonadotrophin (hCG), oestrogens, and progesterone by the
placenta is vital for the maintenance of pregnancy. See question on hormones of the placenta.
(Triplet 44/b)

Placenta as barrier:

Protects the fetus from pathogens and the mothers immune system. Most pathogens can not cross
the placental barrier; exceptions are syphilis, parvovirus, HBV, HCV, HIV, rubella, CMV.
Many drugs cross the placenta, exceptions are: LMWH. Some drugs have little effect on the fetus and
are considered safe (eg paracetamol). Others are teratogenic and cause severe congenital anomalies.

Placental circulation:
Consists of 2 entities: uteroplacental circulation and the fetoplacental circulation.

Uteroplacental Circulation

= maternal blood circulating through intervillous space (150ml) – blood from spiral arteries
pressure and conctentration gradient between intervillous space and fetal capillaries cause oxygen
and nutrients to be transferred to fetus. Uterine veins drain deoxygenated blood (only during uterine
relaxation)

Fetoplacental circulation

Two umbilical arteries carry deoxygenated blood from fetus and enter chorionic plate. Divide into
small branches and enter chorionic villi. Oxygenation of fetal blood (80%). Flows into umbilical vein.
Additionally to O2 transfer of water, glucose, amino acids, vitamins and inorganic salts diffuse freely.
Umbilical Cord
Vascular cable connecting fetus to placenta; contains 2 arteries (with deoxygenated blood) and one
vein (oxygenated blood); embedded in Wharton’s jelly.

108
21.C Failure of uterine contractions during labour
3 factors play a role in normal delivery: Power (Uterine factors), Passenger (Fetal Factors), and
Passage (Maternal Factors). Abnormalities lead to “Dystocia” = difficult labor.

Causes
Uterine factors: necessary for cervical dilatation and fetal descent.

each uterine contraction must generate at least 25 mm Hg of peak pressure. (optimum 50-
60mmHg)
frequency: minimum 3 contractions in a 10 min interval
Increased frequency: fetal oxygenation may be compromised.
Measurement: Montevideo unit: number of contractions in 10 min times average intensity.
Labor associated with 200 or more MVU.

Fetal factors: Weight, presentation, position, attitude


Maternal factors: Inadequate pelvis, cephalopelvic disproportion (CPD)

Classification: Failure to progress:

1st/2nd stage
Protraction/Arrest

Diagnosis and Management: documentation of cervical dilatation+effacement, and head descent.

1st stage: divided into

- latent phase: time to effacement of cervix and dilatation up to 3cm


o >20 hrs in nulliparous
o >14 hrs in multiparous

Management:

observation and sedation (she may not actually be in labor)


Interactions to augment uterine contractions

- active phase: 3cm to full dilatation (10cm)


o dilatation rate of <1cm/hour (<1.2-1.5 in multiparous); arrest no dilatation >2 hrs (> 4
hrs w/epidural)

Management:

Observation
Interactions to augment uterine contractions
CS fetal or maternal status becomes non-reassuring

Interactions to augment uterine contractions

109
Augmentation = stimulation of uterine contractions

Amniotomy: < 3 contractions per 10 minutes or <25mmHg


Oxytocin administration

2nd stage:

- arrest: no descent after 2 hour of pushing


- >3hrs w/ epidural
- w/o epidural: > 2hrs or descent of head at a rate of < 1cm/hour

Management:

FHR monitoring
Rule out cephalopelvic disproportion
Bearing-down efforts by patient, labor positions
Operative vaginal delivery, CS in case of non-reassuring fetal/maternal status.

Risks associated with prolonged labor:

Chorioamnionitis
Fetal infection, bacteremia, pneumonia
Risks of CS, operative delivery

Monitoring:

FHR, every 15 min or continuously on CTG


Contractions (every 30 min)
Maternal puls (hourly)
Maternal BP (4h)
Vaginal examination (4h)
Maternal urine test for ketones and proteins

110
Triplet 22

22.A Preservation of fertility in cancer patients, cryopreservation of


gametes and embryos
Cancer patients face the potential loss of reproductive function and the opportunity to
have children.
At the same time that patients receive a diagnosis of cancer, they also must consider
possible effects on fertility.
To preserve fertility, they may need to accept changes in standard treatment protocols or
undertake steps to preserve gametes or gonadal tissue that carry their own risks and
uncertainties.
Men in these circumstances sometimes find producing sperm highly stressful. Women have
more options, but all are more intrusive
Cryopreservation is freezing tissue or cells in order to preserve it for the future.

Factors affecting the extent of chemotherapy induced gonadotoxicity:

1. Type, duration, dose


2. Gonadotoxicity induced by chemotherapy is almost irreversible
3. Risk of damage increases with age (Loss of oocytes)
4. Temporary or permanent amenorrhea

Factors affecting the extent of radiotherapy induced gonadotoxicity:

1. Patients age
2. Dose of radiation
3. Extent/Severity
4. Type and location of radiation
5. Fractionation of the total dose

Consequences of chemotherapy and radiotherapy:

1. Premature ovarian failure (POF)


2. Early pregnancy loss ( abortions)
3. Premature labour
4. Low birth weight

Fertility Preservation Strategies:

1) Pharmacological Protection
Gonadotrophin-releasing hormone analogues (GnRHa) are the only available medical
protection for chemotherapy. o There is a significantly reduced loss of primordial
follicles during chemotherapy when GnRHa is given in parallel.
2) Ovarian Transposition
Prior to radiotherapy, the ovaries can be transposed out of the radiation field surgically.
o There are two types of transposition: lateral ovarian transposition (LOT) and medial
ovarian transposition (MOT).

111
In the case of LOT the ovary is fixed in the paracolic gutter to the pelvic side wall, which
requires laparoscopic mobilization of the ovary via the ovarian ligament. The ovary can
then be moved laterally via a peritoneal incision and lateral tunneling. To prevent
migration, the peritoneal opening into the pelvic side wall is narrowed. The ovary is
marked with a titanium clip for ease of localization before radiotherapy.

In the case of MOT, both ovaries are fixed to the posterior wall of the uterus with purse
string sutures. Again, a titanium clip is applied. Following radiotherapy the non-
absorbable sutures can be removed and the ovaries restored to the anatomical position.

Complications:

Fallopian Tube Infarction Chronic Ovarian Pain

Ovarian Cyst Formation Ovary Migration

Ovarian Metastasis

Laparoscopic ovarian transposition is a good option if radiotherapy is to be used.

3) Oocyte cryopreservation
o For single women, ethically accepted.
o Oocytes are more sensitive to freezing-thawing procedures than embryos.
o Results are still very low.
o Alternative strategy is to freeze immature oocytes ( primordial follicle).
o Oocyte cryopreservation is gaining popularity

4) Cryopreservation and Transplantation of ovarian tissue


o Still experimental procedure.
o Limited studies.
o Primordial follicles should have better survival rates.
o In vitro - growth of primordial follicles.
o Transplanted back into patient, after cryopreservation.

5) Ovarian cortical strips transplantation.


- in the pelvic wall.
- in the forearm.
- lower abdominal skin

6) Cryopreservation of gametes and embryos


Cryopreservation of embryos is the process of preserving an embryo at sub-zero
temperatures, generally at an embryogenesis stage corresponding to pre-implantation,
that is, from fertilisation to the blastocyst stage.
Cryopreservation holds tissues at temperatures between -140 and -200C, at which no
biological activity can occur, producing a state of "suspended animation" of tissues that
can be maintained indefinitely
Embryo cryopreservation is generally performed as a component of IVF (which generally
also includes ovarian hyperstimulation, egg retrieval and embryo transfer). The ovarian

112
hyperstimulation is preferably done by using a GnRH agonist for final oocyte maturation,
since it decreases the risk of ovarian hyperstimulation syndrome with no evidence of a
difference in live birth rate.
The main techniques used for embryo cryopreservation are vitrification versus slow
programmable freezing (SPF). Studies indicate that vitrification is superior or equal to SPF
in terms of survival and implantation rates. Vitrification appears to result in decreased
risk of DNA damage than slow freezing.
Direct Frozen Embryo Transfer: Embryos can be frozen by slow programmable freezing (SPF)
method in ethylene glycol freeze media and transfer directly to recipients immediately after
water thawing without laboratory thawing process.

22.B Central nervous and respiratory system development


Central Nervous System:

Week 3:
CNS appears as a slipper-shaped plate of thickened ectoderm, longitudinal groove on the
neural plate.
The lateral sides elevate to form neural folds.
Ultimately these folds meet and form into a closed tube.
This tube differentiates into three vesicles:
- Prosencephalon at the front
- Mesencephalon
- Rhombencephalon between the spinal cord and mesencephalon
By Week 6:
The prosencephalon divides into the Telencephalon and Diencephalon o The
rhombencephalon divides into Metencephalon and Myelencephalon

As the vertebrae grows, these vesicles differentiate further.


Telencephalon into the hippocampus, striatum and neocortex, its cavity
becomes the 1st and 2nd Ventricles.
Diencephalon into the subthalamus, hypothalamus, thalamus and epithalamus, its
cavity forms the 3rd Ventricles.
Mesencephalon into the tectum, pretectum, cerebral peduncle and its cavity
grows into the Mesencephalic Duct.
Metencephalon becomes the pons and cerebellum.
Myelencephalon forms the medulla oblongata, and the cavity formed is the fourth
ventricle.

113
Respiratory:

Embryonic Stage (4-5 Weeks):


1. Endonderm - Tubular Ventral growth from foregut pharynx
2. Mesoderm - Mesenchyme of lung buds
3. Intraembryonic coelom - Pleural cavities elongated spaces connecting pericardial and
peritoneal spaces

Lung buds originate as an outgrowth from the ventral wall of the foregut where lobar
division occurs

Pseudoglandular Stage (5-17 Weeks):


1. Tubular branching of the human lung airways continues
2. By 2 months all segmental bronchi are present
3. Lungs have gland-like structure
4. Stage is critical for formation of all conducting airways

Canalicular Stage (16-24 Weeks):


1. Lung morphology changes
2. Bronchioles are produced
3. Increasing number of capillaries in contact with epithelium
4. Beginning of alveolar development

Saccular Stage (24-40 Weeks):


1. Most peripheral airways form airspaces called saccules
2. Alveolar ducts and air sacs are developed

Alveolar Stage (Late Fetal to 8 years old):


1. Near term through postnatal period
2. 1-3 years postnatally aveoli form through a septation process increasing the gas exchange
surface area
3. Increase in number and size of capillaries

114
115
22.C Pregnancy and delivery in cardiac disease
o In pregnancy there is a 40% increase in cardiac output, due to an increase in stroke
volume, heart rate and increase in blood volume.
o Blood pressure often drops in the 2nd trimester but is usually normal by term.

Treatment:

o Patients with significant diseases are assessed before pregnancy.

o Some drugs, such as warfarin & ACE inhibitors are contraindicated.

o Thromboprophylaxis is continued but usually with low molecular weight heparin (LMWH).

o Antibiotics in labour are recommended for some to protect against endocarditis.

o Vaginal Delivery is preferred for most women with heart disease unless there are specific
obstetric indications / deterioration in cardiac performance requiring early delivery.

Types & treatment of Cardiac Disease:

o Mitral valve prolapse, patent ductus arteriosus, ventral / atrial septal defects usually don’t
cause complications.

o Pulmonary hypertension: Because of high maternal mortality (40%), pregnancy is


contraindicated & usually terminated!

o Aortic stenosis: Severe disease causes an inability to increase cardiac output when required
& should be corrected before pregnancy (Beta-blockers are often used).

o Mitral valve disease: Should be treated before pregnancy. Heart failure may develop late in
pregnancy. Beta-blockers are used. Artificial metal valves are prone to thrombosis, warfarin
is used after 12 weeks despite the fetal risks.

o Myocardial infarction (MI): Unusual in women of reproductive age.

o Peripartum cardiomyopathy: idiopathic & Rare cause of heart failure. Develops in last month
of pregnancy up to 6 months after delivery.
o Frequently diagnosed late.
o Treated supportively (diuretics & ACE inhibitors).

116
Triplet 23

23.A Pelviperitonitis and peritonitis diffusa


Pelviperitonitis

Def.: Inflammation of the peritoneum (serosal membrane lining the abdominal cavity) limited to the
small pelvis.

Etiology: Commonly arises as complication (direct continuation) of upper reproductive tract


infection. Common pathogens: C. trachmoatis, N. gonorrhea.

Signs and Symptoms:

Strong lower abdominal pain - exacerbated on movement


Abd. Tenderness, rigidity, guarding
Obstipation, Nausea, Vomiting, Flatulence
Symptoms originating from cervicitis or salpingitis such as discharge, metrorrhagia,
menorrhagia, dyspareunia
Fever
Dg.: Physical examination, Examination in specula, microbial testing, infectious parameter, USG

DD.: Ectopic pregnancy, appendicitis, Diverticulitis, IBD

Complications: Diffuse peritonitis, Ileus, Infertility, Sepsis

T.: ATB (ceftriaxone, ciprofloxacin, metronidazole), antiphlogistics

Peritonitis diffusa

Def.: Inflammation of the peritoneum

Etiology:

Infected peritonitis:
o Perforation of part of the GIT (ruptured appendicitis, diverticulitis, abdominal
trauma,…) – E. coli, Becteroides
o Disruption of the peritoneum (SSI, peritoneal dialysis) – S. aureus
o Spontaneous bacterial peritonitis
o Systemic infections (TB)
o Sequelae of PID (adnexitis, parametritis,…) – N. gonorrhea, C. trachomatis
Non-infected (sterile) peritonitis:
o Sterile body fluids reach abdomen (blood, gastric juice, bile, urine, menstruum,
pancreatic juice, ruptured dermoid cyst)
Signs and Symptoms:

Abdominal pain, tenderness, guarding - exacerbated on movement (peritonism)


Blumberg sign +ve
Bad general constitution, including fever and tachycardia
Paralytic ileus

117
Complications:

3rd space loss – hypovolemia – shock – AKI


Abscess formation
Sepsis, multiorgan dysfunction
Dg.:

Clinical presentation
Blood work (infectious parameters, but non-specific)
Diagnostic paracentesis (Biochemical, microbiological, cytological examination) +/- USG
CT, Laparoscopy

Tx.:
Control the infectious source
Eliminate bacteria and toxins
Maintain organ system function
Control the inflammatory process
Fluid resuscitation, correction of potential electrolyte and coagulation abnormalities, and empiric
broad-spectrum parenteral antibiotic coverage, laparotomy

23.B The development of the cardiovascular system and fetal blond


circulation
The cardiovascular system begins to develop in the third week of gestation. Angiocysts develop in the
extraembryonic mesoderm, and consist of (a) a central group of haemoblasts, the embryonic
precursors of blood cells; (b) endothelial cells.

Angiocysts arteries and veins


Angiocysts form in the cardiogenic
mesoderm and coalesce to form a
pair of endothelial heart tubes.

The endothelial heart tubes fuse to form a single


primitive heart tube with a cranial (arterial) end
and a caudal (venous) end.

118
The primitive heart tube is divided into a
number of primitive chambers separated by
grooves.

By the end of 3rd week, circulation is established:


3 pairs of arteries (to the head, umbilical
arteries, vitelline arteries to yolk sac). Circulation
starts.

The heart tube folds and


becomes S-shaped. The
Cardiovascular system is the first
system to reach a functional
state.

Fetal circulation:

Oxygenated blood carried to the fetus by umbilical vein. 1/3 enters the ductus venosus (shunting
oxygenated blood directly to the fetal brain), 2/3 enter the liver. From the liver portal vein right
atrium of heart.

From right atrium most of the blood flows through foramen ovale to left atrium (bypassing
pulmonary circulation). From left atrium left ventricle aorta body. Part of blood from aorta
to umbilical arteries placenta gas exchange, waste products, nutrients.

Some blood from right atrium enters right ventricle and pulmonary artery. Pulmonary artery and
aorta are connected by ductus arteriosus.

23.C Abruptio placentae praecox


Def.: Premature separation of an otherwise normally implanted placenta (praecox = premature).
Abruption can be complete or partial; it can be concealed (with no apparent bleeding, blood trapped
behind placenta, <20%) or revealed (vaginal bleeding).

Incidence: 0.5-1% of births.

119
Risk factors: Chronic hypertension, preeclampsia, multiple gestations, choriomanionitis, trauma (Car
accidents-seat belts!), previous placental abruption,

Presentation:

Typically sudden onset, constant and severe abdominal pain


Vaginal bleeding (Remember: haemorrhage may be much greater than apparent vaginal
loss), bleeding is often dark
Tender, hard uterus
50% will be in labour
Maternal signs of shock
Fetal distress, preceding fetal death
Dg.: Clinical. USG: Confirm fetal well-being and exclude placenta praevia.

Management:

Admit all women with vaginal bleeding and/or abdominal pain.


CTG, USG, Blood works. Monitoring + fluid administration.
In case of fetal distress or maternal compromise: Resuscitate and deliver. Often CS
If bleeding and pain cease, deliver by term.
Complications:

Fetal or maternal death, coagulopathy, DIC, Couvelaire uterus (blue discoloration)

Triplet 24

24.A Benign lesions of the vulva, vagina and cervix uteri


1) Vulva

Vulvar Dermatoses.

Lichen Simplex Chronicus. Pruritus from environmental factors (clothing, heat, sweat,
chemical substances) triggers itch-scratch cycle leading to chronic trauma; skin responds by
thickening (=lichenification). Skin is thick, gray, and leathery. T: Topical corticosteroids.
Antihistamines. Stop scratching.
Lichen sclerosus. Chronic inflammatory condition. Typically in postmenopausal women.
Idiopathic, suspected autoimmune etiology. Inflammation of nerves terminals causes
pruritus + itch-scratch cycle. Figure 8 appearance. Dyspareunia, burning, skin fragility and
architectural changes (labia minora regression, urethral obstruction). T.: Biopsy, symptom
control to prevent scarring with ultra-potent steroids, surveillance (malignant transformation
in 4-6%)
Inflammatory Dermatoses.

Contact dermatitis. Irritant or allergic. Leading to skin inflammation. Eliminate agent.


Intertrigo. Friction between moist skin surfaces; often complicated with bact. or fungal
infection. Initially erythematous, later erosions, exudate, fissures. Cx. Drying agents.

120
Atopic eczema. Severe pruritic dermatitis, chronic relapsing. Scaly patches+fissuring.
(+allergic rhinitis, asthma!) T.: local hydration.
Psoriasis. Occasionally affects mons pubis or labia. Can be exacerbated by menses. T.:
steroids.
Lichen planus. Uncommon disease involving cutaneous and mucosal surfaces, possibly drug-
induced. Erosive, papulosquamous or hypertrophic. Polygonal papules on trunk, buccal
mucosa, flexor surfaces. Chronic vaginal discharge and pruritus. T.: ultra-potent topical
steroids.
Hidradenitis suppurativa. Recurrent popular lesions with abscess and fistula formation in
axillar, inguinal, perianal, perineal skin. T.: immunosuppressive t.
Apthous ulcers. Self-limited mucosal lesions. Recurring. Painful. T.: steroids.
Vulvar manifestations of generalized conditions. Acanthosis nigricans, Morbus Crohn, Behcet.

Disorders of pigmentation. Nevus. Vitiligo.

Solid vulvar tumors.

Acrochordon. Skin tag, benign polypoid fibroepithelial lesion


Seborrheic keratosis
Kerathoacanthoma
Syringomyoma
Leiomyoma
Fibroma
Lipoma
Ectopic breast tissue
Cystic Vulvar Tumors.

Bartholin gland duct cyst and abscess. Result from blockage of glands in labia majora.
Epidermoid, Sebaceous, Mucinous, cysts.
Condylomata acuminata: HPV 6/11, sessile polypoid mass.
2) Vagina

Foreign Body. Tampons, condoms,…Vaginal discharge, pruritus, discomfort. T.: Removal


Desquamative Inflammatory Vaginitis. Idiopathic condition. Copious diffuse, exudative,
purulent yellow or green discharge. Erythema. Absent infection. T.: hydrocortisone or
clindamycin.
Gartner Duct Cyst. Cyst developing from remnants of Wolffian duct. Marsupialization.
Endometriotic deposit.
Inclusion cyst. Vaginal epithelium embedded under surface during perineal surgery.
3) Cervix

Cervical polyps (adenoma): common. Very rarely malignant. Should be removed.


Hysteroscopy to rule out further polyps.
Nabothian cysts: mucous-retention cysts caused by blockage of endocervical mucous glands.
No T. required.

121
24.B Termination of pregnancy
Elective: TOP required w/o medical reason.

Therapeutic: TOP performed to save the pregnant woman’s life, to protect physical or mental health,
indication that the child will be born with severe mental or physical abnormalities, selectively reduce
the number of foetuses in multiple pregnancies.

Legal aspects:

Legislation varies throughout the world, with terminations remaining illegal in some countries. In
most European countries elective TOP is legal in the 1st trimester, and medically indicated TOP legal
within the 2nd trimester. The Abortion Act of 1967 legalized abortion in the UK and identified 5
categories:

A. Continuance of the pregnancy would involve risk to life of pregnant woman greater than
if pregnancy were terminated.
B. Termination is necessary to prevent grave permanent injury to physical or mental health
of pregnant woman. 

C. Pregnancy has not exceeded 24th week and continuance of 
the pregnancy would
involve risk, greater than if pregnancy were terminated, of injury to physical or mental
health of pregnant woman. 

D. Pregnancy has not exceeded 24th week and continuance of pregnancy would involve
risk, greater than if pregnancy were terminated, of injury to physical or mental health of
any existing child(ren) of family of pregnant woman. 

E. There is a substantial risk that if the child were born it would suffer from such physical or
mental abnormalities as to be seriously handicapped.

A, B, and E have no time limit. Clauses C and D have a legal limit of 24wks.

Methods:

Surgical
o Avoid before 7 wks
o 7-13wks: suction termination
o >13wks: dilatation and evacuation, suction or extraction forceps, with cervical
preparation (eg misoprostol, gemeprost or mifepristone prior to surgery), risk of
incomplete evacuation with increasing GA
Medical
o <9wks: mifepristone priming + prostaglandin
o 9-13 wks: appropriate + safe alternative to surgical TOP
o 13-24 wks: appropriate + safe, consider feticide >20wks
o Not always complete, might require suction after.
- Methothrexate: antimetabolite

- Mifepristone: antiprogesterone, increases contractility+bleeding from placental bed

- Misoprostol: PGE analogue, stimulates myometrium, cervical preparation

122
- Gemeprost: PGE analogue, dilatation of cervix

Complications:

Uterine perforation
Cervical laceration
Haemorrhage
Incomplete removal of products of conception
Infection, septic abortion
Post-abortal syndrome (uterus fails to contract, T.: suction curettage)

24.C Hypertensive disorders of pregnancy


Normal changes of BP in pregnancy:

In early pregnancy until 24 wks

After 24 wks until delivery via in stroke volume

After delivery, but may peak again 3–4 days post-partum

Pregnancy induced (gestational) hypertension.:

Def.: (>/=140/90) in second half of pregnancy w/o proteinuria or other markers of preeclampsia.

Risk for developing preeclampsia. Plan to deliver at EDD. BP returns to normal after delivery.

Chronic hypertension:

HT present before 20 wk GA or persisting >12wks after delivery. >140/>90. Increased risk for
developing pre-eclampsia and eclampsia. 3-5% of pregnancies.

Post-partum HT:

BP peaks normally on 3rd-5th postnatal day. Determine whether pre-existing chronic HT, physiologic,
or new-onset preeclampsia.

Preeclampsia:

Def.: Multisystem disorder characterized by HT and PU thought to arise from placenta. Wide
spectrum of severity & different presentation.

BP≥140/90 and ≥300mg proteinuria/d, previously no HT, after 20wk GA. In women who were
hypertensive before: rise in sBP≥30 or dBP ≥15.

Risk factors: Nulliparity, multifetal gestation, women > 35, previous preeclampsia, chronic HT,
pregestational DM, vascular and connective tissue disorders, nephropathy and CKDs,
antiphsopholipid sy, obesity, African American ethnicity.

123
Signs and symptoms of severe preeclampsia: one or more of the following:

BP ≥160/≥110, marked proteinuria (≥5g/d), oliguria, cerebral disturbances, flashes, pulmonary


edema, right upper quadrant pain, IUGR, hepatic dysfunction, thrombocytopenia, hyperreflexia

Complications: Eclampsia, HELLP, Cerebral haemorrhage, IUGR and fetal compromise, Renal failure,
placental abruption.

Management: the only treatment is delivery, can be delayed with intensive monitoring (<34)

expectant: keep BP <160/<110, PU<300mg/d, warn about development symptoms, review


after 1-2 wks, in mild to moderate: every 4hrs BP, daily urinalysis, daily CTG
Antihypertensives if BP >160/>110. Does not treat pre-eclampsia but aims to prevent
complications. Nifedipine, methyldopa, labetalol. Not below <120/<80!
Immediate delivery: worsening of thrombocytopenia, worsening of liver or renal function,
severe maternal symptoms (abnormal LFT+RUQ pain), HELLP sy/eclampsia, abnormal CTG,
reversed umbilical artery flow.
<34 wks GA: steroids

Eclampsia: obstetric emergency!

Additional presence of convulsions (grand-mal or tonic clonic) in a woman with preeclampsia that is
not explained by a neurologic disorder. In up to 4% of patients with preeclampsia. Seizures may occur
antenatally (38%), intrapartum (18%), postnatally (44%) within first 48 hrs.

Management:

ABC + IV access
MgSO4 control of seizure and prevention of further seizures
Diazepam in further seizures
Monitoring and assessment of reflexes (Mg toxicity!)
If hypertensive: nifedipine, labetalol
Fluid restriction
CTG monitoring of fetus, deliver fetus once mother is stable, vaginal deilvery is not
contraindicated.

124
Triplet 25

25.A Ovarian cancer


Epidemiology:

Leading cause of death from gynecological malignancies


90% are epithelial ovarian cancer, most of them serous and endometrioid adenoca
Peak: 75-84 yrs

Etiology/Risk factors: suspected to be due to irritation of ovarian surface epithelium

Risk if ovulations
Nulliparity
Early menarche and/or late menopause
Risk in COCP
Pregnancy risk
BRCA mutations

Prevention:

Ovarian cancer screening – focus on carriers of BRCA mutations – pelvic examination, USG, CA125

Presentation, Signs, Symptoms:

Vague, common symptoms 75% present when disease has spread to abdomen
Common sy: abd distension, persistent bloating, increased girth, urinary sy, change in bowel
habit, abnormal vaginal bleeding, detection of pelvic mass
Tumor markers: CA 125
Abdominal, pelvic USG: pelvic mass, ascites
CXR for metas, CT abd, pelvis

FIGO Staging:

I: Growth limited to fallopian tubes

II: Growth involving one or both fallopian tubes with pelvic extension. (Limited to pelvis)

III: + peritoneal implants and/or +ve regional LN. Superficial liver metas. (Limited to abd.)

IV: Distant metas.

Treatment:

Surgery:
- Midline laparotomy aims to remove as much tumour as possible (=cytoreduction):
Laparotomy, Hysterctomy, bilateral adnexectomy, omentectomy, LN sampling (pelvic and
para-aortic), peritoneal biopsies, pelvic washings, ascetic samplings.
- III/IV may benefit from neoadjuvant chemotherapy

125
Chemotherapy:
- Adjuvant chemo recommended for most patients
- 6 cycles of carboplatin ± cisplatin every 3 wks

Prognosis:

45% overall 5-yr survival rate. Favourable factors: younger age, good performance status, well
differentiated tumor, smaller disease prior surgery, no ascites, smaller tumor residuum after surgery

Tumors other than epithelial ovarian cancer:

Germ cell tumors: arise form the ovary’s germinal elements, 1/3 of all ovarian neoplasms, mature
cystic teratoma (=dermoid cyst) most common of these. Common features differentiating them from
EOC: present at young age (teens, or early 20s), most have stage I disease, excellent prognosis due to
good tumorsensitivity.

Primitive germ cell tumors:


o Dysgerminoma
o Yolk sac tumor
o Embryonal ca.
o Polyembryoma
o Nongestational chorioca.
Teratoma
Mixed forms
Ovarian Sex cord-stromal tumors: Rare. Originate from ovarian matrix. 90% of SCST are hormone-
producing. will present w/ sy of excess estrogen or androgen. Surgery is primary therapy. Slow
progression and low malignant potential.

Granulosa-stromal cell tumors


Sertoli-stromal cell tumors

25.B Prenatal care


Purpose:

Give evidence based information


Assess risk
Monitor fetal and maternal well-being
Determine timing and mode of delivery

Booking visit:

Ideally in early pregnancy (<12th week)


Discussion of sensitive issues should be possible
History taking + complete physical examination
Hx of past pregnancies, family medical Hx, drug use, risk factors, inheritable diseases
Lifestyle advice (alcohol abuse, smoking, addictive drug use, diet and exercise)

126
Information on antenatal screening and folic acid supplementation
Identify risk for woman and fetus
Calculation BMI
BP
Dipstick Urine (Protein, glucose, blood)
USG for GA and gross anomalies
Blood test: blood type, hemoglobinopathies, anemia, allo-Antibodies; infection screening:
rubella, VDRL, HIV, HBV

2nd trimester: always: *BMI, urine dip, BP, plot SFH

16wks: Discuss screening results, investigate Hb <11


25 wks: *
28 wks: screening for anemia, allo-Antibodies; anti D prophylaxis for Rh –ve,*
3rd trimester: always: *BMI, urine dip, BP, plot SFH; monthly visits

31 wks: *
34 wks: discuss labour, anti D prophylaxis for Rh –ve, BP, *
36 wks: discuss breast-feeding, vit-K-prophylaxis, postnatal self care, postnatal depression, *
38 wks: *
40 wks: *, information about prolonged pregnancy
41 wks: membrane sweep
42 wks: induction of labour

25.C The transverse and oblique fetal presentation


Def.: Axis of the fetus is across the axis of the uterus. Common before term, in 1% of fetuses after 37
wks. Unstable lie: Lie is changing several times a day.

Etiology/Risk factors:

Multiparity (especially > 2) with lax uterus


Polyhydramnios
Uterine abnormalities (eg fibroids)
Fetal abnormalities
Multiple pregnancy
Evaluation:

Assess stability: position still changing?


Palpation asses fetal lie, Leopold maneuver
Neither head nor buttocks will be presenting (presenting= part that enters pelvis first); does
the presenting part move?
Assess laxity of uterine wall
USG
Management:

Admission to hospital – CS can be carried out if labour starts


If lie is stable but not longitudinal: CS at 39 wks
If lie is not stable: CS at 41 wks
Complication/Risks:

Obstructed labor

127
Uterine rupture
Cord prolapse

Triplet 26
26.A Benign tumors of the uterus
Benign Tumors of the uterus

Leiomyoma

Def.: Benign smooth muscle neoplasms originating from myometrium. Symptomaticity depends on
location. Surrounded with a layer of connective tissue. Estrogen- and progesterone sensitive tumors.
Develop during reproductive years, regress during menopause.

Risk factors: prolonged estrogen exposure: early menarche, increased BMI, family history, African
American ethnicity, PCOS; Decreased in increased parity, COCP, smoking.

Classification: acc location and direction of growth: pedunculated serosal, serosal, pedunculated
submucous, submucous, intramural, cervical, intraligamentary. Leiomyomatosis: Extrauterine
smooth muscle tumors.

Symptoms: Most women are asymptomatic.

Bleeding – usually menorrhagia.


Pelvic discomfort, dyspareunia, dysmenorrhea – urinary frequency and pressure sensation
due to distension
Acute pelvic pain – prolapse, necrosis
Infertility
Dg.:

pelvic examination-uterine enlargement – beta-hCG


USG, hysteroscopy, hysterosalpingography

Management:

Observation – if asymptomatic, regardless of size. Annual pelvic examination.


Drug therapy – relive of symptoms in anticipation of menopause and expected regress
o NSAIDS, COCPs, androgens, GnRH agonists
Uterine artery embolization – sy despite medication
Surgical: Hysterectomy, myomectomy,

Endometrial polyps:

Benign lesion originating in endometrium. Likely to be associated with estrogen exposure.

Typically in women > 40. Premalignant in < 1%.

128
Sy.: menorrhagia, metrorrhagia, dysmenorrhea, infertility

Dg.: transvaginal USG, hysteroscopy, curettage reveals histological normal sample; exclude
endometrial malignancy

T: curettage, w/ or w/o hysteroscopy

Adenomyosis

Def.: Uterine enlargement caused by ectopic rest of endometrium inside the myometrium.

Risk factors: 90% in parous women, high estrogen levels

Sy.: asymptomatic in 60%. Menorrhagia, dysmenorrhea

Dg.: elevated CA125, transvaginal USG

T.: NSAIDS for pain relief, definitive T is hysterectomy, endometrial ablation, uterine artery
embolization

26.B Physiological changes during Pregnancy


Haemodynamics, Cardiovascular:

Earliest + most dramatic changes they impove fetal oxygenation and nutrition

Heart is displaced upwards and to the left, enlarges


Increased cardiac output (from 30% to 50% by 8th wk GA)
Increased circulating blood volume and stroke volume. Decreased systemic vascular
resistance (shunting to utero-placental circulation, PG production)
Later: vena cava obstruction decreased venous return decreased CO
Evaluation: increased heart rate. BP decreased until mid pregnancy, then increases to normal values.
Left axis deviaton on ECG.

Respiratory:

Increased oxygen demand.

Elevation of diaphragm Decreased total lung volume, but increased inspiratory capacity
and 30-40% increase in tidal volume.
Increased oxygen consumption by 20%
Changes in acid base balances: compensated respiratory alkalosis.

Hematologic system:

Compensate decreased venous return, enhance oxygen delivery

Increase in plasma volume


Increased erythrocyte volume

129
Increased coagulation factors
increased iron demands (daily intake 60mg!)
Increased WBC
Hypercoaguable state!

Renal:

Increased activity to maintain fluid, solute and AB- balance.


Enlargement and dilatation of kidneys and collecting system
Decreased bladder capacity
Increased renal plasma flow and GFR increased urinary glucose
Increased RAA activity (has no effect for normotensive women)
GI

Due to enlarging uterus+hormones

Displacement of stomach and intestines, enlarged portal vein


Progesterone smooth muscle relaxation lower esophageal sphincter tone, decreased
motility, impaired gall bladder tonus
intake of 200 kcal/day
Nausea and vomiting of pregnancy (4-8 wk GA)
Gastroesophageal reflux
Hemorrhoids
Gingival disease
Constipation
Lab: doubled ALP, serum cholesterol

Endocrine changes

Thyroid: enlarged moderately due to demand. Due to renal clearance iodine


uptake

Pituitary: enlarges. prolactin and ACTH levels. FSH, LH production. Oxytocin release
during first stage of labour and during suckling.

Progesterone: throughout pregnancy, synth. by corpus luteum until 35th day, then by
placenta. causes smooth muscle relaxation and raises body temperature. prevents
preterm labour.

Estrogen: breast and nipple growth, pigmentation

promotes uterine flow, growth, cervical softening

Human placental lactogen: similar to growth hormones, modifies metabolism to support


fetus.

130
Metabolism:

Pregnancy has diabetogenic effect. Increased circulating concentration of lipids. Increased intake and
utilization of protein.

Genital tract and breast:

Uterus: from 70g before to 1.100 g at term, through hypertrophy


Increased vaginal discharge (leucorrhea of pregnancy)
Breast increase in size (rapidly in first 8 wks). Nipples become larger and deeply pigmented

Skin: changes have unclear etiology

Vascular spiders
Palmar erythema
Striae gravidarum
Hyperpigmentation

26.C HIV/AIDS during pregnancy


Antenatal screening introduced in 1999. Opt-out policy.

In the UK in 2010 >98% received assisted reproductive techniques. Pregnancy does not alter the
course of infection, and HIV does not directly influence pregnancy.

High viral load and low CD4 count indicate likelihood of transmission
Women on combination ART with undetectable viral load transmission is 0.1%

Prepregnancy:

Each unprotected intercourse carries 0.03-1% risk of infection.


Mother +ve and partner –ve self-insemination with parters sperm is recommended
Sperm washing or donor insemination if male is +ve and female –ve
Viral load, CD4 counts and AIDS defining illness should be taken into account
HAART regimen continued during pregnancy; protease monotherapy intensified to include
and agent that crosses placenta

Prenatal care:

HAART in pregnancy:
o Commence treatment asap if women require it for their own health
o Lamivudine + Zidovudine + efavirenz
o No routine dose alterations
o Women who do not require HAART for themselves should commence wk 14 if VL >30
K. All women by wk 24

131
Monitoring of HIV +ve pregnant woman
o Minimum one CD4 count at baseline and at delivery
o Women commencing HAART in pregnancy: check VL 2-4 wks after, once every
trimester, at 36 wks, at delivery
o Check LFT (liver function tests) at each visit
o Aim for VL <50c/mL
Delivery

Vaginal delivery: recommended for women with VL<50, at 36 wks on HAART


CS: women on zidovudine monotherapy, irrespective of plasma VL, between 38-39 wks GA if
transmission prevention is the indication
Rupture of Membranes

≥ 34 wks immediate delivery, either CS or induction of labour (acc VL)


PROM < 34 wks: steroids, optimize viral control

Intrapartum zidovudine

If VL < 10k presenting in labour or with ruptured membranes, or admitted for planned CS
Untreated women

Late presentation

HIV diagnostic point of care test (bedside test) if no documentation is present


Reactive test act immediately to prevent transmission to child

Postnatal concerns

Long term follow-up for mother and child


Neonatal post-exposure prophylaxis (PEP) within 4h for 4 weeks
Either zidovudine monotherapy or 3-drug therapy depending on mothers VL
Breastfeeding is not recommended, transmission can occur. In certain countries mothers can
be charged with child endangerment. In low-income countries: breastfeeding if there is no
access to clean water+sanitation.

132
Triplet 27

27.A Malignant tumors of uterus


Endometrial hyperplasia

=premalignant condition that can predispose to endometrial cancer

Overgrowth of endometrial cells.

Etiology: similar to endometrial cancer, see there.

Clinical presentation:

Most commonly in women >40


Irregular menses
Postmenopausal bleeding
Dg.: endometrial curettage – histology to determine degree of hyperplasia and atypia

Management

w/o atypia
o Exclude estrogen secreting tumor, stop estrogen only HRT
o Progestagens
w/ atypia
o High risk to progression to adenoca.
o Recommended is total abd. Hysterectomy (+BSO if >45)

Endometrial cancer

Adenoca. arising from endometrial lining. Incidence rising, linked to “western” lifestyle. Mostly good
prognosis: early detection due to bleeding.

Etiology: unopposed estrogen (=no protective progesterone)

Endogenous
o Adipose tissue
o Estrogen producing tumor
o PCOS, anovulation
Exogenous
o HRT
o Tamoxifen
Risk factors
o Obesity
o Conditions connected to low progesterone: nulliparity, PCOS, anovulatory cycles
o Genetic predisposition
o Breast cancer
Protective: Parity, COCP

133
Presentation:

Post menopausal bleeding


Menstrual disturbances
Vaginal discharge, pyometra

Investigations: Drug/Family/Menstrual/Gyne. History + Bimanual and in specula examination + Blood


works.

Transvaginal USG: measure endometrial thickness.


Staging: CT chest, abd., pelvis; MRI; CXR
Endometrial biopsy: if endometrium ≥ 4mm, or persistent bleeding

FIGO Staging:

I. Tu. limited to uterine body


II. Tu. limited to uterine body and cervix
III. Extension to serosa, peritoneal cavity, and/or lymph nodes
IV. Extension beyond true pelvis and/or involvement of bladder/bowel

Treatment:

Surgery: TAH + BSO + pelvic washings


Adjuvant radiotherapy: brachytherapy
High dose progesterone used for advanced and recurrent disease, palliative.

Other uterine malignancies

Uterine sarcoma

3-5% of uterine ca.

Sy.: abnormal bleeding, pain, pelvic mass.

27.B Sexual transmission diseases


One of the most common gynecologic problems in sexually active women.
Adolescents and young adults are mostly affected.
Partner notification/treatment is vital.
Mode of transmission: oral, vaginal, anal sex
Confidentiality!
Preventable and common cause of infertility!

134
Risk factors

Multiple partners: ≥ 2 in the last year; concurrent partners; recent partner change
No barrier protection
STI in partner
Other STI
Young age (≤ 25yrs)
Involvement in commercial sex industry

History taking:

- Sy: lumps, bumps, ulcers, rash, itching, IMB, PCB, lower abd pain, dyspareunia, change in
discharge
- Past history of STIs, UTIs, last HIV –ve test.
- Sexual partners last 12 months
- Risk behavior, also for blood borne viruses (IV drug use, tattoo)

Physical examination:

- Check: Rashes, lesions, adenopathy, ulcerations – in inguinal region and ext. genitalia
- Bartholin glands, skene ducts, urethra
- Check vagina and cervix for discharge
- Rectum, if patient engages in anal intercourse
- Oral cavity, cervical lymph nodes

Prevention

Education about abstinence until treatment is finished, limiting sexual partners, condoms,
vaccinations. Treatment of partner.

Specific infections:

Chlamydia

trachomatis, obligate intracellular Cervix, urethra, salpinges, uterus, nasopharynx,


epididimys
Clinical: Dysuria, vaginal discharge, IMB, PCB, 70% asymptomatic.
Complications: PID, Fitz-Hugh-Curtis sy, Reiter’s sy, tubal infertility, risk of ectopic pregnancy
Dg.: Endocervical swab + NAAT
T.: Azthromycin
Pregnancy: ass. w/ PROM; neonatal conjunctivitis, neonatal pneumonia.

Gonorrhea

N. gonorrheae: intracellular G+ cervicitis, urethritis, PID, disseminated infection, neonatal


eye infection
Clinical: usually asymptomatic, vaginal discharge, lower abd pain, IMB, PCB

135
Dg.: Endocervical swab + NAAT
Complications: PID, local gland abscess, disseminated: fever, pustular rash, migratory
polyarthralgia, septic arthritis; Tubal infertility, risk of ectopic pregnancy
T.: Ceftriaxone, plus azithromycin
Pregnancy: ass w/ PROM, chorioamnionitis, ophtalmia neonatorum,

Herpes simplex virus

HSV 1 (mostly orolabial), HSV 2 (genital) can cause a variety of disease states, also
depending on immunocompetence.
Clinical: Primary infection usually most severe
- Prodrome: tingling/itching of skin in affected area
- Flu-like illness +/- inguinal LA
- Vluvitis, pain
- Small, characteristic vesicles on vulva
Recurrent: virus invades nerve ganglia and establishes latent infection. Various stimuli can
cause reactivation. Triggers include: Stress, common cold, sexual intercourse, menstruation.
Complication: Meningits, encephalitis, radiculopathy, myelitis, disseminated infection.
Dg.: clinical picture (typical rash), PCR testing of fluid,
T.: No cure. Symptomatic relief with NSAIDS. Oral acyclovir. Condoms/abstinence furing
prodromal/symptomatic phase.
Pregnancy: To mother: meningitis, dissemination; risk for fetus during maternal infection at
delivery: Neonatal herpes, 75% disseminated, mental retardation, death

Syphilis

Treponema pallidum: spirochete.


- Primary syphilis: 10-90 days post infection: Painless genital ulcer (chancre), inguinal LA
- Secondary: within 2 years of infection. Generalized LA. Polymorphic rash affecting palms
and soles. Genital condylomata lata. Anterior uveitis.
- Tertiary: late stage. Neurosyphilis (tabes dorsalis, dementia), Aortitis, Gummata
Dg.: Enzyme immune essay (IgG, IgM), VDRL
T.: Benzyl PCN (can be used in pregnancy), erythromycin,
Pregnancy: Preterm delivery, stillbirth, congenital syphilis, miscarriage.

Trichomonas

Trichomonas vaginalis. Protozoan. Most common STI. Vaginitis, Urethritis, Cervicitis, PID
Clinical: Frothy, greenish, smelling vaginal discharge, vulval itching and soreness, dysuria,
mucopurulent cervical discharge and punctate hemorrhage.
Dg.: wet smear and direct observation, Cx, NAAT
Complications: can facilitate HIV infection,
T.: Metronidazole
Pregnancy: preterm delivery, low birth weight, may be acquired perinatally

136
Human papillomavirus

DNA virus with many subtypes. 6&11 cause genital warts (condylomata acuminata). Subtypes
16, 18, 31, 45 are linked with increased risk for CIN and cervical cancer.
Sy.: majority asymptomatic. Painless lumps in genitoanal area. Perianal warts.
Dg.: clinical appearance. Cervical cytology (Pap-smear), colposcopy
T.: removal of visible wart. High recurrence rate (latency) – cryotherapy, acid, electrosurgery,
excision, podophyllotoxin cream
Pregnancy: tend to grow rapidly in pregnancy, regress after delivery. Babies may develop
laryngeal or genital warts (rare).
Prevention: vaccination. Recommended for girls age 12-13, effective in preventing cervical
cancer even in older women who previously had sexual intercourse.

Human Immunodeficiency Virus and Acquired Immune Deficiency Syndrome

AIDS is the advanced manifestation of HIV, an RNA retrovirus. The virus targets T-Helper cells
(CD4+) and monocytes and causes depletion of these, resulting in immune system
compromise. Two types: HIV-1, HIV-2. HIV-1 most common type in the US. Progression and
manifestation varies between individuals.
Transmission: sexual contact, use of contaminated needles, vertical transmission.
Dg.: Screening: ELISA for HIV Ab. Confirmation with Western blot. Screening for pregnant
women in first trimester greatly reduced mother to child transmission.
Prevention: Use of condoms, safe sex, education about risk behavior
T.: 4 drug classes:
I. Nucleoside/nucleotide reverse transcriptase inhibitors (Zidovudine)
II. Nonnucleoside reverse transcriptase inhibitors
III. Protease inhibitors
IV. Fusion inhibitors

use of multiple drug regimens – highly active antiretroviral therapy (HAART) – aims to decrease
the patient’s viral load (VL). maintenance of immune system, less opportunistic infections. In
some parts of the world HIV turned into a chronic disease and progression to AIDS is becoming
increasingly rare.

Pregnancy: see Question 26/C

27.C Intrauterine Fetal Death – causes, diagnosis, treatment


Def.: Also called stillbirth. Death prior to complete expulsion or extraction from its mother,
irrespective of gestational age with a minimum weight of 500g. Death is indicated by absence of
breathing, heartbeat, pulsation of umbilical cord and movement of voluntary muscles. Most
stillbirths occur in term. Distinction is made to miscarriage (abortion), which is defined as loss before
20th wk GA or of a fetus weighing <500g.

137
Etiology:

Maternal:

- Uterine rupture - Maternal trauma or death


- Prolonged pregnancy (>42 wks) - Poorly controlled DM
- SLE - Antiphospholipid sy
- Infection - Hypertension
- Preeclampsia - Eclampsia
- Hemoglobinopathy - Advanced maternal age
- Rh disease - Inherited thrombophilias

Fetal:

- Multiple gestations - IUGR


- Congenital abmnormality - Genetic abnormality
- Infection - Hydrops

Placental:

- Cord accident (eg prolapse) - Abruption


- PROM - Vasa praevia
- Fetomaternal hemorrhage - Placental insufficiency

Symptoms: often only decreased/absent fetal movement, symptoms of associated conditions (eg
bleeding, pain in uterine rupture,…)

Dg.: USG—visualization of the fetal heart and absence of cardiac activity

Evaluation:

In up to 60% etiology can not be determined


Determining etiology remains important for future pregnancies
Selective workup based on clinical findings (obvious causes need no further testing – eg cord
accidents, anencephaly,…)
Toxicology screening
Autopsy of fetus, alternatively MRI
Examination of the placenta and membranes, including cultures
Consider fetal karyotyping, especially in multiple pregnancy losses
Common maternal studies: DM, Syphilis screening, Thyroid function test, Urine tox.

Treatment:

Offer psychological and spiritual support, emotional needs of family have to be met, some
patients may want to delay induction for hours-days
Dead fetus in utero can cause coagulopathy after 3-4wks
In twin pregnancy: induction may be delayed to allow viable twin to mature
Termination of pregnancy

138
o Induce cervical ripening (Prostaglandins, then IV oxytocin)
o Early demise insertion of laminaria (sticks placed into the cervix to cause
dilatation) followed by evacuation
o Mifepristone + misoprostol most effective
o Pain management: Morphine, hydromorphine, offer epidural

Triplet 28

28.A Medical Definition of Climacterium


Various definitions exist!

Menopause: 1 year after last menstruation (Age ~52).

Post-menopause: years following.

Menopausal transition: usually 4-7-year-long interval, starting from before final menstrual period to
menopause. Typically starts in late 40s.

Climacterium / Climateric / perimenopause: outdated terms describing the transition from late
reproductive years to a non-reproductive state.

Premature menopause (premature ovarian failure): loss of oocytes & secondary amenorrhea < 40y.

Physiologic changes:

Inhibin levels drop FSH starts to rise in late 40s increased follicular response, higher overall
estrogen levels, lower progesterone levels, impaired folliculogenesis, increased anovulation,
depletion of supply of follicles = ovarian failure cessation of steroid hormone release stop of
negative feedback maximum GnRH highest FSH, LH.

139
Endometrial changes: dramatic due to steroid hormone changes

Unopposed estrogen causes proliferative changes abnormal uterine bleeding, endometrial


hyperplasia, endometrial neoplasia
Postmenopause: atrophic

Fertility potential: occasionally ovulatory cycle occur, > 1/3 of pregnancies in women > 40 are
unintended. Contraception can be discontinued at the age of 55.

Changes in central thermoregulation: described as hot flushes and night sweats

Changes in bone metabolism: decrease of estrogen in postmenopausal women bone resorption


and increased incidence of osteopenia and osteoporosis.

Cardiovascular changes: before menopause women have much lower risk for CVD than men in same
age. This is due to higher levels of HDL, an effect of estrogen. decrease in estrogen linked to
increased risk of CVD.

Most common symptoms:

Changes in menstrual pattern: shorter cycles/longer cycles, irregular bleeding.


Vasomotor sy (hot flushes, sleep disturbances)
Psychological/mental: Depression, worse PMS, irritability, poor memory
Sexual dysfunction: vaginal dryness, decreased libido, dyspareunia
Somatic: headache, dizziness, palpitations
Other: urinary incontinence, weight gain, pruritus

Dg.: Documentation of age-appropriate symptoms. FSH or estradiol levels can be performed to


diagnose ovarian failure.

Hormone replacement therapy:

Risks: increased risk of CHD, stroke, thromboembolism, cholecystitis, breast cancer (long term use),
ovarian cancer (long term use)

Benefit: higher bone density, lower rate of colorectal cancer

Indications: very complex and individual – indicated only for vaginal atrophy, vasomotor sy,
osteoporosis prevention/treatment, prescribed in the lowest effective dose.

Available are estrogen, progestin and combination preparations.

28.B Management of normal labor


Evaluation/admittance for labour in case of:

Contractions occur roughly every 5 min for at least 1 hour


Sudden or constant leakage of vaginal fluid suggesting ROM
Vaginal bleeding
Significant decrease in fetal movement

140
Evaluate complications, confirm GA, labs;

Limited physical examination, abd and pelvis, Leopold maneuvers

CTG: contractions and heart rate.

USG: fetal lie and presentation, placental location, amniotic fluid volume, other abnormalities

Vaginal examination: contraindicated in placenta praevia, effacement and dilatation of cervix.

Fetal station: level of fetal presenting part in relation to ischial spines.

Stages of labour:

1st: between onset and full cervical dilatation (10cm); latent phase and active phase.

2nd: From complete cervical dilatation to delivery of infant

3rd: From after delivery of infant to delivery of placenta

General management

Walking (unless contraindicated) may be more comfortable than lying


Position: common dorsal lithotomy, other labouring positions include sitting, crouching, birth
chairs, labour balls, tubs of warm water,…
Fluid management, oral intake in active labour: limited to moderate intake of water; IV saline
if not possible or insufficient
Evaluation of fetal well-being: measurement of FHR and its changes during labour
(intermittent auscultation, Doppler, CTG)
o 1st stage: Every 30 min
o 2nd stage: Continuous fetal monitoring

141
Pain control:
o Epidural block
o Spinal anesthesia
o Combined spinal-epidural
o Local block (eg pudendal block)
st
1 stage management:

Series of pelvic examination: identify cervical dilatation, effacement, station, position of


presenting part, status of membranes.
Discourage pushing at this stage
Artificial ROM is possible
nd
2 stage management:

Pushing added to contractions (Valsalva maneuver)


Episiotomy: facilitates delivery by enlarging vaginal outlet – necessary in instrumental vaginal
delivery; prevention of rupture of perineum.
Modified Ritgen maneuver: support perineum with one hand, other hand placed on vertex.
Delivery of shoulders: gentle downward pressure delivering anterior shoulder
rd
3 stage management

Avoid pulling placenta from uterus (uterine inversion!)


Wait for spontaneous extrusion up to 30 min
Uterotonics (ergometrine, oxytocin), uterine massage
Clamping and cutting the cord
Gentle downward traction on umbilical cord
Manual removal of placenta might be necessary
Suturing of episiotomy

Immediately after delivery

Assess for complete placenta


Uterine palpation to assess tone
Inspection of birth canal
Monitor maternal blood loss, pulse and blood pressure

28.C Coagulation disorders in pregnancy


Implications of coagulation on pregnancy:

Physiology: some clotting factors increase and anticoagulation factors decrease


hypercoaguable state risk for thromboembolism
Pre-existing coagulopathies: affect:
o Course of pregnancy
o Mode of delivery
o Approach to anesthesia

142
Coagulation panel necessary before delivery/CS/regional anesthesia if
o History/physical examination suggestive of coagulation disorder
o Patient is on anticoagulant or antiplatelet drugs
o A disease is present that could alter coagulation.
Recurrent miscarriage syndrome

Parameters: Bleeding time, platelet count, PT, PTT, TT, Fibrinogen, Fibrin degradation products

Clotting factor abnormalities

1) Congenital coagulopathies:
a. Von Willebrand disease in type one: patient improves during pregnancy, not in 2
and 3. Vaginal delivery is safe, higher risk for PPH. Central block not recommended in
type 2+3.
b. Haemophilia A and B women are usually carriers, but clotting factor activity can
be reduced. Desmopressin can be administred prior to labour. Abortions carry a
higher risk for bleeding. Can cause delayed PPH. Regional a. is controversial.
c. Antithrombin deficiency

2) Acquired coagulopathies due to uncontrolled activation of the coagulation system DIC


widespread coagulation depletion of clotting factors
a. Pregnancy induced hypertension
b. Placental Abruption: commonest cause. Bleeding can be concealed.
c. Retained dead fetus: Rare occurrence nowadays, after 3-4 wks after IU death
d. Amniotic fluid embolus: commonly fatal. Respiratory distress, hypotension,
hypoxemia, coma. Cardiopulmonary resuscitation.
e. Liver disease: Hepatitis, acute fatty liver of pregnancy, cholestasis, HELLP sy. T.: Vit K,
FFP
f. Anticoagulants: Aspirin and Heparin

Platelet abnormalities

1) Quantitative abnormalities
a. Gestational thrombocytopenia: Physiologic. Most common cause of
thrombocytopenia, occurs in 5-8% of all pregnant women. The decreased number is
balanced by enhanced activity. No bleeding risk.
b. Idiopathic/immunological thrombocytopenic purpura. T.: steroids, IV Ig,
splenectomy. Platelet counts < 50x109 per L in fetal blood is indication for CS. Avoid
central block.
c. HELLP Syndrome
d. Disseminated intravascular coagulation (DIC)
2) Qualitative disorders

143
Hypercoaguable states:

Certain disorders increase the risk of thromboembolism.

1), 2) and 3): 8 fold increased risk of thromboembolism during pregnancy and puerperium

1) Antithrombin deficiency: also resistance to heparin


2) Protein C deficiency
3) Protein S deficiency
4) Factor V Leiden: most common disorder
5) Lupus anticoagulants, anticardiolipin antibodies (antiphospholipid syndromes APLS)
6) Hyperhomocysteinuria

Prophylaxis: depends on evaluated risk

Antenatal low-dose aspirin


Antenatal LMWH
Postnatal warfarin

Recurrent miscarriage syndrome:

Hypercoaguability and thrombosis placental thrombosis and recurrent miscarriage


syndrome (fetal wastage)
APLS is the most common cause
Hemorrhagic defects are very rare cases of recurrent miscarriage
T.: preconception low-dose aspirin

Triplet 29

29.A Uterine descensus and prolapse


Def.: Pelvic organ prolapse is defined as protrusion and descent of one or more of the following:
anterior vaginal wall, posterior vaginal wall, uterus and cervix, apex of vagina, perineum. The
bladder, urethra, rectum and bowel are also often involved.

Risk factors:

Pregnancy
Vaginal childbirth
Menopause
Chornically increased intraabdominal pressure (COPD, constipation, obesity)
Pelvic floor trauma
Genetic factors (connective tissue disease)
Failure of interactions between pelvic floor muscles, pelvic floor connective tissue, vaginal wall.

Classification: classified anatomically according to site of defect in pelvic viscera

144
Old classification: describes the structures behind the vaginal wall that are suspected to be prolapsed
(cystocele, cystourthrocele, uterine prolapse, rectocele, enterocele).

New classification: describe what is actually seen:

Anterior vaginal wall prolapse


Apical vaginal wall prolapse
Cervical prolapse
Posterior vaginal wall prolapse
Perineal descent
Rectal prolapse

Common symptoms:

Discomfort, heaviness within the pelvis, backache, vaginal pain


Dyspareunia, difficulty inserting tampons
Urinary symptoms: Incontinence, frequency, urgency, feeling of incomplete emptying,
retention
Bowel symptoms: Incontinence, feeling of incomplete emptying, urgency, hard straining,
splinting or digitation (pressing on prolapse or insertion of fingers to initiate defectation)

Staging: POP-Q: several points are taken in reference to the plane of the hymen (=0). Absence of
prolapse: -3, maximal prolapse: +3.

145
Investigations:

Physical examination:

Bimanual examination: palpate for abd mass


Vaginal, in speculum
Sometimes the woman has to be standing to demonstrate prolapse
Asses pelvic floor muscle strength
USG: exclude abd mass
Urodynamics: if urinary incontinence is present
Assess fitness for surgery

Management:

A. Conservative:
- Physiotherapy: Pelvic floor muscle exercises, vaginal cones (weights inserted into vagina)
- Intravaginal devices: Pessaries – for women who decline or are unfit for surgery
B. Surgical:

Obliterative procedures: suturing ant. and post. vaginal walls together. Appropriate for
people who have no future desire for coital activity.

Reconstructive: attempt to restore normal pelvic anatomy. They include:

Hysteropexy: uterus and cervix attached to sacrum with mesh


Sacrospinous ligament fixation: vaginal vault fixed to sacrospinous lig.
Sacrocolpoplexy: vaginal vault attached to sacrum with mesh

29.B Implantation of the embryo and implant failure


Day 4 after fertilization: morula blastocyst enters the uterine cavity
Area of implantation: upper posterior wall of uterine endometrium
Trophoblast cells of blastocyst penetrate uterine mucosa at day 6-7
o Zona hatching: blastocyst prepares
o Apposition: First loose connection
o Adhesion: Stronger attachement
o Invasion: Trophoblast 2 layers:
Cytotrophoblast
Syncytiotrophoblast invades endometrial stroma form chorionic villi
Embryo is fully embedded in endometrium

Causes of implant failure:

2/3: uterine receptivity


Hyperplasia
Polyps
Endometritis

146
Synechiae
Leimyomata
Local dysregulation of cytokines
Immunologic causes
Thrombophilias
Hysteroscopic correction of uterine pathologies, myomectomy

Immunotherapy

Aspirin, LMWH

1/3: embryo
Chromosomal abnormalities of male or female partner, the gametes or the embryo
Failure of zona pellucida rupture
IVF: pre-implantation genetic screening

Multifactorial

Endometriosis: all steps of reproductive process are impaired


Hydrosalpinges
Drugs for ovarian stimulation: may harm endometrium & embryo

Treatment of endometriosis: GnrH agonist

29.C Post-term pregnancy and induced labour


Def.: Postterm pregnancy = exceeding 42 wks from the first day of last menstrual period (LMP) in a
woman with regular 28-day cycles.
Around 10% of pregnancies.
Dates are unreliable in case of:
Uncertainty of LMP
Irregular periods
Recent COCP (combined oral contraceptive pills)

Causes/Risk factors for postterm delivery


Mother:
o Previous prolonged pregnancy
o Maternal obesity
o Nulliparity
o Postterm delivery of mother
o Inaccurate estimation of gestational age (most common “cause”)
Fetus
o Anencephaly
o Fetal adrenal hypoplasia
o Placental sulfatase deficiency

147
Effects
morbidity and mortality rates increase for mother and fetus.
Maternal risks
o Anxiety, psychological morbidity
o Increased intervention
Induction of labour
Operative delivery with > risk of genital trauma
CS
Fetus
o Intrapartum deaths 4 x
o Early neonatal deaths 3 x
o Macrosomia (> 4.500g) Shoulder dystocia, fetal injury – cephalopelvic
disproportion
o Meconium aspiration syndrome (MAS) assisted ventilation
o Oligohydramnios cord compression risk, uteroplacental insufficiency, MAS
o Dysmaturity sy: growth restriction (placental insufficiency), overgrown nails, peeling
skin, little subcutaneous fat. Risk for: MAS, oligohydramnios, hypoglycemia, seizures,
resp. insufficiency
o Fetal distress in labour

Dg.: Review GA! USG is best for determining GA if performed from 6-12 wks GA.

Management after 41 wks

Assess other risk factors and provide counseling (many women refuse induction)
USG: growth and amniotic fluid volume
Daily CTG, fetal movement counting until max. 42 wk
Membrane sweep at 41 wks
USG: growth and amniotic fluid volume

Labour induction: should occur before 42 completed weeks

Consider CS if weight estimation > 4.500g


Cervical ripening
o Intravaginal prostaglandin
o Mechanical: Foley bulb placement
o Misoprostol
o Oxytocin IV

Indications Induction of Labour (IOL)

Uteroplacental insufficiency
Prolonged pregnancy (41-42wks)
IUGR
Oligo- or anhydramnios
Abnormal uterine or umbilical artery Doppler

148
Non-reassuring CTG
PROM
Severe pre-eclampsia
Intrauterine death of fetus
Chorioamnionitis
Medical: eg. severe HTN, uncontrolled DM,

Contraindication:

Active Herpes
Fetal Malpresentation
Non-reassuring fetal surveillance
Prior traumatic delivery
Vaginal bleeding, vasa praevia, placenta praevia, prior CS.

Complications:

All risks associated with postterm delivery


Cord prolapse
Prematurity
Atonic post-partum uterus
Infection with prolonged induction

Risks: All risks associated with postterm delivery.

149
Triplet 30

30.A Prevention and early diagnosis of malignant tumors in gynecology


The female reproductive system contains perhaps a greater variety of malignant tumors than any
other in the human body. Here are the main types:

➢ Carcinomas: most common category of Gyn cancer (epithelial):

- Squamous Cell Carcinoma - epithelial cells protect the body from environmental
dangers. Triggers are mostly environmental - sunlight, tobacco & HPV. Most common
type in the vulva, vagina & cervix. Generally slow to develop, grow & responds well to
therapy.

- Adenocarcinoma (glandular): Most of the endometrial & ovarian cancers & some of the
cervical & vaginal cancers as well.

- Malignant Melanoma (melanocyte): aggressive type of carcinoma that can affect the
vulva & vagina.

➢ Sarcomas - Develop in bone, muscle & connective tissue. Rare in uterus, vagina & vulva.
Usually, less favorable prognosis than carcinomas.

➢ Germ Cell Tumors (ova): rare in ovaries, but have a much higher percentage of benign
ovarian tumors.

➢ Sex Cord-Stroma cells: ovarian, granulosa & theca cells. Rare, ovarian cancer & more often
discovered in Stage 1 than other, ovarian cancers. Granulosa cells release estrogen - the
tumors release too much estrogen (endometrial hyperplasia - abnormal bleeding). Rare in
pre-pubescent girls & the first sign can be early onset of puberty (high estrogen).

➢ Gestational trophoblastic disease - sometimes after the blastocyst implants, the embryo
fails to develop, but the placental cells, continue to grow & may become malignant.

Mixed tumors - "mixed mullerian tumors" of the uterus are also called carcinosarcomas (from
epithelial & connective tissues).

150
Preventive measures:
- Modifiable risk factors such as diet, lifestyle, exercise, drug use etc.
- HPV Vaccine – ideally before first sexual contact.
- Pap test for cervical cancer
- HPV test

Tumor markers

Ovarian cancer, uterine cervical cancer, endometrial cancer, and trophoblastic neoplasms are
gynecologic malignancies for which tumor markers are in clinical use. The following are important
gynecologic tumor markers:

CA-125: values may also be elevated in a number of gynecologic (eg, endometrium, fallopian
tube) and nongynecologic (eg, pancreas, breast, colon, lung) cancers. However, the most
marked elevations (>1500 U/mL) are generally seen with ovarian cancer.

Beta-HCG: Elevations in beta-hCG are found in patients with endometrial cancer,


choriocarcinoma of the uterus, embryonal carcinomas, polyembryomas, mixed cell tumors,
and, less commonly, dysgerminomas. Beta-hCG and alpha-fetoprotein have proved to be
useful markers for ovarian germ cell tumors. In addition, beta-hCG serves as an ideal tumor
marker for monitoring gestational trophoblastic disease and has set the standard with which
other assays must be compared.

Inhibin: An elevated inhibin level in a postmenopausal woman or a premenopausal woman


presenting with amenorrhea and infertility is suggestive of, but not specific for, the presence
of a granulosa cell tumor. Inhibin levels can also be used for tumor surveillance after
treatment to assess for residual or recurrent disease.

Carcinoembryonic antigen: CEA levels are elevated in up to 35% of patients with endometrial
cancer. Most epithelial neoplasms of the ovary also express CEA. The neoplasms include,
with decreasing intensity and frequency, Brenner, endometrioid, clear cell, and serous
tumors.

30.B Secondary amenorrhea


➢ No period for 3 months (if previous regular period) / 6 months (if previous irregular period).

➢ Pathophysiology (based on FSH levels):

Hypogonadotropic: hypothalamic-pituitary dysfunction.

Hypergonadotropic: ovarian follicular failure.

Eugonadotropic: pregnancy, anovulation, uterine abnormalities, outflow tract


obstruction.

151
Etiology:

Anatomic = Pregnancy (most common) or Outflow tract obstruction (upper reproductive tract
adhesions)

Or

Hormonal = Anovulation due to lack of progesterone (No Corpus luteum) or Low estrogen (No
ovarian follicles)

Diagnosis:
- First rule out pregnancy (Test Beta-HCG).
- Then perform Progesterone Challenge Test = IM Medroxyprogesterone acetate (MPA) or
7 days oral MPA pills.

If the patient bleeds within 3-5 days after MPA administration, this is regarded a positive result and
indicates that the diagnosis is Anovulation. In such patients TSH levels should be checked to rule out
Hypothyroidism and also Prolactin levels to rule out Hyperprolactinemia. If these values are normal
then the patient can be managed with Cyclic MPA (7days/1-2months) in order to prevent
endometrial hyperplasia. If the patient would like to conceive consider Clomiphene citrate.

If the patient does not bleed after MPA administration then this is a negative test. This indicates that
amenorrhea is either due to lack of estrogen or outflow tract obstruction. Thus perform Estrogen
Progesterone Challenge Test = Oral estrogen x 21 days, then 7 days MPA. Positive test: bleed within
3-5 days. Thus diagnosis is low levels of estrogen. Work up: check FSH levels = FSH: Ovarian
follicular failure (premature menopause), FSH: H-P axis failure. Can be managed with estrogen
and progestin.

If patient does not bleed it indicates that there is an outflow tract obstruction, which should be
confirmed with hysterosalpigogram, consider Asherman syndrome (interuterine adhesion). Can be
treated via hysteroscopy.

30.C Premature rupture of membranes


Definition: Rupture of membranes before labor onset can be previable (<24w), preterm (24-
35w), or term (36+).

Risk factors: Ascending infection, Membrane defects, Smoking

Clinical presentation: Sudden gush of clear vaginal fluid with olighyrdramnios

Diagnosis: Speculum examination, looking for pooling of amniotic fluid in posterior fornix, then
testing this fluid with Nitrazine paper = (Alkaline PH 5+) dark color means positive test. Also look
for Ferning pattern (sodium chloride crystals) on glass slide microscopy of amniotic fluid.

Differential diagnosis:
Chorioamnionitis: Maternal fever, Uterine tenderness, Confirmed PROM, Absence of Upper

152
respiratory infection or Urinary tract infection

Management of PROM:
Uterine contractions = No tocolysis
Chorioamnionitis = Vaginal cultures (Group B strep), IV antibiotics(Gentamycin/ Clindamycin),
prompt delivery (vaginal or C-sec)
No complications = Triage by gestational age
Before Viability <24 weeks = Send home for bed rest
Preterm Viable 24-35 weeks = Hospitalise, if fever or contractions: deliver, betamethasone
if<32w GA, Cervical culture, 7 days Ampicillin + Erythromycin
At term 36 weeks+ = Prompt delivery

Complications of PROM:

Hazards associated with premature rupture of membranes


If fetus remains in Utero If preterm delivery occurs
Perinatal conditions: Neonatal conditions:

Infection RDS Respiratory distress syn(most


Deformation (olighyrdramnios) common)
Cord compression PDA Patent ductus arteriosus
Pulmonary Hypoplasia (rupture before IVH Intraventricular hemorrhage
20 weeks) NEC Necrotising enterocolitis
ROP Retinopathy of prematurity
Maternal conditions: BPD Bronchopulmonary dysplasia
CP Cerebral Palsy
Infection
DVT
Psychosocial

153
Triplet 31

31.A Urinary incontinence in women


Urinary incontinence (UI) is the loss of bladder control.

IRRITATIVE incontinence:

Etiology: Involuntary detrusor contraction due to Infection (cystitis), stone, tumor, foreign
body
History: Urgency, frequency, dysuria, can occur day or night
Exam: Suprapubic tenderness if infection present, Pelvic exam normal no cystocele, no
rectocele, no prolapse. Neurologic exam normal (S2,3,4 nerve roots)- contraction of anal
sphincter after stimulation of clitoris or labia.
Investigate: Urinalysis (look for WBC, RBC or bacteria), positive urine culture and WBC/
bacteria indicates infection, RBC’s in urine indicates stone, tumor or foreign body.
Management: Antibiotics for infection (based on culture), Cystoscopy if microscopic
hematuria.

Genuine stress incontinence:

Etiology: Loss of bladder support of bladder neck and proximal urethra drop below pelvic
floor (anatomic problem)
History: Small amounts of urine lost in spurts during coughing, sneezing, running etc. Does
not occur at night, unless patient coughs or sneezes!
Exam: Normal neuro exam, pudendal nerve, S2,3,4. May have positive Q-tip test: change in
angle of Q-tip placed within urethra after increase in intra-abdominal pressure (Valsalva
manoeuvre, coughing etc.) indicates loss of bladder support.
Investigate: Normal urinalysis. Normal cystometry, no detrusor contraction present
Management: Medical: Kegels exercises, HRT (estrogen can strengthen tone of pelvic
structures) Surgical if severe: urethropexy, elevations of urethrovesicle junction above pevic
floor (Burch procedure or MMK).

Motor URGE hypertonic incontinence:

History: Large amounts of urine lost (day or night) without warning. Sense of urgency, always
searching for the nearest WC.
Investigate: Normal urinalysis, Detrusor contraction present
Management: Anticholinergics (oxybutynin chloride)

Overflow hypotonic incontinence:

History: Small amounts of urine lost intermittently day and night


Exam: Neuro exam abnormal (loss of S2,3,4 function), can be due to drugs (analgesics in
post-partum patients) or MS
Investigate: Cystometry = Increased post voidal residual urine volume. Loss of Detrusor
contraction ability
Management: Cholinergics (bethanechol), intermittent self-catheter if patient has MS

154
Bypass fistula incontinence:

History: Continual urine loss, day & night, often associated with radical pelvic surgery
(hysterectomy or anterior colon resection) or radiation therapy
Investigate: Intravenous pyelogram: shows dye leaking out from fistula.
Management: Surgical fistula repair, transvaginal or transabdominal

Summary of Urinary Incontinence in females


QUESTIONS Dif. DIAGNOSIS MANAGEMENT
1) Does patient keep Document urine loss, amount, time, activity
voiding diary?
2) UA: RBC,WBC, Antibiotics for infection
Bacteria, Culture IRRITATIVE Cystoscopy for foreign body,
abnormal? stone or tumor
3) Not at night? Kegels, Estrogen,
STRESS
Urethropexy
4) Involuntary bladder
URGE Anticholinergics, NSAIDs
contractions?
5) Increased residual Cholinergics,
OVERFLOW
volume? Alpha-adren blockers
6) Continual leakage? BYPASS Surgery

31.B Differential diagnosis of ectopic pregnancy


Definition: The term ectopic pregnancy refers to a gestation in which the fertilized ovum implants on
any tissue other than the endometrial membrane lining the uterine cavity.

Most commonly occurs in ampulla of fallopian tube.

Risk factors:

Previous ectopic pregnancy (strongest risk factor)


Pelvic inflammatory disease (PID)
Intrauterine devices (IUD)
Tubal damage due to endometriosis or surgery

155
Presentation:

Period of amenorrhea
Unilateral lower abdominal or pelvic pain
Vaginal bleeding (usually 6-8 weeks after LMP)
Normal symptoms of pregnancy are often present: breast tenderness, frequent urination and
nausea
If ectopic pregnancy is ruptured, may present with abnormal vital signs (fever, hypotensive)
or signs of peritoneal irritation (rebound tenderness, abdominal guarding)

Differential diagnosis:

Abortion (anembryonic gestation, threatened abortion, incomplete abortion, complete


abortion, missed abortion.)
Acute appendicitis
Adnexal torsion
Corpus luteum cyst rupture
Diverticulitis
Endometriosis
Gastroenteritis
PID
UTI

Diagnostic tests:

Beta-hCG to confirm pregnancy (1500-200 IU/L) (levels double every 48 hours)


Ultrasound, transvaginal in order to locate implantation site
In unstable patients, laparoscopy may be required to visualise and remove EP

Treatment:

If the patient has a ruptured ectopic pregnancy and has haemodynamic stability, then
laparoscopic surgery can be performed. (Salpingostomy or Salpingectomy if tube is not able
to be saved)
In an unstable patient, IV fluids, blood products, dopamine, may be administered prior to
laparoscopic surgery.
If it has not yet ruptured then either medical or surgical treatment can be given. Optimal
candidates for Methotrexate medical treatment include hemodynamically stable, willing and
able to comply with post treatment follow up, beta-hCG <5000.

In the case of methotrexate therapy, must obtain baseline labs for:

CBC (rule out anemia)


LFTs (rule out underlying liver diease-methotrexate can cause hepatotoxicity)
Kidney function (methotrexate primarily excreted through kidney’s)

b-hCG (checked before and after administration, must drop by more than 15 percent 7 days later, if
not give 2nd dose and check again 7 days later, if drop is successful after first or second dose, observe

156
for side effects and no other treatment necessary, however if high levels of hCG persist, then
consider surgery.

Other exclusion criteria for Methotrexate:

Signs of impending or ongoing ectopic mass rupture


Immunodeficiency, active pulmonary disease or peptic ulcer disease
Coexistent viable intrauterine pregnancy
Breastfeeding
Gestational sac greater than 3.5cm or positive fetal heart activity
No timely access to medical institution

31.C Genetic birth defects


Birth defects, also known as congenital disorders are conditions existing at or before birth regardless
of cause. Birth defects can be minor or severe, and they can affect appearance, organ function, and
physical or mental development. They may be the result of genetic or environmental factors. This
includes errors of morphogenesis, infection, epigenetic modifications on a parental germline, or a
chromosomal abnormality.

Cytogenetic disorders:

Turner syndrome (45,X) Affects 1-10,000 in USA. 98 percent of Turner syndrome conceptions
are spontaneously aborted. Loss of the paternal X chromosome. Gonadal dysgenesis leads to
absence of secondary sexual characteristics, very short stature. Increased nuchal skin fold
thickness & cystic hygroma (often lethal) on USG, Increased lymphedema and webbed neck
(residual of cystic hygroma), coarctation of aorta.
Klinefelter syndrome (47,XXY) Affects1-2000. Tall stature, central obesity, small genitals
(micro penis & small testicles), Infertile, long arm span.
Down syndrome (Trisomy 21) Affects 1-800 (most common). Endocardial cushion defects
(atrioventricular canal defect 50%), ASD, VSD, duodenal atresia (double bubble appearance
on USG), Brushfield spots on iris. Simian crease on palms in 50%.
Edward dyndrome (Trisomy 18) Affects 1-8000. Rockerbottom feet, clenched fist on USG
Patau syndrome (Trisomy 13) Affects 1-6000. Profound mental retardation, poor survival.
Lethal condition. Major anatomical abnormalities, cleft lip & palate, cyclops & proboscis
(single eye & protruding appendage), holoprosencephaly in 40%.

Mendelian disorders:

Autosomal DOMINANT:
o Most common, Present in every generation of family tree, No carriers, Can affect
both genders
o Examples: (Anatomic disorders): Achondroplasia, Marfans, PCK

Autosomal RECESSIVE:
o Can skip generations in family tree, can have carriers, and can be both male and
female, higher incidence in consanguinity.

157
o Examples: (Metabolic/ enzyme disorders) CF(most common in Caucasians), Sickle cell
anemia, Tay sachs, PKU, congenital adrenal hyperplasia

X-linked Recessive:
o Can skip generations, can have carriers (only females), and only affects males.
o Examples: Hemophilia, color blindness

X-linked Dominant:
o Much rarer than recessive, every generation is affected; both males and females can
be affected, usually lethal.
o Example: Hypophosphatemic rickets

Multi-factorial inheritance:

Causes more birth defects than cytogenetic or Mendelian disorders.


Also known as polygenic: more than one gene affected causing similar phenotype.
Accounts for 70% of all birth defects.
Mechanism: Interaction of genes with environment (combination of genetic predisposing
factors and environment) (recurrence rate of 2-3%)

Examples:

Neural tube defects (spinna bifida, meningocele, meningomyelocele, anencephaly), usually develop
around 24-28 days post conception, can cause hydrocephalus. Can be prevented with adequate folic
acid intake during gestation (4mg/d in high risk, 0.4mg/day in low risk) High risk = positive family
history, previous child with NTD, diabetes, taking anti-convulsants.

Congenital heart disease, Cleft lip/palate

158
Triplet 32

32.A Cervical cancer


Cancer arising from cervix: one of the most common cancers in women worldwide.

Cervical intra-epithelial neoplasia (CIN) is the potential precursor & can invade / spread to other
parts of the body.

➢ Successfully treated when found early (Pap test).


➢ Mostly caused by human papilloma virus (HPV).
➢ A PAP test can find changes, before becoming cancer.
➢ Vaccination & Early treatment may prevent cervical cancer.

Symptoms:

Abnormal cervical cell changes rarely cause symptoms. Symptomatic when transformed to cancer:

Abnormal vaginal Bleeding (between menstrual periods, after sex / menopause).


Pain in the lower abdomen / pelvis.
Pain during sex.
Abnormal vaginal discharge.

Risk Factors:

HPV.
Multiple partners (risk of HPV).
Smoking (promoter).
Immunocompromise: HIV, immunosuppressive agents, major disease.
Use of the combined oral contraceptive pill (COCP).
Multiple pregnancies.

Normal & abnormal physiology of transformation zone:

➢ Endocervix (thin secretory glandular epithelium).

➢ Ectocervix (stronger stratified squamous epithelium).

➢ The two meet at the squamocolumar junction.

➢ Under the influence of estrogen the glandular epithelium is pushed out onto the ectocervix &
in response to low pH undergoes physiological squamous metaplasia (the transformation
zone (TZ)).

159
➢ The TZ is usually ectocervical in women of reproductive age, but tends to become
endocervical in post-menopausal women.

As an area of high mitotic activity the TZ is vulnerable to HPV-driven neoplastic changes.

~9 years from HPV infection to cancer (preventable disease).

Cancer subtypes:

squamous cell carcinoma (about 80-85%


adenocarcinoma (about 15% of cervical cancers in the UK
adenosquamous carcinoma
small cell carcinoma
neuroendocrine tumour
glassy cell carcinoma
villoglandular adenocarcinoma

160
Staging:

Cervical cancer is staged by the International Federation of Gynecology and Obstetrics (FIGO) staging
system, which is based on clinical examination, rather than surgical findings. It allows only the
following diagnostic tests to be used in determining the stage: palpation, inspection, colposcopy,
endocervical curettage, hysteroscopy, cystoscopy, proctoscopy, intravenous urography, and X-ray
examination of the lungs and skeleton, and cervical conization.

Prevention:

I. Screening:

Cervical cytology:

Due to problems with sensitivity & specificity, any abnormal cytology is further assessed by
colposcopy.

Colposcopy:
Involves the magnified (6–40x) visualization of the transformation zone
after application of 5% acetic acid (preferentially taken up by neoplastic

161
cells) or Lugol’s iodine (not taken up by glycogen-defi cient neoplastic
cells). Upon identification of colposcopic abnormalities either:
Directed punch biopsy to gain histological confirmation; or
Definitive treatment .

Referral criteria for colposcopy


o Any smear showing borderline nuclear changes or mild dyskaryosis with high risk HPV.
o Any smear showing moderate or severe dyskaryosis.
o Any smear suggestive of malignancy.
o Any smear suggestive of glandular abnormality.
o Three consecutive inadequate smears.
o Keratinizing cells (?underlying CIN).
o Post-coital bleeding.
o Abnormal-looking cervix.

II. Barrier protection


III. Vaccination: HPV vaccines.
IV. Nutrition : Vitamin A is associated with a lower risk, as are vitamin B12, vitamin C, vitamin E, and
beta-carotene.

Treatment:

CIN can be managed conservatively, by excision, by destruction (chemotherapy/Radiotherapy) , or


rarely by hysterectomy. Management depends upon the grade of CIN and patient preference, but
excision is the preferred treatment modality. This is usually by LLETZ (large loop excision of the
transformation zone).

Complications of LLETZ:
A) Short term
o Haemorrhage.
o Infection.
o Vaso-vagal reaction.
o Anxiety (disproportionately high in colposcopy clinic attenders).
B) Long term
o Cervical stenosis (dysmenorrhoea and/or difficulty in follow-up).
o Cervical incompetence and premature delivery (evidence suggests absolute risk of adverse
effect on neonatal outcome is very low).

32.B Cardiotocograph plus ST analysis of fetal electrocardiogram and pulse


oximetry
Technical means of recording the fetal hear rate and the uterine contractions during pregnancy,
typically in the third trimester. The machine used to perform the monitoring is called
a cardiotocograph, more commonly known as an electronic fetal monitor (EFM).

162
Abnormalities in baseline rate:

A bradycardia is a baseline FHR of less than 110 beats/min.


o 100–110 bpm not considered to be associated with fetal compromise if the baseline
variability is normal & accelerations are present.

o < 100 bpm is suspicious of hypoxia / other pathology.

o Maternal factors: Beta-blockers / local anesthetics, may lower fetal HR.


.
A tachycardia is a baseline FHR >160 beats/min
o Associated with maternal pyrexia & tachycardia, prematurity & fetal acidosis.

o Maternal: Beta-agonists / parasympatholytics, may raise fetal HR.

o > 180 bpm is suspicious of an underlying pathology.

Fetuses with either baseline bradycardia or tachycardia are not necessarily in trouble, thus an
abnormal fetal heart rate reading does not correlate a sick baby.
Corticosteroids administered to mother may reduce FHR variability.

The most common mechanism of FHR accelerations are fetal movements (sympathetic stimulation).

Decelerations

Early decelerations:
The peak of the deceleration
coincides with the peak of the
contraction; it is related to fetal head
compression and, therefore, should
only be seen in active second stage
of labour, more commonly after
rupture of membranes. Normal and
not troublesome.

Variable decelerations:
Have variable pattern in timing, size, and shape and
are associated with cord compression. If severe VDs

163
can be troublesome (they last 60 seconds or more and drop 60 bpm or more).

Late decelerations:
Have at least a 15s time lag between
the peak of the contraction and the
nadir of the deceleration. They may be
suggestive of acidosis, intrauterine
growth restriction IUGR, bleeding,
infection, prolonged pregnancy, or
severe pre-eclampsia. Always
troublesome (ureteroplacental
insufficiency).

Variability:
Normal rapid rise and fall of fetal heart
rate which is a reflection of the interplay
between sympathetic and
parasympathetic nervous system.
Decreased variability is a bad sign.

Other abnormalities
Sinusoidal pattern: a rare undulating pattern (sine wave) with little, or no, variability.
Can indicate significant fetal anaemia.
A sinusoidal pattern should always be taken seriously. Blood group antibodies, Kleihauer
test, and a scan for middle cerebral artery velocity to detect fetal anaemia may be
indicated.

The STAN system: (ST Analysis, Neoventa Sweden): is a system for fetal surveillance that displays the
FHR & information resulting from the computerized analysis of ST interval of the fetal ECG. This
system is based on ECG changes determined by the myocardial adaptation to oxygen deficiency.

➢ If the amount of oxygen available is insufficient, a fetus with intact defense mechanisms
reacts by releasing stress hormones & switches to anaerobic metabolism.

164
➢ Oxygen deficiency & anaerobic metabolism in the heart muscle cells will produce changes in
the ST interval.

➢ Normal fetal ECG ST interval: horizontal / upward sloping ST segment & T-wave with
constant & stable amplitude.

➢ A normal ST interval usually indicates the fetus is capable of coping with the stress of labor.

➢ During oxygen deficiency / labor, hypoxia initiate anaerobic metabolism, releases


potassium ions, which increase the T-wave height.

➢ ST depression & negative T-waves indicate compromised cardiac performance.

Fetal pulse oximetry (saturation level of the blood with O2): A probe attached to the baby's head
inside the birth canal.

FPO has not met its expectation, because of its inability to reduce overall C sections.

32.C Multiple pregnancy


A multiple birth occurs when more than one fetus result from a single pregnancy. The preceding
pregnancy is called a multiple pregnancy. Different names for multiple births are used, depending on
the number of offspring. Common multiples are two and three, known as twins and triplets,
respectively.

Multiple birth siblings are either:

Monozygotic [single fertilized egg (zygote) splitting into two or more embryos (identical),
each carrying the same genetic material] Siblings are commonly called identical (same sex,
blood type and look).
Polyzygotic – multiples instead result from multiple ova being ripened and released in the
same menstrual cycle by a woman's ovaries, which are then fertilized to grow into multiples
no more genetically alike than ordinary full siblings, sharing 50% of their genetic material.

Multiple Gestation according to Zygocity, Chorionicity, and Amniocity and associated risks

165
Antenatal care:

All multiple pregnancies are by definition ‘high risk’ and the care should be consultant led.
Establish chorionicity—most accurately diagnosed in 1st trimester so an early USG should be
considered
Routine use of iron and folate supplements should be considered.
A detailed anomaly scan should be undertaken.
Advise aspirin 75microgram if additional risk factors for pre-eclampsia.
Serial growth scans at 28, 32, and 36wks.
More frequent antenatal checks because of increased risk of pre-eclampsia.
Offer delivery at 37–38wks: induction or lower segment Caesarean section (LSCS).

Special Maternal risks:

The risks of pregnancy appear to be heightened with twins compared


with singletons, leaving mothers at increased risk of:
Hyperemesis gravidarum.
Anaemia.
Pre-eclampsia (5 ×greater risk with twins than singletons).
Gestational diabetes.
Polyhydramnios.
Placenta praevia.
Antepartum and post-partum haemorrhage.
Operative delivery

166
Fetal risks associated with multiple pregnancy:

All fetal risks increased with monochronic twins.


Increases Risk of miscarriage: especially with MC twins.
Congenital abnormalities more common only in MC twins including:
o neural tube defects
o cardiac abnormalities
o gastrointestinal atresia.
Intrauterine growth restriction (IUGR): up to 25% of twins.
Preterm labour: main cause of perinatal morbidity and mortality:
o 40% twins deliver before 37wks
o 10% twins deliver before 32wks.
Increased Perinatal mortality:
o intrauterine death .
Increased Risk of disability (mainly, but not entirely, due to prematurity and low birth
weight).
Increases Incidence of cerebral palsy (CP):
Vanishing twin syndrome: one twin apparently being reabsorbed at an early gestation (1st
trimester).
Twin-to-twin transfusion syndrome (TTTS). This affects about 5–25% of MC twin
pregnancies and left untreated has an 80% mortality rate. It is caused by aberrant vascular
anastomoses within the placenta, which redistribute the fetal blood.

Triplet 33

33.A Precancerous lesions of the vulva, vagina and cervix uteri


Vulvar intraepithelial neoplasia

Potentially precancerous condition that can be either white, dark or red. Excess keratin production
leads to a white appearance, whereas excess melanin production leads to dark lesions. It Currently
VIN is classified according to its severity: I/ mild dysplasia; II/ moderate dysplasia; III/ severe dysplasia
(carcinoma in situ). Progression to cancer is uncommon in younger women, but is more likely in older
patients.

Etiology: Treatment:
- HPV - Topical 5-fluoroacil (chemotherapy)
- HSV-2 - Topical estrogen therapy
- Smoking - Laser vaporization
- Immunosuppresion - Local excision
- Chronic vulvar irritation - Skinning vulvectomy
- Lichen Sclerosus - Intracavitary radiation therapy

Prevention:
- HPV vaccination

167
Diagnosis:
- May be asymptomatic
- Local symptoms
o Itching
o Burning
o Pain
- Biopsy

Vagina

Vaginal intraepithelial neoplasia

Most common precancerous vaginal condition that is characterized by dysplastic changes. It is a rare
condition which usually is asymptomatic. Changes occur in the epithelium and often occurs together
with cervical intraepithelial neoplasia (CIN).

Classification:

VAIN I – mild dysplasia


o Mild epithelial changes
VAIN II – moderate dysplasia
o Moderate epithelial changes
VAIN III – severe dysplasia or carcinoma in situ
o Severe epithelial changes and considered a premalignant lesion
o Usually occurs when CIN III (severe dysplasia of the cervix) is present

Etiology: Diagnosis:
- HPV infection – 16 & 18 types - Pap test – abnormal cells
- History of CIN or cervical cancer - Pelvic examination
- - Colposcopy
Treatment: - Laser surgery
- Mild form – resolves spontaneously - Intracavitary radiation therapy
- Topical estrogen therapy - Surgery
- Topical 5-fluorouracil o Partial or total vaginectomy
- Surgical excision o Vaginal reconstruction

Cervix uteri

Cervical intraepithelial neoplasia:

Premalignant condition of the uterine cervix. The ectocervix (surface visible in vaginal speculum
exam) is covered in squamous epithelium, and endocervix is covered with glandular epithelium.

Classification:

CIN 1 – low grade, mild atypical cellular changes in the lower third of the epithelium
CIN 2 – mid grade, moderately atypical cellular changes confined to the basal two-thirds of
epithelium with preservation of epithelial maturation
CIN 2 – high grade, severe atypical epithelial changes, more than 2/3 of epithelium involved
and includes full-thickness lesions (carcinoma in situ)

168
Etiology and RFs: Treatment:
- HPV infection - Ablation and excision
- AIDS - Cryocautery
- Smoking - Laser cautery
- HSV - Loop electrical excision procedure
- Chlamydia (LEEP)
- Oral contraceptives - Cervical conisation

Prognosis: Most of the CIN cases regress spontaneously within 1 year (70%) and 90% within 2 years.

33.B Screening for congenital birth defects in pregnancy


Birth defects can develop as a result of gene mutations, chromosomal changes, organ & body
chemistry related problems

Genetic disorders – Down syndrome, trisomy 18, Fragile X syndrome, PKU


Familial inherited diseases – Tay-Sachs, sickle cell anaemia, cystic fibrosis
Anatomical defects – hypo/agenesis of the limbs, spina bifida, cleft lip or palate, Congenital
Heart Defects

Since pregnancy is a constantly changing (fetal) process, depending on the timing (trimester) we can
run screening tests targeting more or less specific possible diseases/defects so proper treatment
steps can be taken in the future.

Note: All screening tests are not diagnostic! Those only show that there might be a problem and for
diagnosis we need to run diagnostic tests to either confirm or rule out possible disease/defect.

First Trimester Screening Tests

Fetal Nuchal Translucency (USG)

Examination of the area at the back of the fetal neck for increased fluid or thickening. Thicker than
normal (up to 2.5 mm) might indicate Down syndrome, trisomy 18 or CHDs. Usually done between
11 and 14 weeks. In order to confirm or rule out suspected problem one should run Chorionic Villus
Sampling or Amniocentesis.

First-trimester blood test

This test is done usually done in 14-16 weeks evaluates levels of:

- Beta human chorionic gonadotropin (beta-HCG)


- Pregnancy associated plasma protein A (PAPP-A)

Abnormalities in both indicate increased risk of chromosome abnormalities as well as molar


pregnancy.

169
Cell free fetal DNA

Relatively new test that is used to assess the risk of chromosomal disorders (Down syndrome,
trisomy 18, Patau syndrome (13)) and is usually taken 10th week or later and/or in high-risk group
women. Negative result means low probability of trisomy. If positive, chorionic villus sampling
and/or amniocentesis is needed to confirm diagnosis.

Second Trimester Screening Tests

Alpha-fetoprotein screening (AFP)

AFP is a protein produced by fetal liver and present in amniotic fluid that crosses the placenta to
mothers blood. Abnormal levels (>10-150 ng/mL) may indicate:

Open neural tube defects (for example: spina bifida, anencephaly)


Down syndrome (and other trisomies)
Defects in the abdominal wall of the fetus
Twins

Triple or quadruple blood test

1. Beta human chorionic gonadotropin (beta-hCG)


2. Estrogen (estriol) – decreased in trisomy 21, 18, 13
3. Hormone inhibin A
4. Alpha-fetoprotein (AFP)

Show chance of chromosomal abnormalities (Down syndrome etc.), spina bifida, anencephaly etc.

Ultrasound

Done usually around 18-20 week of pregnancy. Used to evaluate the size of the baby, anatomical
abnormalities and other birth defects.

Diagnostic tests

Amniocentesis

Procedure used in which amniotic fluid is sampled and tested and has a diagnostic value in
evaluation of problems in the developing baby, such as:

Birth defects
Neural tube defects (anencephaly and spina bifida) by AFP levels
Genetic problems
Infection
Lung development (surfactant/albumin ratio if less than 2:1 than its deficient)
Rh incompability
Decompression of polyhydramnios

170
Chorionic Villus Sampling

Used to test for chromosomal abnormalities as well as other genetic conditions such as Cystic
Fibrosis. Indications include: abnormal first trimester screen results, increased nuchal translucency,
chromosomal abnormalities, history of familial genetic disorders, advanced age of mother.

33.C Bleeding in pregnancy – causes, diagnosis, therapy


Bleeding during pregnancy is common, especially during the first trimester and usually it should not
cause alarm, but it may be a sign of more serious problem, thus it is good to check the cause. Among
the most common causes we can find:

Early pregnancy – 1st Trimester Late pregnancy – 2nd and 3rd Trimester
1. Abortion/miscarriage 1. Placenta Praevia
2. Ectopic pregnancy 2. Placental abruption
3. Hydatidiform mole (molar pregnancy) 3. Vasa Praevia
4. Placenta Accreta
other, less common: 5. Premature labour
4. Infection - STDs
5. Cervical changes due to intercourse or a other, less common:
Pap test 6. Injury to the cervix or vagina
7. Polyps
8. Cancer

Abortion/Miscarriage

Def: 1/ any fetal loss from conception until the time


of fetal viability at 24 weeks gestation

or 2/ Expulsion of a fetus or an embryo weighing 500 mg or less

Classification:

1. Spontaneous – occurs without medical or mechanical means


2. Induced abortion

Pathophysiology:

Bleeding/haemorrhage into the decidua placentalis


Necrotic changes in the tissue adjacent to the bleeding
Detachment of the conceptus
The above will stimulate uterine contractions
Expulsion

171
Types of abortion:

1. Threatened abortion (see def.)


2. Inevitable abortion
3. Incomplete abortion
4. Complete abortion
5. Missed abortion
6. Septic abortion – any type of abortion complicated by infection
7. Recurrent abortion – 3 or more successive spontaneous abortions

Clinical features & management:

Threatened abortion Inevitable abortion


o Short period of amenorrhea o Short period of amenorrhea
o Mild bleeding (spotting) o Heavy bleeding accompanied with clots
o Mild pain (may lead to shock)
o P.V.: closed cervical os. o Severe lower abdominal pain
o Pregnancy test (hCG): + o P.V.: opened cervical os.
o USG: viable intra uterine fetus o Pregnancy test (hCG): +
o Management: o USG: non-viable fetus and blood inside the
Reassurance uterus
Rest o Management:
Repeated USG Blood transfusion – fluid maintenance
Ergometrine/Synthosinone – induces
the uterus to contract
Evacuation of the uterus
(medical/surgical)

Incomplete abortion Missed abortion


Partial expulsion of products Gradual disappearance of pregnancy
Bleeding and colic pain Brownish vaginal discharge
P.V.: opened cervix, retained products Milk secretion
may be felt through it Pregnancy test: negative
USG: retained products of conception USG: absent fetal heart pulsations
Management: Management:
- Same as inevitable abortion - Wait 4 weeks for spontaneous
expulsion
- Evacuate if:
Spontaneous expulsion does not
occur
Infection
DIC
Manage according to size of uterus:
- < 12 weeks: dilatation and
evacuation
- > 12 weeks: try oxytocin or PGs

172
Ectopic pregnancy:

Def: implantation outside uterine cavity

Most common in fallopian tube (98%), with incidence 1/100 of all pregnancies. Tubal implantation
incidence according to the place: Ampulla (64%), Isthmus (65%), Infundibulum (9%), Interstitial (2%).
Risk factors include: Chlamydia infection, previous ectopic pregnancy, tubal ligation etc.

Cl. Features:

- Amenorrhea, abdominal pain


- Adnexal or cervical motion tenderness
- Constant pain – may be referred to the shoulder if blood tracks to the diaphragm and
stimulate the phrenic nerve
- Vaginal bleeding – dark, scanty, irregular

Diagnosis: Treatment:
- History of last menstrual period - If haemorrhage and shock present:
- Think of tubal pregnancy in case of o Restore blood volume
abdominal pain until proven (transfusion)
otherwise o Laparotomy
- USG to exclude intrauterine - Early diagnosis
pregnancy o Laparoscopic removal
- Laparoscopy: identify unruptured
tubal pregnancy and exclude
salpingitis

Placenta Praevia:

Placental plantation that overlies or is within 2 cm of the internal cervical os. Can be: complete,
partial, marginal, low lying.

173
Diagnosis: Causes: Abnormal vascularisation of the
- Hx + physical exam. endometrium due to scarring or atrophy; age
- Know placental placement before digital >35; endometritis.
examination could cause life-threatening
haemorrhage
- USG Management: Caesarean section (decision not
- Sterile speculum exam – evaluation of made until 36 weeks because the placenta will
ruptured membrane migrate

Placental abruption:

Def: separation of the placenta from the uterine wall before delivery (marginal/partial/complete).
Classified according to severity 0-3 (asymptomatic to severe). Present with painful vaginal
bleeding, abdominal or back pain and uterine tenderness, abnormal uterine contractions,
fetal death.

Diagnosis: Management:
- Hx + phys. Exam. - Mature fetus homodynamic stabilization
- USG will show abruption in 50% of cases - Premature fetus close monitoring as long
- Clinical picture once other causes excluded as no fetal/maternal distress
- Rule out coagulopathy (PT/PTT, Platelets,
fibrinogen, fibrin)
Placenta Accreta:

All or part of the placenta attaches abnormally to the myometrium:

IV. Accreta – chorionic villi attach to the myometrium


V. Increta – chorionic villi invade into the myometrium
VI. Percreta – chorionic villi invade through the
myometrium

Diagnosis is done mainly by USG, but MRI can be helpful if USG is


inconclusive. Treatment include planned caesarean section and abdominal hysterectomy.

Vasa Praevia:

Complication in which fetal blood vessels cross or run near the external orifice of the uterus, usually
from velamentous insertion of the umbilical cord. Bleeding is fetal and result in death of the fetus.

Diagnosis: (classic triad)

- Membrane rupture
- Painless vaginal bleeding
- Fetal bradycardia

Treatment is an emergency caesarean delivery.

174
Triplet 34

34.A Vaginal and vulvar cancer


Squamous cell carcinoma (most common 90%)
Melanoma (2nd most common 5%)

Diagnosis: All vulvar lesions of uncertain etiology should be biopsied. Consider pre-invasive or
invasive vulva carcinoma if pruritic vulvar lesion present.

Surgical staging:

Stage 0: Carcinoma in situ (full-thickness involvement, BM intact)


Stage 1: Size ≤ 2cm, no nodes
Stage 2: Size ˃2cm, no nodes
Stage 3: Any lower urethral, vaginal or anal involvement with unilateral nodes
Stage 4: Any upper urethral, bladder or rectal involvement with bilateral nodes

Management of vulvar carcinoma:

1. Radical vulvectomy (remove entire vulva) – can


result in sexual dysfunction
2. Modified radical vulvectomy (wide local excision) –
less sexual morbidity
3. Lymphadenectomy (Inguinal node dissection) –
may cause lower extremity edema

34.B The Newborn Infant – postpartum treatment and examination


Immediately following delivery, the newborn should be assessed to decide whether resuscitation is
necessary. Four characteristics define a newborn that requires no additional resuscitation:

1. A full-term infant
2. Clear amniotic fluid with no evidence of meconium and infection
3. Spontaneous breathing and crying
4. Good muscle tone

Routine care:
Warming: after being thoroughly dried, maintain appropriate body temp, warm blankets,
skin to skin contact with mother etc.

Premature newborns may require warming pads.

Nose & oropharynx suction to ensure open airway.

175
Umbilical cord: after umbilical cord is clamped & cut, it is left exposed to air to facilitate
drying & separation.

Vital signs: temperature, heart & respiratory rate, core & peripheral color, level of alertness,
tone & activity should be monitored at delivery & every 30 mins thereafter until stable for at
least 2 hours.

Apgar score (Appearance, Pulse, Grimace, Activity, Respiration) 0 - 10 points:


- Prophylactic, antibiotic ointment (erythromycin / tetracycline) to both eyes in order to
prevent gonococcal opthalmia neonatorum.

- Every newborn should also receive a parenteral dose of vitamin K to prevent vitamin K
dependent, hemorrhagic disease.

- Jaundice, which occurs in most newborns, is usually benign, but because of the potential
toxicity of bilirubin, all newborns should be assessed prior to hospital discharge to identify
those at high risk for severe hyperbilirubinemia.

176
- Late preterm (35-37w) infants are at higher risk for hyperbilirubinemia. Elevation of direct
serum bilirubin concentration always requires further investigation & possible intervention
(phototherapy / exchange transfusion).

Worrying signs that warrant close observation: temp instability, refusal of feeding, unusual skin
coloration, abnormal cardiac or respiratory activity, abdominal distention, bilious vomiting,
excessive lethargy or sleeping, delayed abnormal stools, and delayed voiding.

Initial care of the ill newborn:

Resuscitation may be necessary in up to 10% of all deliveries, 1% of these require major


resuscitative efforts. If the neonate is having difficulty breathing, ventilation chest
compression, and epinephrine should be instituted. If infant does not respond to
epinephrine, consider hypovolemic shock (especially if there is evidence of blood loss), in this
case give normal saline at 10mL/kg.
A neonate who is apnoeic or gasping and has a HR of <100bpm requires positive pressure
ventilation.
During the resuscitation process, the metabolic well-being of an ill-newborn is most
accurately assessed using umbilical cord blood gasses.

Newborn screening: tests designed to detect infants with specific conditions who may benefit from
early diagnosis and treatment, e.g. metabolic disorders, endocriopathies, hemoglobinopathies,
hearing loss, and cystic fibrosis.

34.C Complications of 3rd stage of labour


The third stage of labour begins once your baby is born, and ends when you deliver the placenta and
membranes.

Complications:

Postpartum hemorrhage:

Bleeding more than 500mL after deliver


Can be early (within 24 hours) or late (up to 12 weeks after delivery)

Etiology:

Uterine atony (most common cause) – failure of uterine muscle contraction


Laceration of cervix or vagina
Retained products of conception
Coagulopathy
Obstetric shock
Pulmonary embolism

177
Risk factors for atony:

Anesthesia
Uterine overdistention (twins, macrosomia, polyhydramnios)
Prolonged/rapid labor
Augmented labor
Uterine leiomyoma
Preeclampsia with magnesium therapy

Evaluation/ Treatment:

Inspect perineum, vagina and cervix to rule out laceration


Bimanual exam to ensure no uterine rupture or retained products, if unremarkable findings,
bimanual compression/ massage to control bleeding.
Administer uterotonics (oxytocin, methylergonovine maleate, 15 methyl-PGF2 alpha/
Hemabate, misoprostol)
Consider packed RBCs, FFP, Cryoprecipitate, platelets
Operative management:
Uterine artery embolization
Dilate and curettage
Bakri balloon placement, inserted in uterus, then filled with fluid in order to tamponade
uterine bleeding
B-Lynch stitch
Uterine artery ligation
If all else fails, hysterectomy

Obstetric shock can be divided into:

1. Hypovolemic (postpartum hemorrhage, hematoma-broad ligament/paravaginal)


2. Neurogenic (uterine rupture, inversion) can also be iatrogenic
3. Obstructive (air embolism)
4. Anaphylactic like (amniotic fluid embolism)
5. Septic shock (prolonged ROM, retained placental tissues)

Management:

Ensure airways, give oxygen


Control any active bleeding
IV fluids, crystalloids, colloids, blood
Antibiotics in sepsis
Monitor: BP, ECG, pulse oximetry, urine output, serum electrolytes, CVP, ABG

Amniotic fluid embolism (AFE): is a rare obstetric emergency in which amniotic fluid, fetal cells, hair,
or other debris enters the mother's blood stream via the placental bed of the uterus and trigger an
allergic reaction.

178
Clinical features:

Sudden chest discomfort


Dyspnea
Hypotension
Hemorrhage due to DIC
Collapse

Managed by stabilising patient (cardiovascular and respiratory resuscitation and correction of the
coagulopathy)

Triplet 35

35.A Breast cancer


Most common invasive cancer in women. 1 in 12 lifetime risk, increases with age. Two-thirds occur in
women over 50. Should be considered systemic disease at diagnosis.

Risk factors:

Advanced age
BRCA gene
Ductal or Lobular CIS
Atypical hyperplasia
Positive family history
Breast irradiation therapy

Types of breast malignancy:

Infiltrating Ductal Carcinoma: (most common 80%), Most unilaterally start as ductal
carcinoma in situ which then invades basement membrane. Stony hard mas with increasing
size and fibrotic response.

Infiltrating lobular carcinoma: (2nd most common 10%), often unilateral, starts as lobular
carcinoma in situ which then invades BM. Better prognosis than ductal carcinoma.

Inflammatory carcinoma: (uncommon breast malignancy. Rapid growth, early metastasis:


blocks lymphatics leading to redness, swelling & warmth on physical exam. Edematous skin
appears pitted, like the skin of an orange.

Pagets disease: (uncommon breast malignancy, better prognosis than above). Can appear as
pruritic, red, scaly rash of nipple spreading to areola. Nipple becomes inverted, may have
discharge. Can mimic eczema or psoriasis.

179
Prognostic factors:

Size of tumor
Axillary node involvement
Estrogen and Progesterone receptors
DNA content (euploidy or aneuploidy)

Management:

Surgery:
Lumpectomy indicated in early-stage, small <4cm, unilateral.
Mastectomy recommended in DCIN, LCIS, larger tumors
Radiation therapy always after surgery
Hormone therapy if E or P receptor positive (Tamoxifen, Aromatase inhibitors)
Chemotherapy if positive nodes or size ˃1cm

35.B Latest news on hormone replacement therapy


As you approach menopause, your levels of estrogen and progesterone start to decline, often
triggering a whole host of symptoms, such as hot flashes, depression, mood swings, vaginal
dryness, sleep disturbances, and clouded thinking.

Some women may be able to take hormone replacement therapy to treat hot flashes and other
menopausal symptoms in the short term without any ill effects. However, the latest news about
associated health risks has led doctors to believe that the risks are serious enough to outweigh the
benefits of the therapy for many women.

Hormone replacement therapy for menopause is based on the idea that the treatment may prevent
discomfort caused by diminished circulating estrogen and progesterone hormones, or in the case of
the surgically or prematurely menopausal, that it may prolong life and may reduce incidence of
dementia. It involves the use of one or more of a group of medications designed to artificially boost
hormone levels.

In 2002 researchers called a halt to a major government-run study of a hormone therapy used by
millions of older women. Researchers stopped the study, one of a series of clinical trials under the
Women's Health Initiative (WHI), after they found that long-term use of estrogen and progestin
raised the risk of heart disease, stroke, blood clots, and invasive breast cancer.

The next year, the Journal of the American Medical Association (JAMA) reported more bad news for
women getting HRT. The journal published a study of women who took a combination of estrogen
and progestin, a synthetic form of progesterone, and showed they were at risk of getting a more
aggressive form of breast cancer than women who didn't get HRT.

The WHI study did find a few benefits from the HRT regimen, including a 37 percent reduction in the
rate of colorectal cancer, one-third fewer hip fractures, and a 24 percent reduction in total fractures.

Another study of the WHI data published in JAMA in 2008 confirmed the health risks associated with
long-term use of estrogen/progestin hormone therapy in postmenopausal women. While the study

180
showed diminished risk of heart disease three years after the WHI intervention stopped,
researchers found the overall risks of hormone therapy -- such as stroke, blood clots, and cancer --
outweighed the benefits.

35.C Uterine rupture


Def: Spontaneous complete transection of uterus from endometrium to serosa. Usually occurs during
labor and can be life-threatening to the mother and baby.

Management is immediate delivery followed by uterine repair or hysterectomy.

Risk factors:

Previous C-section (higher risk with classical than low transverse)


Trauma (motor vehicle collision)
Uterine myomectomy
Uterine overdistention (polyhydramnios, multiple gestations)
Placenta percreta

Clinical presentation:

Extreme abdominal pain


Abnormal bump in abdomen
Lack of uterine contractions
Regression of fetus after normally progressing labor

Treatment: Emergent laparotomy and delivery, repair of uterus or hysterectomy

Future management: Early delivery via C-section

181
Triplet 36

36.A Non-epithelial ovarian cancer

Ovarian cancer: 5th most common cause of cancer death in U.S.

The mortality rate of this disease is the highest of all the gynecologic malignancies, primarily
because of the difficulty in detecting the disease before widespread dissemination.
Presents most commonly in the 5th & 6th decades of life.

Risk Factors:

Whites are 50% more likely to develop it.


Age, multiparty, primary infertility & endometriosis.
~10% of cases are caused by inherited mutations in the cancer susceptibility
genes BRCA1 & BRCA2.
Women affected with hereditary nonpolyposis colorectal cancer have a 13 x times greater
risk than the general population.

Early signs/symptoms:
Increase in abdominal size
Abdominal bloating
Fatigue
Abdominal pain
Indigestion
Inability to eat normally
Urinary frequency
Constipation
Back pain
Urinary incontinence of recent onset
Unexplained weight loss

Gonadal Stromal Cell Tumors


The gonadal stromal cell tumors make up an unusual group of tumors characterized by hormone
production; hence, these tumors are called functioning tumors. The hormonal output from these
tumors is usually in the form of female or male sex steroids, or on occasion, adrenal steroid
hormones.
➢ Granulosa cell tumor (most common): Occurs in all ages, but in older patients, more likely to
be benign.
➢ May secrete large amounts of estrogen & trigger endometrial hyperplasia / endometrial
carcinoma.
➢ Endometrial sampling is especially important when hormonally active tumors (granulosa)
are diagnosed.
➢ Surgical treatment: total hysterectomy + both ovaries in postmenopausal women & women
who do not wish to remain fertile.
o This tumor may demonstrate recurrences < 10 years later.

182
➢ Sertoli-Leydig cell tumors (rare): testosterone secreting, usually in older patients & should
be suspected in case of hirsutism / virilization + adnexal mass.
o Treatment: total hysterectomy + both ovaries.

Other Ovarian Cancers:

Malignant mesodermal sarcomas (carcinosarcomas): rare, usually aggressive & diagnosed


late with poor survival rate.

Metastasis to the Ovary:

Often bilateral & associated with widespread metastatic disease.

May be associated with abnormal uterine bleeding / virilization.

Breast cancer metastasis to the ovary is common.

In patients previously treated for breast / GI cancer, consider removal of ovaries at the time of
hysterectomy, due to a high predilection for ovarian cancer.

The prognosis for most patients with carcinoma metastatic to the ovary is generally poor.

Treatment: ?????? Chemo/radio?.??

Primary surgical therapy as cytoreductive surgery (“tumor debulking”).

Direct peritoneal seeding is the primary method of intraperitoneal spread.

1. Peritoneal cytology: Ascites / saline irrigation (“wash”) are aspirated & submitted for
cytologic analysis.
2. Inspection & palpation of the entire peritoneal cavity.
3. Partial omentectomy is usually performed for histologic examination.
4. Sampling of pelvic & periaortic lymph nodes.

36.B Planned parenthood and contraception


50% of all pregnancies in the U.S are unplanned although many kinds of effective contraceptive
methods exist.

2 general mechanisms of contraception:


1. Inhibiting the development & release of the egg via oral contraceptives (OCP), long-acting
progesterone injection / contraceptive patch / ring.

2. Imposing a mechanical, chemical barrier between sperm & egg via condoms, diaphragm,
spermicide, fertility awareness & intrauterine contraception.

As a secondary mechanism, some methods also alter the ability of the fertilized egg to implant &
grow (intrauterine device (IUD)). May be used individually / in combination (advantages,
disadvantages, risks & benefits)

183
Each approach may be used individually or in combination and has its own advantages,
disadvantages, risks, and benefits. Seen from another perspective, contraception allows conception
to be a planned rather than an unexpected event.

STERILIZATION AS A METHOD OF CONTRACEPTION

➢ Absolute birth control with no continuing expense, effort / motivation.

➢ Most frequently used method of controlling fertility in the U.S.

➢ All available surgical methods of sterilization prevent the union of sperm & egg by male
sterilization (vasectomy) / female sterilization (tubal ligation & hysteroscopic sterilization).

➢ It is possible to reverse some forms of surgical sterilization with generally poor rates of
success & high financial expense.

➢ Tubal ligation is immediately effective but Complete azoospermia is usually not obtained
until 10w after vasectomy.

➢ Surgical sterilization for women can be by laparoscopy, minilaparotomy & hysteroscopy.

184
Laparoscopy
Done under local, regional / general anesthesia.

o Occlusion of the fallopian tubes by electrocautery / application of a plastic & spring clip
(Filshie clip) / silastic band (Yoon / Falope ring).

o Electrocautery are fast, but they risk electrical damage to other structures, poorer
reversibility & greater incidence of ectopic pregnancies.

o The Hulka clip: most reversible method because of its minimal tissue damage, but has the
greatest failure rate.

o The Falope ring: intermediate reversibility & failure rates.

o The Filshie clip: lower failure rate than the Hulka clip.

Hysteroscopy

Gaining access to the fallopian tubes through the cervix.


The Essure system: introduction of 2 coils into each fallopian tube. Efficacy of ~100%.

Patients are instructed to use an additional contraception for 3 months after the procedure,
until the efficacy of the device can be proven with hysterosalpingography.

Contraindications: nickel / contrast allergies, active pelvic infection & suspected pregnancy.

Used for obese patients, unsuitable candidates for laparoscopic tubal ligation due to their
body habitus.

Side Effects & Complications:

o Infection, bleeding, injury to surrounding structures, ectopic pregnancy & anesthesia


complications.

36.C Umbilical Cord Pathology /Abnormalities


The umbilical cord is a conduit between the developing fetus and the placenta. During prenatal
development, the umbilical cord is physiologically and genetically part of the fetus, which normally
contains two arteries and one vein, buried within Wharton's jelly. The umbilical vein supplies the
fetus with oxygenated, nutrient-rich blood from the placenta. Conversely, the fetal heart pumps
deoxygenated, nutrient-depleted blood through the umbilical arteries back to the placenta.

Pathology of the umbilical cord can be divided into:

Abnormal length (Short cord/ Long cord)


Insertion abnormalities
Abnormal coiling
Umbilical vessels pathology
Umbilical cord accidents which compromise the blood flow
Thrombosis of umbilical blood vessels

185
Umbilical cord inflammation (Acute funisitis/ Subacute necrotizing funisitis)
Miscellaneous rare cord lesions

Abnormal length

Cord length is determined by mechanical stretching of the cord by fetal movements. The cord grows
in 1st and 2nd trimester in particular and approaches most of its ultimate length by 28 weeks.

Short cord

Umbilical cord is shorter than 35 cm at term.


Etiology: Congenital neuromuscular disorders, skeletal disorders, multiple malformations and
similar conditions characterized by decreased fetal movements in utero.
Clinical signs: perinatal asphyxia, fetal malpresentation (transverse, oblique)
Extremely short or absent umbilical cord is found in limb body wall complex. It is a severe
malformation complex with large abdominal/thoracic wall defect, limb anomalies, severe
scoliosis and internal anomalies. Limb body wall complex is uniformly lethal.

Long cord

Cord longer than 70 – 80 cm at term


Clinical signs: increased risk of cord prolapse, entanglement, knotting or torsion.

Insertion abnormalities

Clinical signs: Velamentous (membranous) insertion: Occurs in 1% of singletons and in


almost 15% of twins. The cord inserts into the membranes far away from the placental
margin. Umbilical vessels run unprotected by Wharton’s jelly and are vulnerable to injury.
Velamentous vessels run over the internal os are called vasa previa. These vessels are at risk
of rupture during delivery. The mortality of vasa previa hemorrhage is very high, the fetus
exsanguinates within minutes. Velamentous insertion can be detected before the delivery
using color Doppler ultrasound. The fetus is delivered by elective caesarean section.
Marginal insertion: Insertion at the placental margin (battledore placenta) occurs in 5 – 7%
pregnancies.

Abnormal coiling

Clinical signs: Hypocoiled umbilical cord, achirality (absence of coiling) has adverse fetal
outcome (intrauterine distress, intrauterine demise).
Hypercoiled cord, torsion Umbilical cord torsion (hypercoiling) is a frequent cause of
abortion in the 2nd trimester. Characteristic findings include long hypercoiled cord, stricture
of the cord usually at the fetal end (focal depletion of Wharton’s jelly) or multiple strictures.
The fetus is macerated.

Umbilical vessels pathology

SUA — single umbilical artery.

186
Clinical signs:

about 1% of singletons at term, up to 10 % of twins


common association with some malformations e.g. sirenomelia, trisomy 18, 13, vertebral
anomalies, anal atresia, cardiac defects, tracheoesophageal fistula and/or esophageal
atresia, renal & radial anomalies and limb defects.
occult malformations (renal in particular) should be searched for in an otherwise healthy
infant with SUA

Umbilical cord accidents which compromise the blood flow

Classification:

true knot
nuchal cord, cord entanglements
strangulation by amniotic bands
cord prolapse — cord presents in front of the fetus

Clinical signs:

The incidence of true knots is 0,5% – 1%. Nuchal cord with one loop is found in 20% of
deliveries, two or three loops in 2.5% and 0.5% respectively. Cord prolapse is estimated to
occur in 0.4% of deliveries.
Blood flow is compromised if the knot or nuchal cord tightens. The tightening occurs most
often during delivery. Decreased venous return from the placenta leads to asphyxia. Cord
compression may cause neurologic damage, intrauterine or intrapartum death. Perinatal
mortality is approximately 10% in the presence of prolapsed cord or true knot.
Strangulation by amniotic bands causes fetal demise.

Thrombosis of umbilical blood vessels

Etiology:

umbilical cord compression and stasis in true knots, amniotic band constriction, torsion,
velamentous insertion etc.
maternal or fetal thrombophilia
funisitis
unknown reason often
Venous thrombi are more common than arterial thrombi. Fetal outcome is poor.

Umbilical cord inflammation

Acute funisitis

Etiology: accompanies acute Chorioamnionitis, inflammatory cells are of fetal origin and
constitute the fetal inflammatory response
Macroscopic appearance: The umbilical cord looks normal. White, tan or yellow plaques are
seen with Candida infection.

187
Histology: Inflammatory cells migrate from the umbilical vessels towards the amniotic
surface.

Subacute necrotizing funisitis

Subacute or chronic inflammation of the umbilical cord is associated with high perinatal
mortality.
Etiology: Treponema pallidum, herpes simplex virus, other organisms with low virulence such
as Mycoplasma are also suspected.
Macroscopic appearance: Thickened whitish umbilical vessels resembling macaroni.
Histology: Necrotizing basophilic exudate accumulates in concentric perivascular rings or
crescents. The exudate may become calcified.

Miscellaneous rare cord lesions

Classification:

Cysts: develops from embryonic remnants (allantois, ductus omphaloentericus/vitelline


duct)
Tumors: haemangioma is the most common
Hematomas: invasive prenatal procedures, rupture of a varix
Rupture of the umbilical cord: very rare, described as either complete or more often
partial with local hematoma

Triplet 37

37.A Benign tumors of the ovary


Benign tumors can be divided into:

• Epithelial cell tumors, the largest class of ovarian neoplasm


• Germ cell tumors, benign cystic teratoma or dermoid (most common of all)
• Stromal cell tumors

Benign Epithelial Cell Neoplasms

Benign Epithelial Cell tumors:


o Serous, mucinous & endometrioid (most common is serous cyst-adenoma).

o Treatment: for serous tumors is surgical, because relatively high rate of malignancy. Ovarian
cystectomy may be attempted to minimize ovarian damage. In some cases, Unilateral
oophorectomy (to maintain fertility). If past her reproductive age, bilateral oophorectomy &
hysterectomy.

o Mucinous cystadenoma (2nd most common). May become large, sometimes filling the
entire pelvis & extending into the abdominal cavity. Treatment: Surgery.

188
Benign Germ Cell Neoplasms:
Germ cell tumors are derived from the primary germ cells. They arise in the ovary & may contain
relatively differentiated structures (hair & bone).

The most common tumor found in women of all ages is the benign cystic teratoma (dermoid
cyst / dermoid).

Frequently asymptomatic, unilateral adnexal mass, mobile & non-tender.

Treatment: surgical, because of the possibility of ovarian torsion, rupture & their complications.

Benign Stromal Cell Neoplasms:


Solid tumors, derived from specialized sex cord stroma (granulosa theca cell (estrogen
producing) - feminizing / precocious puberty / Sertoli-Leydig cell tumors (androgen producing) -
hirsutism / virilization).

Both have malignant potential.


Ovarian fibroma (made of collagen) usually small & solid.

37.B Obstetrical analgesia and anaesthesia


o Nerve roots involved in Stage1 of labor: T10-T12
(Cervical effacement and dilation)
o Nerve roots involved in Stage2 of labor: S2-S4
(Stretching of perineum)

Narcotic analgesia:

Usually administered during stage 1 (active phase) of labor before complete dilation of
primigravida or <8cm dilation of multigravida patient. In order to avoid neonatal depression.
If narcotic depression occurs, manage with naloxone.

189
Paracervical block:

Injecting 2-3cc local anaesthetic into Frankenhauser ganglion on each side of the cervix.
May cause transitory fetal bradycardia, which resolves by itself.

Pudendal block:

Injection into pudendal nerve around sacrospinous ligament on both sides. Administered
during the second stage of labor.

Epidural/spinal block:

Needlestick though ligamentum flavum into epidural space. Most concerning side effect is
hypotension from sympathetic blockade. Can be managed with IV-fluids, put patient in left lateral
position (reducing weight of uterus on inferior vena cava) and give ephedrine.

37.C Premature infant


A premature birth: > 3w before term (< 37w).

Late preterm: 34 - 36w.


Moderately preterm: 32 - 34w.
Very preterm: < 32w.
Extremely preterm: < 25w.
Non viable < 24w.

Preterm Signs:
▪ Small body & disproportionately large head.
▪ Less rounded features than a full-term baby's features, due to a lack of fat stores.
▪ Fine hair (lanugo) covering much of the body.
▪ Low body temperature due to a low body fat & less thermoregulation ability.
▪ Labored breathing / respiratory distress (lack of surfactant).
▪ Lack of sucking & swallowing reflexes (feeding difficulties).

➢ Delivery > 34w is safer as adverse outcomes are rare.

➢ May cause long-term handicap (blindness, deafness & cerebral palsy).

➢ Earlier delivery = higher risk.

➢ > 50% of women with painful preterm contractions will not deliver preterm.

190
Risk factors for preterm delivery:

Previous preterm birth or late miscarriage


Multiple pregnancy
Cervical surgery
Uterine anomalies
Medical conditions, e.g. renal disease, high blood pressure and diabetes
Pre-eclampsia and IUGR (spontaneous and iatrogenic).
Smoking cigarettes or using illicit drugs
Poor nutrition/ not gaining enough weight during pregnancy
Some infections, particularly of the amniotic fluid and lower genital tract
Being underweight or overweight before pregnancy
Stressful life events, such as the death of a loved one or domestic violence
Multiple miscarriages or abortions
Physical injury or trauma

Management of preterm labour:

Establish whether threatened or ‘real’ preterm labour


Transvaginal cervical length scan (>15mm unlikely to labour)
Fibronectin assay: if –ve, unlikely to labour
Admit if risk high
Inform neonatal unit
Check fetal presentation with USS
Steroids (12mg betametasone IM—two doses 24h apart). 2 Antenatal steroids reduce
rates of respiratory distress, intraventricular haemorrhage, and neonatal death
Consider tocolysis not >24hrs

The chance of survival at less than 23 weeks is close to zero, while at 23 weeks it is 15%, 24 weeks
55% and 25 weeks about 80%.

Neonatal care:

After delivery, plastic wraps or warm mattresses are useful to keep the infant warm on their
way to the Neonatal Intensive Care Unit (NICU).
Premature babies are kept in incubators designed to keep them warm and limit their
exposure to germs.
Measurement of temperature, respiration, cardiac function, oxygenation, and brain activity.
Treatments may include fluids and nutrition through intravenous catheters, oxygen
supplementation, mechanical ventilation support, Bili lights may also be used to treat
newborn jaundice

191
Complications:

Some of the complications related to prematurity may not be apparent until years after the
birth.
A long-term study demonstrated that the risks of medical and social disabilities extend into
adulthood and are higher with decreasing gestational age at birth and include cerebral palsy,
intellectual disability, disorders of psychological development, behaviour, and emotion,
disabilities of vision and hearing, and epilepsy.

Triplet 38

38.A Benign breast diseases


Def: A spectrum of non-cancerous, heterogeneous disorders that affect the breast. Come to
clinical attention as imaging abnormalities or as palpable lesion found on physical examination, self
or not.

Classification of benign breast lesion can be classified histologically into :

non-proliferative,
proliferative without atypia,
atypical hyperplasia.

PROLIFERATIVE WITHOUT
NON-PROLIFERATIVE ATYPICAL HYPERPLASIA
ATYPIA

simple breast cyst intraductal papilomas atypical ductal hyperplasia

papillary apocrine changes fibroadenomas

Or; developmental abnormalities, inflammatory lesions, epithelial and stromal proliferations, and
neoplasms.

Some congenital abnormalities: Amastia, polymastia, Athlelia (no-nipple), polythelia psychological


morbidity surgical treatment.

Most (80%) of the breast pathology are benign.

The female breast is an appendage of the skin with modified sweat glands, located between the skin
and pectoral fascia. in adolescent and young adult the brest is firm, with age, the glandular and
furors tissue are atrophies.

192
Fat necrosis

Def: Traumatic in nature ( surgery - hematoma,seroma; biopsy; trauma), typically in female with
large fatty breast.

Caused by focal fibrosis and scar tissue formation. early in course:

edema of the fat lobules and irregular swelling clinically indistinguishable from carcinoma.
on USG - echogenisity increased.

Microscopically: central area of necrotic fat cells surrounded by granulomatous reaction


(macrophages..)

Tx: Surgical excision - infiltrative yellowish white mass

Ductal Ectasia

Def: Most frequently in 40s, 50s of age. its a benign lesion, widening of the ducts if scar tissue is
formed around its impossible to differentiate from carcinoma by gross appearance.

193
Present as solitary or multi tender swelling in the sub or peri areolar region.

Sx: gray/green, thick and sticky discharge. tenderness and erythema of nipple and surrounding
breast. if scar forming - mass. nipple retraction, skin adherence and axillary adenopathy.

Microscopically: periductal elastic tissue is destroyed and surrounding tissue are infiltrated with Lym
+ plasma cells.

Tx: conservatively or major duct excision.

Galactocele

Def: Cystically dilated terminal ductules which are filled with milk. (complecation are swelling and
infection)

Classicaly appears weeks - months after cessation of breast feeding as painless lamp.

Tx: surgical excision.

Intraductal Papiloma

Def: A benign lesion of the lactiferous duct’s wall, produce bloody discharge and can be painful.

Tx: surgical - duct excision (a wedge resection).

Fibroadenoma

Def: a focal developmental abnormality of a breast lobule and as such are not true neoplasms. It’s a
benign tumors composed of stromal and epithelial elements,represent a hyperplastic/proliferative
process in single terminal ductal unit. grow rapidly in pregnant female, during hormonal therapy or
immunosupresion. commonly seen in young females and can be described as well circumscribed
lesion developed pre menopause.

In adolescent - juvenile fibroadenoma.

194
Peri-canalicular Intra - canalicular

common till 30 y/o older than 30 y/o

firm discrete mass softer

freely mobile (“breast mouse”) can grow till forms skin necrosis = Giant
fibroadenoma or cystisarcoma phylloides

On clinical examination may be palpable or not, oval freely mobile rubbery masses. size from 1 to 15
cm (surgically removed when bigger than 3-4 cm) but can vary during the menstrual cycle and
pregnancy. 20% will increase in size, 20% will completely resolve, 10% will reduce in size and 50% will
remain static.

Can present solitary, few ( < 5/breast) or multiple (> 5 /breast).

In postmenopausal period tumor can regress and often calcified.

Dx: triple assessment

1) Hx + clinical examination
2) 2) imaging: USG - hypoechoic mass + smooth partially lobulated margins, mammography -
newly developing mass in the outer part of the breast.
3) 3) biopsy (FNAC - fine needle aspiration cytology, core biopsy, open biopsy).

Tx: younger than 35 with triple assessment + supporting diagnosis: observation. Older than 35 or
younger with patient’s request - excision biopsy.

Fibrocystic Disease

Also known as cystic lobular hyperplasia (synonyms - mammary dysplasia, cystic


disease/mastopathy/hyperplasia), most common lesion of the female breast (~30% of patients).
cystic hyperplasia is a variant of physiological process in the breast during menstruation. is the most
benign breast condition, manifestation: mid 30s , 40s (~75%) usually present bilaterally in the outer
upper quadrant of the breast, most painful in the premenstrual periods.

195
Exact etiology is unknown , but :

Hormonal basis:

- estrogen predominance over progesterone can be causative (estrogen stimulate proliferation


of CT and epithelial tissue).
- high levels of serum circulated estrogen.
- progesterone level decresed to 33% from normal.
- corpus lutium deficiency / anovulation.
- premenstrual syndrome
- estrogen dominance on pituutary prolactin levels increasing.
- methylexanthiones - high intake of coffee, tea, chocolate is associated with fibrocystic
disease

Risk factors:

- menstrual abnormalities.
- nuliparous.
- history of spontaneous abortion.
- female with early menarche and late menopause

3 phases of the disease:

phase 1 - moderate stromal fibrosis (hardness of breast and premenstrual tenderness).


phase 2 - progresive fibrosis (increased hardness and tenderness, cyst formation).
phase 3 - pronounced fibrosis (tenderness and macrocyst formation).

Sx: mastodynia ,history of many months up to several years, associated; dysmenorrhea,


menometrorragia and ovarian cyst.

Rare in ovulating females, multiparous female and usage of oral contraceptive.

Nipple secretion - in ~33% in patients with FDB discharge is spontaneous or secretion can be expelled
( yellow/greenish or bluish).

Dx: cytology - may include amorphous material (fat and proteins), ductal cells, erythrocyte and foam
cells.

USG - mass with uniform margin demonstrating NO asymmetry or unusual thickness of the wall. the
central part show no echoes and there is posterior wall enhancement.

FNAC - CT predomination (fibrous), ductal epithelium consist of cohesive cells with a scant rim of
cytoplasm and round/oval small, slightly hyper chromatic nuclei.

Goal of treatment :

conservative - goal: stop progression, relieve pain and reverse changes.

when? - no increasing in size, no nipple discharge.

surgical - when? - increasing in size, discharge from nipple.

196
Tx:

conservative - diet (caffeine restriction), iodine containing agents, diuretics, antiprolactin


drugs, vitamin E.
surgical - lumpectomy in in severe cases.

Rx:

low estrogen combined oral contraceptive,


antiestrogens (Tamoxifen)
androgens (Danazol)

Breast abscess (acute or recurrent)

Etiology: S. aureus and less commonly strep’ pyogenes. infection transmitted by infant via nipple.

Non-puerperal abscess typically contain mixed flora (S.aureus + strep species and anaerobes.

First the infection limited to the segment drained by lactiferous duct, than, may spread and involve
other areas of the breast.

Sx: localized edema, erythema, warm and very painfull breast.localiztion typically on the areolar or
periareolar areas. history of breast abscess, fever, emesis, +/- axially lymphadenopathy, nipple
inversion.

Dx:

USG - localize the abscess.


needle aspiration - presence of pus.
FANC and mammogram - exclude carcinoma ( chronic breast abscess tend to encapsulated,
can’t be distinguished from carcinoma).

Tx:

If cellulitis , give appropriate antibiotic. (undrained pus + antibiotic = antibioma, large sterile
brawny edematous swelling for weeks).
No feeding on infected breast.
Pain relive.
Don’t resolve in 48 hours?! - breast incision and drainage. can be done by: simple nipple
aspiration, guided drainage, surgical drainage.
in case of Periductal mastitis - excision of all major ducts.

197
Other benign miscellaneous disease :

o Lipoma - mature fat cells


o Adenoma - pure epithelial neoplasma
o Diabetic mastopathy - form of lymphocytic mastitis and stromal fibrosis.
o Hamartoma - (fibroadenolipoma) compused of varying amounts of..
o Sarcoidosis
o Foreign body reaction - silicone granulomas (“siliconomas”) usually occur after direct
injection of silicone into the breast tissue or after extracapsular rupture of an implant.

Pseudoangiomatous stromal hyperplasia (PASH) - is a benign myofibroblastic proliferation of


nonspecialized mammary stroma

38.B Abnormal puerperium


Puerperium is defined as the time from the delivery of the placenta through the first few weeks
after the delivery. This period is usually considered to be 6 weeks in duration (postnatal care).

By 6 weeks after delivery, most of the changes of pregnancy, labor, and delivery have resolved
and the body has reverted to the non-pregnant state. When those changes fail to occur,
abnormal pureperium is termed.

Hemorrhage

Postpartum hemorrhage (PPH) is defined as excessive blood loss during or after the third
stage of labor. The average blood loss is 500 mL at vaginal delivery and 1000 mL at cesarean
delivery

Early PPH - within the first 24 hours after delivery. causes are: uterine atony,
lower genital tract lacerations, uterine rupture, uterine inversion, placenta
accreta, coagulopathy and hematoma.

Late PPH - most frequently occurs 1-2 weeks after delivery but may occur up to 6 weeks
postpartum. causes are : retained products of conception, infection, subinvolution of
placental site, and coagulopathy.

Factors predisposing to uterine atony include


- overdistension of the uterus secondary to multiple gestations
- polyhydramnios
- macrosomia,
- rapid or prolonged labor
- grand multiparity
- oxytocin administration
- intra-amniotic infection
- the use of uterine-relaxing agents such as terbutaline, magnesium sulfate, halogenated
anesthetics, or nitroglycerin.

198
In uterine atony, lack of closure of the spiral arteries and venous sinuses coupled with the
increased blood flow to the non-pregnant uterus causes excessive bleeding.

Active management of the third stage of labor with administration of uterotonics before the
placenta is delivered (oxytocin still being the agent of choice), early clamping and cutting of the
umbilical cord, and traction on the umbilical cord have proven to reduce blood loss and decrease
the rate of postpartum hemorrhage.

Lower genital tract lacerations, including cervical and vaginal lacerations (sulcal tears), are the
result of obstetrical trauma and are more common with operative vaginal deliveries, such as with
forceps or vacuum extraction. Other predisposing factors include macrosomia, precipitous
delivery, and episiotomy.

Hx: multiple gestation, polyhydramnios, previous episodes of postpartum hemorrhage, history of


bleeding disorders, and desire for future fertility. Usage of prolonged oxytocin administration, as
well as the use of magnesium sulfate during the patient's labor course.

On physical examination:
Vigorous bimanual examination, which may reveal a retained placenta or a hematoma of the
perineum or pelvis, and which also allows for uterine massage. Closely inspect the lower genital
tract in order to identify lacerations. Closely examine the placenta to determine if any fragments
are missing.

The onset of postpartum hemorrhage is acute, intervention is immediate, and resolution is generally
within minutes check patient's CBC count and prothrombin time/activated partial thromboplastin
time (PT/aPTT) to exclude resulting anemia or coagulopathy. Upon admission of each patient to the
labor ward, obtain ABO and D blood type (Rh +/-) determinations, and acquire adequate intravenous
access.

Treatment:

Initial therapy includes oxygen therapy, bimanual massage, removal of any blood clots from the
uterus, emptying of the bladder, and the routine administration of dilute oxytocin infusion.

If retained products of conception are noted, perform manual removal or uterine curettage.
If Oxytocin is ineffective, Carboprost in an intramuscularly administered dose of 0.25 mg can
be administered every 15 minutes, not to exceed 3 doses.
Methylergonovine can also be intramuscularly administered. Because this agent causes
intense vasoconstriction and may cause transient hypertension, it is contraindicated in
patients with hypertensive disease. Check blood pressure prior to administration.
When postpartum hemorrhage is not responsive to pharmacological therapy and no vaginal or
cervical lacerations have been identified, consider the following more invasive treatment
methods:

o Uterine packing is safe & effective therapy. Use prophylactic antibiotics & continue
oxytocin with this technique. Remove after 24 - 36h. If unsuccessful, it still provides time
in which the patient can be stabilized before other surgical techniques are employed.

199
o Foley catheter with a large bulb can be used as an alternative to uterine packing. This
technique can be highly effective, is inexpensive, requires no special training & may
prevent the need for surgery.

o Uterine artery embolization (under local anesthesia). Believed to preserve fertility.


Complications are rare and include fever, infection & non-target embolization. In
patients at high risk for postpartum hemorrhage, such as those with placenta previa,
placenta accreta, coagulopathy / cervical pregnancy, the catheter can be placed
prophylactically.

o The B-Lynch suture technique: Proven to preserve fertility.

When conservative therapy fails, the next step is surgery with either bilateral uterine artery
ligation/hypogastric artery ligation.

When all other therapies fail, emergency hysterectomy is often a necessary in lifesaving procedure.

Infection – Endometritis

Is defined as an ascending polymicrobial infection. The causative agents are usually normal
vaginal flora or enteric bacteria.

Occurring on:
- 1 or 2 days postpartum, most frequently is caused by group A streptococci.
- on day 3 or 4, the causative organism is frequently enteric bacteria, most commonly E coli, or
anaerobic bacteria.
- more than 7 days after delivery is most frequently caused by Chlamydia trachomatis.
- Endometritis following cesarean delivery is most frequently caused by anaerobic gram-
negative bacilli, specifically Bacteroides species.

Risk factors for endometritis:


Cesarean delivery, young age, low socioeconomic status, prolonged labor, prolonged rupture of
membranes, multiple vaginal examinations, placement of an intrauterine catheter, preexisting
infection or colonization of the lower genital tract, twin delivery, and manual removal of the
placenta.

Tx: intravenus antibiotic, oral if afebrile.

Rx: Gentamicin and Clindamycin every 8 hours. If, Enterococcus Faecalis (25%) addition of
Ampicillin (or Vancomycin for patients with a Penicillin allergy) emergency c-section:
ampicillin/sulbactam, cefazolin, and cefotetan for single-dose antibiotic prophylaxis

200
Urinary tract infection

UTI is defined as a bacterial inflammation of the bladder or urethra. Greater than 105 colony-
forming units from a clean-catch urine specimen or greater than 10,000 colony-forming units on
a catheterized specimen is considered diagnostic of a UTI.

Risk factors for postpartum UTI:


Cesarean delivery, forceps delivery, vacuum delivery, tocolysis, induction of labor, maternal renal
disease, preeclampsia, eclampsia, epidural anesthesia, bladder catheterization, length of hospital
stay, and previous UTI during pregnancy. The most common pathogen is E coli.

In pregnancy, group B Streptococci are a major pathogen. Other causative organisms include
Staphylococcus Saprophyticus, E. faecalis, Proteus, and K. pneumoniae.

Sx: frequency, urgency, dysuria, hematuria, suprapubic or lower abdominal pain,or asymptomatic.

Tx: antibiotic regime for 3 or 7 days.

Rx: trimethoprim/sulfamethoxazole (not with breast feeding), ciprofloxacin, and norfloxacin.

Mastitis

Milk stasis and cracked nipples, which contribute to the influx of skin flora, are the
underlying factors associated with the development of mastitis.. The most common causative
organism, isolated in approximately half of all cases, is Staphylococcus aureus. Other common
pathogens include Staphylococcus epidermidis, S saprophyticus, Streptococcus viridans, and E coli.

Tx: moist heat, massage, fluids, rest, proper positioning of the infant during nursing, nursing or
manual expression of milk, analgesics and antibiotics (penicillinase-resistant penicillins and
cephalosporins).

Rx: Dicloxacillin or Cephalexin. if resistens for penicillin give Erythromycin, Clindamycin, and/or
Vancomycin

Wound infection

Infections of the perineum developing at the site of an episiotomy or laceration, as well as


infection of the abdominal incision after a cesarean birth. Wound infections are diagnosed on
the basis of:

Erythema
Induration
Warmth
Tenderness
purulent drainage from the incision site, with or without fever.

201
Risk factors:
Diabetes, hypertension, obesity, treatment with corticosteroids, immunosuppression, anemia,
development of a hematoma, chorioamnionitis, prolonged labor, prolonged rupture of
membranes, prolonged operating time, abdominal twin delivery, and excessive blood loss.

Etiology:

Mycoplasma species

S aureus, either from the skin or from an exogenous source.

Tx: Perineal infections includes symptomatic relief with NSAlDs, local anesthetic spray. Identified
abscesses must be drained, and broad-spectrum antibiotics may be initiated. Abdominal wound
infections are treated with drainage and inspection of the fascia to ensure that it is intact.
Antibiotics may be used.

Septic Pelvic Thrombophlebitis

Venous inflammation with thrombus formation (fever unresponsive to antibiotic therapy).

Etiology:

- Bacterial infection enters the venous circulation, damages the vascular endothelium and
results in thrombus formation.
- The thrombus acts as a suitable medium for anaerobic bacteria.
- Ovarian veins are often involved because they drain the upper half of the uterus.
- Ovarian vein involvement usually manifests within a few days postpartum.
- Disease with later onset more commonly involves the iliofemoral vein.

Treatment: Anticoagulation with intravenous heparin to an aPTT that is twice normal & antibiotics
for 5-7 days (Gentamicin & Clindamycin)

Endocrine Disorder - Postpartum thyroiditis (PPT)

PPT is a transient destructive lymphocytic thyroiditis occurring within the first year after delivery.

Develops 1-8 months postpartum and is an autoimmune disorder in which microsomal antibodies
of the thyroid play a central role. PPT has 2 phases:

1. Thyrotoxicosis: Thyrotoxicosis occurs 1-4 months postpartum and is always self-limited. The
condition is caused by the increase release of stored hormone as a result of disruption of the
thyroid gland.
2. Hypothyroidism: Hypothyroidism arises between the 4-8 month postpartum.

Hx: may report fatigue, palpitations, heat intolerance, tremulousness, nervousness, and emotional
lability. Patients in the hypothyroid phase often complain of fatigue, dry skin, cold intolerance,
depression, and memory and concentration impairment. Because many of these symptoms are mild

202
and nonspecific and are often associated with the normal postpartum state, PPT may go
undiagnosed.

Sx: Tachycardia, mild exophthalmos, and a painless goiter. TSH is decreased during the
thyrotoxicosis stage and increased during the hypothyroid phase. If the TSH level is abnormal, check
thyroid stimulating antibodies, free thyroxine index (FTI), and radioactive iodine uptake (RIU) in order
to distinguish this disorder from Graves disease. In PPT, RIU is low, thyroid-stimulating antibodies are
undetectable, and FTI is high.

Tx: For thyrotoxicosis phase no treatment is required unless the patient's symptoms are
severe. In this case, a beta-blocker is useful (Propranolol). Since the hypothyroid phase is transient
no treatment is required unless not sustained, than give thyroxine (T4) replacement for 12-18
months.

Postpartum Graves disease

Similar to classic Graves disease, postpartum Graves disease is an autoimmune disorder


characterized by diffuse hyperplasia of the thyroid gland caused by the production of antibodies
to the thyroid TSH receptor, resulting in increased thyroid hormone production and release. No
clinical features distinguish postpartum Graves disease from Graves disease in other settings.

Lymphocytic hypophysitis

Is a rare autoimmune disorder causing pituitary enlargement and hypopituitarism, leading to a


decrease in TSH and to hypothyroidism.

Sx: headache, visual field deficits, difficulty lactating, and amenorrhea.

Dx: requires histopathologic examination. Most patients do not require transsphenoidal


hypophysectomy, so diagnosis is based on history, physical, diagnostic imaging, and the
temporal relationship to pregnancy. Identification of the disorder becomes clearer as the
pituitary reverts to its normal size and recovers some of its normal function.

During the acute phase of this disease, hormone replacement is often necessary.

Sheehan syndrome

Postpartum pituitry gland necrosis. Is the result of ischemia, congestion, and infarction of the
pituitary gland, resulting in panhypopituitarism caused by severe blood loss at the time of delivery.
Patients have trouble lactating and develop amenorrhea, as well as symptoms of cortisol and thyroid
hormone deficiency. Treatment is with hormone replacement in order to maintain normal
metabolism and response to stress.

Psychiatric Disorder

Three psychiatric disorders may arise in the postpartum period:

1) Postpartum blues - Is a mild, transient, self-limited disorder that usually develops when the
patient returns home. It commonly arises during the first 2 weeks after delivery and is

203
characterized by bouts of sadness, crying, anxiety, irritation, restlessness, mood lability,
headache, confusion, forgetfulness, and insomnia. (50-70% of females)

2) Postpartum depression (PPD) - is a more prolonged affective disorder that lasts for weeks to
months.Patients suffering from PPD report insomnia, lethargy, loss of libido, diminished
appetite, pessimism, incapacity for familial love, feelings of inadequacy, ambivalence or
negative feelings toward the infant, and an inability to cope. Consult a psychiatrist when PPD
is associated with comorbid drug abuse, lack of interest in the infant, excessive concern for
the infant's health, suicidal or homicidal ideations, hallucinations, psychotic behavior, overall
impairment of function, or failure to respond to therapeutic trial. (10-15% of females)

3) Postpartum psychosis - occurs in the first postpartum year and refers to a group of severe
and varied disorders that elicit psychotic symptoms. The signs and symptoms of postpartum
psychosis typically do not differ from those of acute psychosis in other settings. Patients with
postpartum psychosis usually present with schizophrenia or manic depression, which signals
the emergence of preexisting mental illness induced by the physical and emotional stresses
of pregnancy and delivery. (< 0.5%)

There is no unified etiology, rather, its a multifactorial model of pathogenesis.

Risk factors:

Undesired pregnancy, feeling unloved by mate, age younger than 20 years, unmarried status, low
self-esteem, dissatisfaction with extent of education, economic problems with housing or income,
poor relationship with husband, being part of a family with 6 or more siblings, limited parental
support, and past or present evidence of emotional problems. Women with a history of PPD and
postpartum psychosis have a 50% chance of recurrence. Women with a previous history of
depression unrelated to childbirth have a 30% chance of developing PPD.

Tx: for blues, educate the patient, for PPD antidepressant (SSRI's) and ECT. postpartum psychosis
must supervised by a psychiatrist and should involve hospitalization.

38.C Sectio Cesarea


Definition: A surgical procedure in which incisions are made through a women's abdomen and
uterus in order to deliver her baby. About 30% of birth in USA.

Most common primary indication (1st C-section) is a failure* of delivery to progress.

Failure to progress?? —> two hours without cervical changes with adequate
contraction in the active phase of delivery.

Most common overall indication is a prior C-section.

The three P's :


Parent (the mother)
Passenger (the fetus)
Power (of uterine contractions).

204
Fetal Maternal Placental

Cord prolapse Prior C-section Placenta previa

Non reassuring fetal testing Active HSV or HIV placenta acreta


(bradycardia..)

Fetal distress Cervical cancer placental abruption

vasa previa +/- velamentus cord


Malpresentation Hypertension or diabetic insertion

CPD - cephalic pelvic disproportion —> both fetal and maternal aspects.

Pre-surgical preparation:
- Insertion of urinary catheter.
- Intravenous accesses.
- Leads for maternal monitoring (HR, BP and rhythm).
- Application of anesthesia - general or regional (spinal or epidural).
- Portion of pubic hair is shaved if needed and abdomen is washed with antibacterial solution.

Surgical procedure:
- First, skin incision is preformed, Pfannesteil incision (horizontally curved ,smile face like) also
known as “bikini cut”. using this type of cut, allows us to perform future VBAC ( Vaginal Birth After
C-section). no more saying : “once C-section,always C-section).
- Longtitudial midline approach can be preformed but rarely done due to Pfannesteil incision
superiority (cosmetic, less pain, faster recovery).
- Abdominal layers opened; subcutaneous, fatty layer, fascia, rectus sheets (external, internal and
oblique muscles), the abdominal rectus muscles and finally the peritoneum. this dissection made
bluntly to avoid complication.
- Now we enter the abdominal cavity
- Bladder is placed for urinary bladder protection.
- The uterus is visible.
- Hysterotomy - small incision is made ( fetus is just under the scalpel ! ! ! avoid injuries) and
proceed bluntly to retract the uterus.

205
- Uterus is opened and amniotic sac is raptured.
- Baby is delivered.
- Umbilical cord is clamped and cut
- Delivery of placenta (3rd and last stage of delivery). give Oxytocine to avoid hemorrhage.
- Close with sutures the hysterotomy, direct uterine massage is helpful to avoid atony.
- suture the fascia and close the skin ( with staple or running subcuticular).
- Dressing of the wound.

(net time less than an hour)

Complication :

- Hemorrhage (>1000 ml)


- Uterine atony
- Placenta accreta
- Bladder injuries
- Infections ( endometritis, wound, thrombophlebitis) - prophylactic antibiotics?!?
- Others; ileus, thrombosis +/- embolism (pulmonary), psychologic disturbances
- Adhesions - may induce infertility, tissue must be dissected meticulously.
- Death ~ 0.02 %

Complication of anesthesia:

1. Spinal
- Intraoperative hypotension: leads to uteroplacental hypoperfusion and can provoke an acute fall
in intervillous blood flow with the potential for fetal acidemia.
- Cardiac arrest.

2. Epidural
- Nerve damage - paralysis.
- Infection.
- Post lumbar puncture headache.
- Nausea and vomiting.
3. Geneal ana’
- Failed intubation.
- Gastric content aspiration
- More blood loss.?
- Surgical “awareness” - low pharmacological dose
- Thromboembolism - prophylaxis anticoagulant?

Post-surgically:

- Patient mobility after 24 hours for prevention of clots.

206
- Remove stitches if present after 10 days.
- NSAID’s +/- opioides (oral morphine)?

Triplet 39

39.A Hormonal treatment in gynecology


Hormone replacement therapy in menopausal transition

Hormonal changes in menopause:

- Lower Inhibin release Rise in FSH – increased ovarian follicular response


- Overall higher estrogen levels
- Lower progesterone levels
- Ovarian failure cessation of steroid hormone release
Symptoms:

- Meno-/Metrorrhagia
- Vasmotor sy (Hot flushes, night sweats)
- Psychological (Depression, worsening of PMS)
- Sexual dysfunction (Vaginal dryness, irritability)
Indication of HT: prescribed in lowest effective dose for shortes period of time

- Vasomotor sy
- Vaginal atrophy
- Osteoporosis

Risk/Benefit of hormone replacement to a smaller or lesser extent:

Risk Benefit
CHD Bone mineral density
Stroke Relief of vasomotor sy
DVT, PE Relief of vaginal dryness
Breast, Ovarian, Uterine cancer Colorectal ca risk lower

Estrogen:

- Oral, transdermal patch, transdermal gel, vaginal


- Systemic ET is most effective treatment for vasomotor sy
- Some contraindications: genital bleeding, Hx/suspicion breast ca, estrogen dependant
neoplasia, DVT, liver dysfunction

Progestins:

- Oral, vaginal
- Indication: vasomotor sy, when estrogen in contraindicated
- Protection against estrogen induced endometrial hyperplasia

207
Combination preparations: oral or transdermal

Hormonal treatment in cancer

Used for palliative treatment of endometrial and ovarian cancer

Tamoxifen

- Estrogen receptor antagonist in breast tissue


- Blocks breast cancer cell growth
- Occausionally used to treat endometrial and ovarian ca.

Megestrol Acetate

- Synthetic derivate of progesterone


- Antiestrogenic effect
- Endomterial hyperplasia, non-operable endometrial ca, recurrent endometrial ca

Contraception

CHECK triplet 40/a

- Intrauterine contraception
- Progestin implants
- Combined hormonal contraceptives
- Progestin-only contraceptives
- Injectable Progestins
- Emergency contraception

Hormonal treatment in infertility

Ovulation enhancement/Controlled ovarian hyperstimulation/Ovulation induction

- Clomiphene citrate
- Exogenous gonadotropins: FSH, LH (+hCG)

Hormonal treatment in benign diseases (endometriosis, PCOS)

Endometriosis

- COCPs:
o Mainstay for treatment of pain
o Inhibit FSH, LH release decrease menstrual flow
- Progestins:
o Antagonize estrogenic effect on endometrium
- Progesterone antagonists
Adenomyosis: COCPs, Progestin only, Levonorgestrel IUD

208
PCOS:

- COCP: first line treatment for oligo/anovulation, hirsutism, acne


- Cyclic progestogens

Termination of pregnancy: Antiprogesterone (Mifepristone), oxytocin

Induction of labour: Oxytocin infusion, Antiprogesterone (Mifepristone)

39.B Imaging modalities/methods in obstetrics and gynecology


Ultrasonography

Low-energy sound waves sent into body – some waves are reflected and received by probe
Safe in pregnancy
Transabdominal sonography: first approach
Transvaginal sonography: at vaginal fornices – closer to region of interest
o Better view on: cervix, uterus, ovaries, tubes
Doppler technology: determine blood flow
o Resistance index, pulsatility index
Saline infusion sonsography: saline infused into uterus and cavity is observed with TVS
Uses of USG
o Uterus: Leiomyomas, Adenomyosis, Endometrial abnormalities (thickness)
o Ovaries: Lesions (cystic/solid), torsion (w/ Doppler)
o Intraabdominal fluid
o Breast (cystic/solid masses)
USG in obstetrics (Check triplet 16/b)
o 3D USG
o TVS visible by 5th wk GA
o Doppler
Computed tomography

Multiple slices of X-ray 2D image


Uses:
o Evaluation and surveillance of malignancies
o Staging
o Post surgical: complications of hysterecomty
PET

Cancer diagnosis and management

MRI

Based on excitement of hydrogen ions


Uses:
o Distorted pelvic anatomy, large masses difficult to identify w/ USG

209
o Leiomyomas, Adenmyosis, Congenital anomalies, Adnexal masses,
o Preoperative assessment for neoplasms
Mammography

X-ray used to screen for breast cancer


Recommended screening between 50-74 in normal population;
Colposcopy

Performed to evaluate Pap results: malignancy/premalignancy


Evaluate surface of cervix, vagina, vulva
Hysteroscopy

Visualization of endometrial cavity: light source, camera, distension medium


Lesions: polyps, intrauterine adhesions, septa, submucous myomas
Hysterosalpingography

Used to evaluate patency of fallopian tubes


Radiopaque dye injected transcervically + live x-ray

39.C Injuries of fetus and newborn


Fetus

Trauma is a leading cause of mortality in pregnancy

Etiology:

- 90-95% blunt trauma, 5-10% penetrating trauma


- Motor vehicle accidents
- Falls
- Assaults
- Gunshuts
- Stab wounds

Serious abdominal injury is more common in pregnant than in non-pregnant women.

Injury to uterus as a result of trauma:

o Placental abruption
o Placental laceration or infarction
o Spontaneous abortion
o Preterm labour
o PROM

o Direct injury to fetus is uncommon. Skull fractures/head injuries occur in pelvic fractures.

Newborn - Birth trauma

210
Def.: Injuries resulting from mechanical forces during birth process
Mostly self-limiting w/ favourable outcome

Risk factors:

- > 4500g
- Intrumental vaginal deliveries
- Vaginal breech delivery
- Excessive traction during delivery
Soft tissue injury:

Also associated with fetal monitoring (eg. scalp blood sampling)

- Cephalhematoma
- Subgaleal hematoma: from vacuum
- Caput succedaneum – deformation caused from cervix. Resolves after few days
- Abrasions and lacerations – vacuum, forceps, scalpel from CS
Brachial plexus:

- Large babies with shoulder dystocia or breech delivery


- Include: Fractured clavicle or humerus
Cranial or laryngeal nerve injury – usually resolves after a few weeks.

Spinal cord injury: from excessive traction/rotation haemorrhages – often stillbirth, rapid neonatal
death, failure to establish resp. function

211
Triplet 40

40.A Hormonal contraception

Hormonal methods of birth control contain either combined estrogen & progestin / progestin only.

A safe & reliable way to prevent pregnancy for most women.

A daily pill taken orally (21d & 7d break / placebo) / (24,4) / (28).
Continuous dosing (12w & 7d placebo).
A skin patch that is changed weekly (transdermal).
An injection given once every 3 months (depot medroxyprogesterone acetate I.M / S.C).
An implant that is worn under the skin (< 3y).
A ring worn in the vagina (monthly).
An intrauterine device (IUD < 5y).
Emergency contraception (morning after pill, < 72h after intercourse).

Mechanism of action:

Preventing ovulation.
Keeping the mucus in the cervix thick & impenetrable to sperm.
Keeping the lining of the uterus thin.
➢ Taken properly - birth control pills are a very effective form of contraception (~99%).

➢ Missed pills are a common cause of pregnancy.

➢ Should be taken a.s.a.p after a pill has been missed.

➢ Backup birth control should be used for 7 days if > 2 pills missed.

Side effects:

Nausea, breast tenderness, bloating & mood changes - typically improve within < 3 months
without treatment / change.
Breakthrough bleeding / spotting. This is particularly common during the first few months of
taking oral contraceptives. This almost always resolves without any treatment within < 3
months.
Forgetting a pill can also cause breakthrough bleeding.
Severe side effects: Women should notify their doctor if they experience abdominal pain,
chest pain, severe headaches, visual disturbances / severe leg pain.
These could be symptoms of several serious conditions including heart attack, blood clot, PE,
stroke, liver & gallbladder disease.

212
Contraindications / Drug interactions:

> 35 years + smoking: high risk of cardiovascular complications - pulmonary embolism (PE).
Are pregnant.
History of blood clots / stroke (coagulopathies): more likely to have clotting complications
while taking the pill.
History of an estrogen-dependent tumor (breast / uterine cancer).
Abnormal / unexplained menstrual bleeding (the cause of the bleeding should be
investigated before starting the pill).
Active liver disease (may worsen liver disease).
Migraine headaches associated & visual / neurologic symptoms (aura) - increased risk of
stroke.
Anticonvulsants — some anticonvulsants may decrease the effectiveness of hormonal birth
control methods (pill, patch & ring).
Including phenytoin, carbamazepine, barbiturates, primidone, topiramate & oxcarbazepine.
Consider depo-medroxyprogesterone acetate.

Antibiotics — Rifampin can decrease the efficacy of hormonal birth control. As a result,
women who take rifampin should not use any hormonal birth control method (pill, patch,
ring, implant, injection).??????
Consider other methods (condom, diaphragm, IUD, sterilization).
St. John's Wort: herbal supplement sometimes taken to treat depression, may reduce the
effectiveness of birth control pills.

Special considerations — some women may take the pill under certain circumstances, but need close
monitoring:

Women with high blood pressure can experience a further increase in blood
pressure and should be monitored more frequently while on the pill.

Women who take certain medication for seizures (epilepsy) and take the pill have a
slightly higher risk of pill failure (pregnancy) because the seizure medicines change
the way the pill is metabolized.

Women with diabetes mellitus who are on the pill may need a slightly higher dose of
insulin or oral diabetes medication. Women with diabetes and vascular
complications from diabetes should not use the pill.

Other uses of hormonal contraceptives:

Dysmenorrhea/ Menorrhagia
Ovarian cancer/ cancer of the endometrium
Acne
PCOS/ Hirsutism
Iron-deficiency anemia

213
40.B Development of the urogenital system and malformations of female
genitalia

Both urinary & genital system are derived from the intermediate mesoderm.

The development of the kidneys involves the formation of three kidney systems:

Pronephros.
Mesonephros.
Metanephros- 3rd system to develop & eventually forms the permanent kidneys.

The Ureteric Bud gives rise to the Ureter, Renal pelvis, Major & Minor Calyces & Collecting tubules.

The kidneys begin to produce urine ~ 10w:

➢ 12w female / male characteristics of external genitalia can be recognized.

➢ The Müllerian ducts form the fallopian tubes, uterus & upper part of vagina.

Abnormalities in female genital system development:

➢ Abnormalities of the Ovaries:

1. Agenesis.

2. Gonadal dysgenesis (Turners syndrome).

3. Failure of descend into the pelvis.

4. Ovotestis (true hermaphrodite).

➢ Uterine (Mullarian) Anomalies:

1) Hypoplasia / agenesis of uterus / vagina:

Women may lack a vagina, cervix, fallopian tubes & body of uterus.
- These anomalies are commonly associated with urinary tract anomalies.

2) Unilateral development (Unicornate uterus):

When one of the mullerian ducts fails to form, a single horn (banana-shaped) uterus
develops from the other healthy side.

214
3) Didelphus uterus (fusion defect):

A double uterus results from failure of the mullerian ducts to fuse.


Each duct develops into a separate uterus each of which is narrower than a normal uterus &
has only a single horn.
These two uteri may each have a cervix / they may share a cervix.
Associated with two vaginas.

4) Bicornate uterus:

Most common (45%).


Results from a failure of fusion between the mullerian ducts at the ‘top’.
This failure may be ‘complete’ which results in two separate single horn uterine bodies
sharing one cervix.

5) Septate uterus:

A failure in the degeneration of the median septum.


In a median septum that persists in the entire uterus into two single-horned uteri which
share one cervix.
6) Arcuate uterus:

The uterus is essentially normal in shape with a small, midline indentation in the fundus
which results from failure to completely dissolve the median septum.
This usually does not seem to have any negative effects on pregnancy.

7) Diethylstillbesterol (DES) uterus:

The daughters of mothers exposed to DES during pregnancy are predisposed to uterine
anomalies.
2/3 have abnormalities including a small, hypolpastic uterus with a T-shaped cavity, & 50%
have incompletely formed / very short cervix.

215
Diagnosis:

Gynaecologic/ obstetric History


Imaging: USG, MRI, laparoscopy, IVP

Treatment: Surgical correction of congenital abnormality

40.C Maternal death


Maternal death is defined as "The death of a woman while pregnant or within 42 days of termination
of pregnancy, irrespective of the duration and the site of the pregnancy, from any cause related to or
aggravated by the pregnancy or its management, but not from accidental or incidental causes."

The most common causes are postpartum bleeding (15%), complications from unsafe abortion
(15%), hypertensive disorders of pregnancy (10%), postpartum infections (8%), and obstructed
labour (6%). Other causes include blood clots (3%) and pre-existing conditions (28%). Indirect causes
are malaria, anaemia, HIV/AIDS, and cardiovascular disease, all of which may complicate pregnancy
or be aggravated by it.

Determinants of maternal morbidity and mortality:

Socioeconomic background: poverty/access to healthcare (A woman’s lifetime risk of dying


as a result of pregnancy or childbirth is 1 in 39 in Sub-Saharan Africa, as compared to 1 in
4,700 in industrialized countries.)
Generally, adequate prenatal care encompasses medical care and educational, social, and
nutritional services during pregnancy.

HIV/AIDS:

Maternal HIV rates vary around the world, ranging from 1% to 40%, with African and Asian
countries having the highest rates. HIV/AIDS can be transmitted to the offspring during the
prenatal period, childbirth, or breastfeeding. If a mother is infected with the HIV/AIDS virus,
there is a 25% chance that she will pass on the virus to her offspring if she does not receive
proper treatment during pregnancy; on the other hand, if a mother is treated during her
pregnancy, there is a 98% chance that her baby will not become infected.
Having HIV/AIDS while pregnant can also cause heightened health risks for the mother. A
large concern for HIV-positive pregnant women is the risk of contracting tuberculosis (TB)
and/or malaria, in developing countries.

Maternal weight:

Increased rates of hypertension, diabetes, respiratory complications, and infections are


prevalent in cases of maternal obesity and can have detrimental effects on pregnancy
outcomes. Obesity is an extremely strong risk factor for gestational diabetes.

Triplet 41

216
41.A Hormonally active ovarian tumors
Are those neoplasm (benign or malignant)
that recreate one or more hormones
(frequently steroids and peptide
hormones) that are clinically manifested in
the patient.

Steroid-hormone-producing ovarian tumors


include those in which the neoplastic cells
secrete hormones as well as a wide variety
of tumors in which the neoplastic cells
stimulate the ovarian stroma or adjacent
hilus cells to become hormonally active.

Some germ cell tumors of the ovary are


hormonally active because they contain
syncytio-trophoblastic cells which produce
HCG. such as specialized types of mature
teratomas:

I. Sex cord-stromal tumors

- sex-cord-cord stromal tumors

1. Granulosa-stromal tumor
a) granulosa-stromal cells tumor
b) tumors of the thecoma & fibrothecoma group

2. Sertoli - leyding cell tumors (adroblastoma)


a) Well differentiated
I. *sertoli cell tumor
**sertoli cell tumor with lipid storage
***sertoli-leyding cell tumor
****hilus cell tumor

b) Intermediate differentiated
c) Poorly differentiate
d) With heterologous elements

3) Gynandoblastoma
4) Unclassidied

• Represent approximately 6% of all ovarian tumors and and nearly 90% percent of hormone-
producing ovarian tumors are Sex Cord-Stromal Tumors (SCSTs).
• SCTS are a heterogeneous group of rare neoplasms that originate from the ovarian matrix. Cells
within this matrix have the potential for hormone production.
• These tumors are subdivided into categories based on histologic criteria.

217
• Present with signs and symptoms of estrogen or androgen excess.
• Hx: The classic presentation is a postmenopausal woman with rapidly evolving stigmata of
androgen excess and a complex adnexal mass. Abdominal pain or a mass palpable by the patient
herself are other telling signs and symptoms.

• Dx: Isosexual precocious puberty is the presenting sign in more than 80 percent of prepubertal
girls, serum testosterone levels >150 g/dL or dehydroepiandrosterone sulfate (DHEAS) levels
>8000 g/L strongly suggestive.

• Tx: surgery : the main treatment for patients with an ovarian SCST is complete surgical resection.
due to their relative insensitivity to adjuvant chemotherapy or radiation. chemotherapy:
malignant stage I ovarian SCSTs may require adjuvant chemotherapy when large tumor size, high
mitotic index, capsular excrescences, tumor rupture, incomplete stag- ing, or equivocal pathology
results are noted

• Prognosis: in general, ovarian SCSTs portend a much better prognosis than epithelial ovarian
carcinomas chiefly because most women with SCSTs are diagnosed with stage I disease. Stage II-IV
tumors are rare, but women with these cancers have a poor prognosis similar to their epithelial
counterparts.

Stroma ovarii
- a specialized type of mature teratoma where the major constituent of the tumor is
thyroid tissue.
- at 50 y/o.
- Histochemically these tumors have been shown to contain both thyroglobulin and
triiodothyronine (T3).
- in some cases clinical hyperthyroidism.

Carcinoid tumors

- Second most common type of malignancy found in mature cystic teratomas.


- To make this diagnosis argentaffin (take up silver stain) granules should be present within
tumor cells.
- Insular carcinoids, if big enough may include
o flushes, diarrhea, weight loss
o murmurs of pulmonary stenosis or tricuspid insufficiency
o asthma
o pellagra-like skin lesions
o peripheral edema
- The syndrome is thought to be caused by production of metabolic end product of serotonin
and can be measured in the urine
- Trabecular carcinoids Strumal carcinoids of the ovary contain an intimate admixture of
thyroid tissue (trabecular or mixed pattern) and carcinoid.

218
Granulosa tumors

Two types of granulosa cells

1) AGCT - Adult Granulosa Cell Tumor - commonly postmenopausal.

- If in reproductive age - Menometrorrhagia and postmenopausal bleeding are common


signs and reflect a prolonged exposure of the endometrium to estrogen. Related to this
estrogen excess, coexisting pathology such as endometrial hyperplasia (hyperestranism)

- Abdominal pain and distension. Acute pelvic pain may suggest adnexal torsion, or
tumor rupture with hemoperitoneum can mimic ectopic pregnancy

- Grossly, adult granulosa cell tumors are large, multicystic, and often exceed 10 to 15 cm
in diameter

- Adult granulosa cell tumors are low-grade malignancies

- Dx: made by histology at the time of surgical excision. an echogenic, septated cystic or
solid mass related to the ovary on USG not specific.

o Serum tumor markers in the diagnostic evaluation:


Inhibin – a peptide that is produced by the ovaries in response to follicle
stimulating hormone and luteinizing hormone. Inhibin usually becomes
undetectable after menopause, unless produced by tumor.

Estradiol — is not a sensitive marker for the presence of a granulosa cell


tumor. Approximately 30 percent of these neoplasms do not produce
estradiol, perhaps related to the lack of theca cells, which produce
androstenedione, a necessary precursor for estradiol synthesis.

Mullerian inhibiting substance (MIS) — MIS is typically undetectable in


postmenopausal women. this test is not really available for clinical use.

2) JGCT - Juvenile - first two decides of life

- These rare neoplasms develop primarily in children and young adults, and approximately 90
percent are diagnosed before puberty.

- Estrogen, progesterone, and testosterone levels may be elevated and lead to suppression of
gonadotropins. As a result, menstrual irregularities or amenorrhea.

- Isosexual precocious puberty, which is characterized by breast enlargement and development of


pubic hair, vaginal secretions, and other secondary sexual characteristics.

Serotoli cell tumor


- Diagnosis is 30 years, but ages range from 2 to 76 years.
Serotoli - Leyding tumors
- During reproductive period (~25 y/o)
219
- These tumors frequently produce sex-steroid hormones, most commonly androgens. As a
result, frank virilization develops in one third of women, and another 10 percent have
clinical manifestations of androgen excess characterized by hirsutism, temporal balding,
deepening of the voice, and clitoral enlargement.
- Microscopically, these morphologically diverse tumors contain cells resembling epithelial and
stromal testicular cells in varying proportions

Theca cell tumors and fibrothecomas


- Postmenopausal women in their mid-60s
- These tumors are composed of lipid- laden stromal cells that are occasionally luteinized.
Half of these luteinized thecomas are either hormonally inactive or androgenic with the
potential for inducing masculinization. ???????????????
- Fortunately, ovarian thecomas are clinically benign, and surgical resection is curative

Gynandroblastoma:
- Rarest type of ovarian SCST. Patients present at a mean age of 30 years and typically
have menstrual irregularities or evidence of hormonal excess.

II. Germ cell tumor

Germ cell tumors represent 20-25% of all ovarian neoplasms. Most of these are benign cystic
teratomas (dermoid cysts).

Teratoma
- contain two or three of the embryonic layers (endoderm, mesoderm, ectoderm). These are the
most common of the germ cell tumors and represent 20% of all ovarian neoplasms.
- They make up over two-thirds of the ovarian tumors that occur under age 15.
- contained within a keratinized epidermis with associated hair follicles, sebacous glands, and sweat
glands.
- can undergo malignant degeneration.
- divided into

Immature (tissues resemble those found during embryonic development). malignant and usually
contain significant amounts of neural tissue. These tumors may cause sexual precocity secondary to
HCG production by syncytiotrophoblasts and mature teratomas (all adult mature tissue), diagnosed
as asymptomatic pelvic masses but there is a 16% incidence of torsion and a 2% incidence of
perforation.

Dysgerminomas (rarely)
- Most common malignant germ cell tumor with endocrine effects. These neoplasms typically occur
in patients less than 30 y/o.

- Virilization, paraendocrine effects associated with dysgerminomas include hypercalcemia due to


parathyroid hormone-related protein. may contain syncytiotrophoblastic elements - serum
elevated hCG.

220
and embryonal carcinomas (frequently) may contain scattered syncytiotrophoblastic cells which
produce HCG.

Some embryonal carcinomas also produce a-fetoprotein.

Endodermal sinus tumors (yolk sac tumors) produce elevated levels of a-fetoprotein and/or
1-antitrypsin and rarely produce HCG

Primary choriocarcinoma of the ovary


- rare (slightly over 100 cases reported) and is composed of both cytotrophoblasts and
syncytiotrophoblast.

- These tumors are usually unilateral and occur in women under age 20 combined with another
germ cell tumor, most frequently with an immature teratoma.

- abnormal menses and a rapid increase in breast size in postpubertal children.

Molar pregnancy - Gestational Trophoblastic Disease (GTD).

41.B Infectious diseases in pregnancy


The fetus depend on the mother good health to undergo proper and physiological development.
When maternal health is compromised by exogenous or endogenous pathogen, the fetus is in a risk.
There are verity of entities which can may influence the health of the mother and her future baby,
and potentially harmful the fetus.

Bacterial infections:
Group B Streptococcal infection -
Leptospira
Treponema pallidum
Viral infection:
Cytomealovirus
Viral hepatitis ( A,B,C)
Varicella zoster
Parasitic infection:
Toxoplasmosis

Maternal infection may be transmitted to the fetus across the placenta (vertical) during fetal
development period (prenatal) or during labor, passage through birth canal (perinatal).

Some infection are venereal origin (sexually transmitted), some can be acquired via aerosols.
Xoonosis are thous who transmitted by infected/carrier animals and environment, other by ingestion
of contaminated food.

221
Viral Respiratory Infection

1. CMV:

- Severe cases when a primary infection occurs in the 1st trimester.

- Transmission to the fetus & its risks: miscarriage / congenital defects (defense, seizurs, mental
retardation, optic athropy & pneumonitis).

- Infection in later pregnancy may cause preterm labor, stillbirth & PROM.

- Diagnosis: blood test for maternal antibodies.

2. Fifth disease (Human parvovirus B19).

- Mostly not dangerous for the mother / fetus. Carriers are children, risk to be a teacher.

- Can cause miscarriage / sever fetal anemia, thrombocytopenia & hydrous fettles, 5w after
infection.

- Clinical presentation: non-specific, rash, arthralgia / arthritis.

- Termination is not indicated because of low risk of fetal damage.

- Pregnancy should be monitored by repeated USG examination to detect fetal anaemia.

- Diagnosis: USG + blood.

- Treatment: if anemia give blood transfusion.

3. Rubeola (Paramyxovirus):

- Clinical presentation: fever, rash, cough, rhinorrhe, Koplis spots, pneumonia, encephalomyelitis,
hepatitis.

- Treatment: prevention ,isolation precautions & vaccination.

- On fetus: purpura, splenomegaly, meningoencephalitis, microcephaly, congenital cataract,


thyroid abnormalities.

- Diagnosis: bacterial culture (blood, CSF, vagina & rectum).

- Treatment: immunoglobolins for non immune mother.

222
4. Chickenpox - Varicella Zoster Virus (VZV):

- Clinical presentation: fever, malaise, pruritic rash, pneumonia, myocarditis, pericarditis, adrenal
insufficiency.

- Fetal effects: preterm delivery, neonatal varicella, varicella syndrome (limb hypoplasia, limb
paralysis, psychomotor retardation, microcephaly, optic disc hypoplasia /chorioretinitis).

Anthropozoonosis (animal vector).

Toxoplasmosis (Toxoplasma gondii):

- Protozoal oocyte in animal intestine (cats).

- ~ 33% of childbearing age females carry the pathogen.

- Production of maternal IgG antibodies almost assuring fetal infection in the future
pregnancy.??????

- If primary infection established in particularly the 1st trimester, toxoplasma established in the
placenta & infects the fetus.

- Screening is advised for immunoglobulins.

- Mother asymptomatic / mild flu like symptoms.

- Clinical presentation: low birth wight < 2.3kg, anemia, hepatomegaly, splenomegaly, jaundice,
neurosensory damage (retinopathies), micro / hydrocephalus & mental retardation.

- Congenital: can appear months - years after birth; neurosensory loss (hearing loss, visual
impairment), mental retardation & other neurological conditions (epilepsy).

- Severe illness / death after birth.

- Diagnosis: maternal blood test for antibodies, amniocentesis, CVS, USG.

- Postnatal fetal examination:

- Diagnosis: cord blood & CSF antibodies test.

- Ophthalmologic & neurologic examination.

- Axial CT.

- Treatment: antibiotics, corticosteroids for lungs, heart & for chorioretinitis add clindamycin.

223
Bacterial Infection

1. Pathogen: B group streptococcus (BGS)


- most common cause of life threatening infection in newborn. ~ 20% carriers at the last trimester
trimester. perinatal infection.
- in the past…. newborn sepsis, meningitis. pneumonia
- if infection during conception or first weeks may led to hearing/visual loss and mental retardation.
- Tx: antibiotic during delivery decrease the risk of infection. immediately intervenes antibiotics.

2. Pathogen: listeria monocytogenes——> Listeriosis


- Ubiquitous in soil and groundwater, plants and animals. most common food ingestion (like
samonella) - food poising.
- Can cause misscareghe, fetal/newborn death, premature delivery, and sever illness to mother and
fetus.
- Tx: antibiotics

3. Pathogen: treponema pallidum (sperochete)——> Syphillis


- trabsmeted prenatally and perinatally.
- in adult: genital lesion
- congenital syphilis is a disabling, sever, and often life threatening disease. premature birth or
stillbirth
- Sx: facial deformities , blidness and deafness.
- Newborn left untreated:
- early Sx: fever, sever congenital pneumonia, bone lesson. erythema;
- late Sx: dental deformities, rashes, neurosensorry loss,

4. Pathogen: chlamydia trachomatis and nisseria gonorroeae


- Rupture of membrane.
- Conjectivitis and pus containing discharge from the eyes.
- Dx: maternal urine, cervical sample for bacterial DNA
- Tx: oral/ointment antibiotics

Sexually transmitted disease

1. Pathogen: human immunodeficiency virus (HIV)——> (AIDS) acquired immune deficiency


syndrome.
- Vertical transmission to fetus ( give anti-retroviral therapy, reduce traumatically)
- Rx: zidovudine

2. Pathogens: herpes simplex virus type 1 and 2 (HSV1 and HSV2)


- Type 2 is 4 times more often.
- Infection before third trimester and mother has no genital sores - little risk for fetal infection.
- If infection during last trimester, no maternal antibodies leading to congenital HSV infection.
- Damaging fetal CNS and internal organs, but rarely death.
- Tx: acyclovir, valacyclovir for prevention and newborn intravenously.

224
3. Pathogens: hepatitis B (HVB)
- Risk for premature delivery.
- If untreated newborn may develop chronic liver disease.
- Tx: prevention: vaccination; if newborn to infected mother give first shot of vaccination and
immunoglobulins within 12 hours second dose of vaccine after 3o days and last dose after six
month.

4. Pathogen: papilloma virus (HPV)——> genital warts


- Genital warts are highly infectious and have more potency to grow during pregnancy.
- If genital warts are anatomically obstruct the birth canal, c-section is indicated.
- Premature birth.
- Congenital HSV; early (first weeks after delivery) - if early, mild: fever loss of appetite.
Severe: cataract, chronic skin infection (blisters), herpes encephalitis.

41.C IUGR
Intra Uterine Growth Restriction (previous called retardation, but its misleading), since most
(70%) will be only constitutionally small, meaning genetic design.

Definition – Fetus of EFW (Estimated Fetal Wight) < 10 % for gestational age.

Etiology:

- Fetal: early infection (Toxo’-Rubela-Cytalom’-Hepris TROCH) , aneuploidy (trisomy - 21,18,13;


Tuner syndrome), anatomical structural abnormalities (Neural Tube Defect - NTD), multi-
pregnancy. symmetrical IUFR - decreased potential growth (not match to do… amniocentesis,
Chorionic Villus Sampiling - CVS, USG)

- Placental: infarction, abortion ,TTTS (Twin-to-Twin-Transfusion), velamentous cord inversion


(inadequate blood supply), asymmetrical (low substrate for the fetus- starvation, decreased
placental perfusion).

- Maternal: HTN (pre-eclampsia), fetal alcohol syndrome, tobacco and street drug abuse, DM1, SLE.
symmetrical.

Type I : Symmetrical IUGR alls known as intrinsic IUGR - HC (Head Circumference) is small
proportionally with the AC (Abdominal Circumference) and the rest of ultrasonographical
measurement (FL –Femoral Length, BPD – Bi Parital Diameter).
Type II: Asymmetrical IUGR - unproportional values. (normal HC but small ABD). brain sparing
effect- fetal response to chronic hypoxia, redistribution of blood to brain, myocardium and
adrenal glands, check MCA (Middle Cerebral Artery) pulsatility, if present, a evidence.

Fetal grow in 3 phases:


1) weeks 4-20; rapid mitosis, increase DNA content, hyperplasia.
2) weeks 20-28; mitosis start decrease and cell size increase, hyperplasia + hypertrophy.
3) weeks 28-40; only hypertrophy

225
If insult occur during phase 1 symmetrical IUGR will developed, if later asymmetric with brain sparing
effect, liver and spleen will be affected.

Non-Doppler feature:
- Oligohydramnions without rapture of membrane.
- Increased HC to AC ratio, if asymmetric.
-
Doppler fetures:
- umbilical artery assessment: increased systolic/diastolic ratio.
- umbilical venous assessment: presence of pulsatility.
- uterine artery assessment: increase S/D ratio.

Complication:
• Hypoglycemia
• Polycythemia
• Thrombocytophenia
• Babes are more prone to birth asphyxia, aspiration syndrome, meconium.
• Hyperbilirubenemia
• Hypothermia
• Necrotizing colitis
Long term:
• More prone to HTN and metabolic syndrome.
• Neurodevelopment.
• Neuropsychiatric manifestation - mental abnormalities.
• CNS defects - cerebral palsy.

Tx: underlining cause and corticosteroids.

Rx: Bethamethasone for lungs maturation.

226
Triplet 42

42.A Diagnosis of amenorrhoea

Amenorrhea - absence of periodical vaginal discharge of blood and mucosal tissue of a non pregnant
female, primary vs secondary, congenital vs acquired.

Most common secondary “acquired “ etiology is PREGNANCY!!!


All reproductive-aged women with amenorrhea should be assumed pregnant until proven otherwise.
Therefore, urinary or serum B-hCG level measurement is essential.

Definition Diagnosed in a female who has

- not menstruated by age 14 years & lacks other evidence of pubertal development
- not menstruated by age 16 in presence of other pubertal signs
- has previously menstruated but has been w/o menses for 6 months

Classically classified into primary (no prior menses) / secondary (cessation of menses) – but it may
lead to errors in diagnosis.

Classification:

227
- History taking: surgical history should focus on prior pelvic surgery, particularly intrauterine
surgery. newonset headaches or visual changes may suggest a tumor of the central nervous
system (bitemporal hemianopsia)

- General appearance: low BMI, +/- tooth enamel erosion from recurrent vomiting (anorexia
nervosa).acanthosis nigricans, hirsutism, or acne, which may indicate PCOS or other causes of
hyperinsulinemia and/or hyperandrogenism

- Examination of genitalia: lack or absent female hair pattern may be due to either lack of androgen
(Kallman syndrome) or androgen insensitivity syndrome. Conversely, elevated androgen levels will
result in a male pattern of genital hair growth

- Proceed to examination of pelvic.

- Labs: complete blood count and blood differentiate, serum chemistries and urinalysis should be
obtained to help rule out systemic disease (autoantibody if suspicion).

- First line of testing:


TSH - Thyroid-stimulating hormone
Prolactin
FSH - Follicle-stimulating hormone
LH - Luteinizing hormone

- If hirsutism is predominant on examination —>androgen testing:


measure testosterone, dehydroepiandrosterone sulfate (DHEAS), androstenedione, and 17-
OH progesterone to determine the organ of cause (adrenal gland vs ovary).

- Progesterone challenge test - (givingprogesterone, than stop!) to demonstrate estrogen effect at


the level of the endometrium.

Amenorrhea with delayed puberty

Thyroid function (TSH) and (T4) and bone age.


- TSH elevated and T4 levels are low——>hypothyroidism.
- bone age is delayed——>constitutional delay.

Bone age normal + high LH and FSH——> karyotyping.


- 45,X, the cause is gonadal dysgenesis ( Turner syndrome)
- 46,XY, the cause is Swyer syndrome; testosterone nor anti-müllerian hormone is
produced.
- bone age is normal + LH and FSH levels are low, obtain a head MRI ——> MRI findings
are abnormal? the cause is pituitary tumor, pituitary destruction, or hypothalamic
disease.

Prolactin levels are elevated——> head MRI


- MRI abnormal?—->may be marijuana use or psychiatric medicine, specificallydopamine
antagonist medications——>decrease in prolactin inhibiting factor and a subsequent
increase in serum prolactin levels
- MRI normal? drug use, an eating disorder, athleticism, or psychosocial stress.

228
Amenorrhea with normal puberty with uterus present
TSH level is elevated, the diagnosis is hypothyroidism.
Prolactin level is elevated, the diagnosis is hyperprolactinemia (causes of
hyperprolactinemia include prolactinoma, CNS tumors, and medications).
Elevated FSH——> Premature ovarian failure is the diagnosis (POF).
Obtain a karyotype.
- Normal karyotype? mosaic Turner syndrome may be present.
- Abnormal? the cause is premature ovarian failure.
Consider premature ovarian failure due to the following:
- Autoimmune oophoritis
- Exposure to radiation or chemotherapy
- Resistant ovary syndrome
- Karyotype abnormality (Turner syndrome)
- Multiple endocrine neoplasm (MEN) syndrome
TSH, prolactin, FSH levels are norm’——>progestin challenge test
- If withdrawal bleeding occurs——>anovulation secondary to PCO syndrome.
- If no withdrawal bleed occurs——> estradiol (E2) priming, followed by a progestin
challenge.
- If the challenge does not induce menses——>consider Asherman syndrome, outlet
obstruction, or endometrial thinning secondary to elevated androgens (PCOS) or
hypothalamic amenorrhea with decreased estrogen production.
The FSH and LH levels may be low or may be below the reference range. The causes include
eating disorders, caloric restriction, exercise, stress, and medications.

Amenorrhea with genital tract abnormalities

Pelvic sonography - uterus is absent and the vagina foreshortened ?——> karyotype.
- If the karyotype is 46,XY ——>testosterone levels.
- testosterone high (male range)——>androgen insensitivity or 5-alpha-reductase
deficiency.
- testosterone low (female range)——>testicular regression.
- karyotype is 46,XX ?——> Müllerian agenesis.

Imaging methods:

Ultrasonography
Pelvic ultrasonography may identify congenital abnormalities of the uterus, cervix, and vagina, or
absence of these organs.

Magnetic resonance imaging


MRI of the pituitary, hypothalamus and olfactory cortex is often indicated in the evaluation of
amenorrhea.

MRI is indicated in the following circumstances:


- Associated headaches or visual-field cuts .
- Profound estrogen deficiency with otherwise unexplained amenorrhea.
- Elevated gonadotropins in the setting of elevated serum estradiol level.
- Hyperprolactinemia.
229
Figure 1 Algorithm for Amenorrhea Diagnosis

230
42.B Anthropozoonosis and pregnancy
An infectious disease transmitted from animals to humans (&vice versa).

Infectious inflammatory disease which can arise from various pathogens, including; bacterias, viruses
and protozoa.

Some typical common symptomatology are bacteremia/viremia which can lead to DIC and shock,
abortions, PROM and stillbirth.

Regarding the fetus the pathogens tend to cross placenta and induce some dramatic and devastating
anomalies, such as; mental retardation, CNS abnormalities and congenital infections.

I. Leptospiriosis - spirochaete bacterial infection - Leptospira


Carrier animals (rodans, pigs, cows), suitability of the environment (tropical/subtropical) for
the survival of leptospires and interaction between man and animals & environment are the
core factor for transmission and infection.

o Direct: source of infection is animal tissue, body fluids or urine (risk population:
Cattle and pig farmers, veterinarians, butchers, laboratory personnel working with
animals).
o Indirect: environment contamination with the urine of the carrier animals (risk
population: water related sportsperson; swimming, canoeing, and agricultural
workers).

It is classically presents as a biphasic illness both icteric and anicteric form of leptospirosis.
The clinical manifestations of leptospirosis ranging from subclinical infection to fulminant
disease.

Invades vessels and lymphatics, and cause endothelial damage and hemorrhage, symptoms
are caused by the capillary endothelial damage.

Maternal effect: 90%-mild disese and full recovery, 10% will have chronic effect.
First phase characterized by abrupt onset of severe headache, chills with rapidly rising
temperature, myalgia, abdominal pain, diarrhea, anorexia, vomiting, cutaneous
hyperesthesia, lymphadenopathy, rash, and hepatosplenomegaly.

The second phase is the immune phase (IgM antibodies) leads to interstitial nephritis,
myocarditis, coronary arteritis, hepatic failure, adrenal insufficiency, aseptic meningitis,
pulmonary hemorrhage, acute pancreatitis,and iridocyclitis.

Mothers with suspected post-partum leptospirosis should immediately stop breastfeeding


and the infant should be blood-tested

Fetal effect: spontaneous abortion in first trimester, intrauterine fetal death, stillbirth, and congenital
leptospirosis may occur, mothers with suspected postpartum leptospirosis should immediately stop
breastfeeding and the infant should be blood-tested.

Dx: Pregnant women with clinical symptoms - Hx of fever for more than 7 days accompanied with ;
severe headache, severe myalgia, conjunctival suffusion, uveitis, arthralgia, rash,
hepatosplenomegaly, evidence of hemorrhage, renal failure, icterus, aseptic meningitis, acute
respiratory distress syndrome (ARDS), HELLP syndrome and pregnancy induced hypertension.

231
Some criteria are more specific , like the modified Faine’s criteria.
Tests: CBC, ABG, liver/kidney function and obtain CSF?, ELISA, PCR, culturing.
Screening: Macroscopic Slide Agglutination Test (MSAT)
Diagnostic test: Microscopic Agglutination Test (MAT) - gold standart.

Tx: for mild disease antipyretic and non aspirin contain analgesic, sever - course of antibiotics.
vaccination is available in some countries (Japan)

Rx: Paracetamol, Amoxicillin, Erythromycin.

II. Toxoplasmosis: protozoal Toxoplasma gondii.


Transmitted by protozoal oocyte, extract in cat’s feces. —> ingestion of cyst or contaminated food
(pork, lamb) or by indirect contact with cats feces.

Primary prevention of a pregnant woman; watching hand before food! wearing gloves, no raw meats,
avoid contacts with cats? (feces).

Maternal: Sx - asymptomatic - flu like symptoms, 6 month after infection consider to be safe for
conception (maternal antibody produced). if primary active infection —> vertical transmission to the
fetus (rates of transmission; 1st trimester - 15% sever disease in neonate, 2nd - 25%, 3rd - 65%
severity is less of neonate).

Fetal: abortion or still birth, hydrocephalus, intracranial calcification, retinochoroiditis is the most
frequent. if sub clinical infection - later developing; hearing defects (10%), visual defects (80%),
mental retardation and learning disabilities, life threatening infamous later if left untreated.
Can cause severe neurological changes. It is a leading cause of blindness in South America) as well as
cardiac and cerebral anomalies.

Consider elective abortion.

Dx: Screening to all female in ideal world, histological investigation, serology, PCR, USG- cerebral
ventricular dilatation or cranial classification, amniocentesis.
Tx: ATBs
Rx: Spiranycin till week 26 no teratogenic. Sulfadiazine + Pyrimethamine.

III. Plague: Yersina pestis a gram (-) rod shaped coccobacillus, faculative anaerobe
Organism carried by lymphatic system to regional lymph nodes - affected lymph - hemorrhagic
necrosis.

Reservoir- prairie dogs and ground squirrels (Slyvatic plaque), rats (urban plaque) and pets (cats).
vectors are fleas, or transmitted by ingestion contaminated food or respiratory route (pneumonic
plaque).
- Bubonic plague - pronounced swelling of the lymph nodes and surrounding edema located at
groin, axilla and neck.
- Pneumonic plague - purulent pneumonia, highly contagious and fatal if untreated.
- Septicemic plague - massive bacteremia, septic shock and death

232
IV. Rabies: Rabies virus and Australian bat lyssavirus.
Acute viral infectious of the CNS of mammals, encephalomyelitis.
~ 100% death without treatment.

Infection is usually via the bite or scratch of a infected animal, most frequently a dog, other animals
such as bats, cats and monkeys are important sources of exposure.

Urban Rabies ‐ is most frequently transmitted to humans through rabid dogs. Transmission may also
occur via rabid cats.

Sylvian (wild) rabies ‐ is maintained in the wild by a host of animal reservoirs including foxes, skunks
and bats.
Tx: Postexposure prophylaxis (PEP).

V. Psittacosis: intracellular Chlamydia psittaci.


Closely associated with birds, acquired mostly from parrots, parakeets, and budgerigars affecting
mostly the lung (pneumonitis) but can be systematic.

Transmitted via air-born (environment occupied by infected bird) or more rarely human to human.
Tx: Antibiotics

VI. Brucellosis: facultative intracelular gram (-) rod bacteria.


Caused by Brucella; abortus, canis, melitensis and suis are a major zoonosis infection.
Involving primarily the reticuloendothelial system. Brucella abortus usually infects cows and is the
usual organism causing human infection in the UK. B. melitensis is more common in the
Mediterranean region and affects goats.

Infection is contracted from animals (cattle, swine, goats, sheep, dogs, coyotes). Transmission of
infection from humans rarely, if ever, occurs.
Mode of transmission: by contact with tissue, blood, urine, ingestion of raw milk and dairy products
(unpasteurized milk)

Maternal: abortion, preterm labour, PROM, febrile illness.

Fetal: intra uterine fetal death (IUFD) - still birth.


there is no significant increase comparing to non-zoonotic infection.

42.C Abnormal puerperium


Same as Question 38/C.

- Hemorrhage
- Infection
- Endocrine
- Psychiatric

233
Triplet 43

43.A Gynecology Preventive Care


Definition of preventive care: Routine health care that includes:
Annual Examinations
Pelvic Ultrasound
Bone Health
Breast Health
Laboratory Examinations

Annual Examinations:
Yearly well woman exam will include pelvic exam, Pap smear and breast exam. We will discuss all age
related issues including contraception, family planning, menopause issues, PMS issues and all other
health related issues pertaining to women.
Pap test: where a sample of cells is taken from the cervix (the opening to the uterus), is the
best way to find changes that may lead to cervical cancer. By finding the abnormal cells and
treating them early we can prevent cervical cancer.
Pelvic exam: is necessary during the annual exam to rule out any abnormalities with the
pelvic organs which include the uterus and the ovaries.
Premenstrual syndrome (PMS) has a wide variety of symptoms, Including: mood swings,
tender breasts, food cravings, fatigue, irritability and depression. It's estimated that as many
as 3 of every 4 menstruating women have experienced some form of PMS.

Pelvic Ultrasound:
Is recommended for patients with pelvic pain or an abnormal finding on physical exam..
Patients who have a family history of ovarian cancer should have the opportunity for a
screening ultrasound.
Types of pelvic ultrasound:
o Transabdominal ultrasound
o Transvaginal ultrasound

Bone Health:
Bone density study is offered to all patients over the age of 40. Because preventive medicine has
more advantages to the health of the patients,
To Find normal bone mass, low bone mass (osteopenia) or severe bone loss (osteoporosis).
The procedure takes approximately 15 minutes and will allow us to assess the health of the
bones in two major problem areas: hips and spine.

Breast Health:
Screening mammograms start at the age of 40.
If a patient has a strong family history of breast cancer the screening age will be lowered
accordingly.
A breast ultrasound is an adjunct to the mammogram and the self-breast exam and physical
exam.
The ultrasound will detect breast cysts easily and can help in diagnosis and treatment of the
patient with a breast lump.

234
Laboratory Examinations:
Full blood tests.
Hormone testing is also part of the routine exam. If patients suffer from hormonal
imbalance,
evaluation of the ovarian hormones, as well as other hormone testing including thyroid
profile and testosterone levels weight fluctuation,
Moodiness
Hair loss.

43.B Normal puerperium


Definition – Puerperium or postpartum period – Is the time from the delivery of the placenta
through the first few weeks after the delivery and the body returns to the normal, non-pregnant
state.
Approximately lasts 6 weeks (42 days).
3 stages:
o Immediately after within 24 hrs: the uterus can be palpated at or near the
umbilicus, Most of the reduction in size and weight occurs in the first 2 weeks.
o Early up to 7 days:
o Remote up to 6 wks.

Reproductive System Changes The Uterus:


Involution of Uterus occurs by:
1. Contraction of the uterus
2. Autolysis / Catabolism – Release of a proteolytic enzyme into the endometrium and
myometrium. This breaks down the protein material in the hypertrophied cells causing the
uterine muscle cells to decrease in size.
3. Regeneration of the Endometrium
a. Placenta site heals in about 6 weeks with the other part healing in 3 weeks.
b. Heals by exfoliation rather than by forming scar tissue.

Assessment of Uterine Changes:


Placement and size -- should be level with the umbilicus after delivery. The uterus then
should decrease, also should be midline and the size of a grapefruit
Tone -- should be firm. Assess by supporting lower portion with one hand and palpate fundus
with other. If found boggy(uterus is more flaccid than would be expected), then massage. Do
not over massage. causes relaxation and more bleeding.

Lochia:
Vaginal discharge, lasts about 5 weeks
o 15% of women have lochia at 6 weeks postpartum.
Assess Type
o Rubra 1-3 days; dark red; consists mainly of blood
o Serosa 3-10 days; pinkish serum with tissue and debris
o Alba 10-14 days; creamy yellowish, brownish
Assess Amount
Assess – Fleshy, not foul smelling Lochia

235
Characteristics of Lochia:
1. Should not be excessive in amount
2. Should never have an offensive odor
3. Should not contain large pieces of tissue
4. should not be absent during the first 3 weeks
5. Should proceed from rubra -- serosa – alba

Cervix
Remains soft and flabby, appears bruised and may have some laceration
No longer does the external os have the pre-pregnant appearance –
Now appears as a jagged slit not a circle (nulliparous state).

Vagina
May be edematous and bruised.
Rugae begin to appear when ovarian function returns.
May teach the mom to do Kegels exercises to increase the blood flow to the area and aid in
healing.

Perineum
May have tears, lacerations, or an episiotomy
Assessment Procedure:
o Turn patient to side-lying / sims position
o Gently spread buttocks apart and with penlight inspect
Assess:
o the episiotomy the same as with any incision.
o Assess for hematoma
Teach hygiene measures to aid in healing

Breasts:
Changes to the breast that prepare for breastfeeding occur throughout pregnancy
Lactation can occur by 16 weeks’ gestation
Colostrum (milk that contains antibodies to protect the newborn against disease).
o 1st 2-4 days after delivery
o High in protein and immune factors
Milk matures over the first week*
o Contains all the nutrients necessary

Allow the mother to assess her own breasts -- similar to doing a self-breast exam
o ask if feels any nodules, lumps
o ask if nipples are sore, reddened, blisters, cracks
Assess nipples for averted, flat, inverted
Teach to care for breasts according to whether they are breastfeeding or bottle feeding.

43.C Abnormalities of pelvis and birth canal


In vaginal delivery, the baby passage from the uterus through the birth canal is largely affected by
the size and shape of the pelvis, as well as the tissues it contains.

The urinary bladder, portions of the pregnant uterus, part of the colon, and numerous nerves and
blood vessels are all supported from underneath by layered sets of muscles in a hammock-like
structure, all of which is surrounded by the bony pelvis.

236
Soft Tissue Abnormalities:

The soft tissues of the pelvis or the presence of any abnormal masses or growths may block the
passage of the baby through the birth canal. The most common abnormal masses arising from the
tissue of the uterus are called uterine fibroids (also known as myomas). Most growths of this type
pose no problems because they are mainly found in the upper part of the uterus. But any fibroids in
the lower pelvis may block the descent of the fetus.

Diagnosis : by Ultrasound evaluation of the size and position of fibroids and/or other masses of soft
tissue may help determine if vaginal delivery is possible, though, the real test is labor itself.

Bony Pelvis abnormalities:

Definition: ring of bone, consisting of the right and left hip bones (each made of an ilium, an ischium,
and a pubis), the sacrum, and the coccyx.

The bony pelvis has four general shapes, which may occur separately or in combination:

Gynecoid: This is the most common pelvic shape and is best for vaginal delivery. It has an oval-
shaped inlet (wider from side to side than from front to back) with parallel sides, dull ischial spines,
and a pubic arch that is 90 degrees or wider.

Anthropoid: This shape has an oval inlet but is wider from front to back than from side to side
compared to the gynecoid pelvis. The sidewalls are parallel or flare outward, and the back part

is roomy enough to accommodate the back of the fetus's head. This may result in the baby being
born face up.

Platypoid: This type of pelvis has an oval-shaped inlet that is compressed from front to back. This
may result in a fetus that traverses the pelvis with its head in a transverse or sideways position.

Android: This is a male-type pelvis, with a small inlet that is somewhat heart-shaped. The sidewalls
converge, the ischial spines are prominent, and the pubic arch is narrow.

The platypoid and android pelvis types are responsible for most obstructed labor

Diagnosis:

The shape of the pelvis is usually assessed in early pregnancy during the initial pelvic exam.
Pelvis ultrasound.

Treatment:

Depended on shape and size of the bony pelvis.


Can be used forces, vacuum and CS.

Birth canal abnormalities:

237
Skeletal abnormalities

Spontaneous and induced deformities are rare


Callus after fractures
Dislocation of sacro-coccygeal joint

Soft tissue abnormalities

Excess fat in vaginal wall


Persistent remnants of Mullerian duct
Tumours
Bladder problems
Failure of soft tissues to dilate, Stenosis of vulva and vestibule

Excess fat in vaginal wall

Decreases effective diameter, Due to excessive feeding in pregnancy

Persistent remnants of Mullerian duct

May trap a leg


Band can be cut with little haemorrhage

Tumours

Lipoma, papilloma, myofibroma


Especially in sexually active women with multi-partners.

Bladder

Bladder can turn inside out


Herniated through floor of vagina.

Incompetence of cervical dilation

Most common indication for caesarean section

True ringwomb

Fault in the ripening of collagen, Often localized to part of cervical canal


TX: indication for caesarean section.

238
Triplet 44

44.A Pediatric and Adolescent Gynecology


Encompasses:

Pediatrics + Gynecology + Reproductive endocrinology + Pediatric endocrinology + Pediatric urology

Physiology and Anatomy:

Childhood years: low plasma levels of FSH, LH, estradiol until puberty GnRH pulse
generators
Growth of uterus and ovaries
Pubertal changes: primary and secondary sexual characteristics (Tanner stages) around 8-13
years

Gynecologic examination:

Prior 18: child must have consent of parent/legal guardian


Breast inspection, examination of external genitalia introitus, hymen, lower portion of
vagina
Internal examination: rarely necessary (foreign body, tumor, bleeding) +anesthesia

Problems in pediatric gynecology:

Labial adhesion: fusion of labia minora – hyperestrogenism?


Congenital anatomic anomalies (CHECK 40/b) – imperforate hymen, transverse vaginal
septum, vaginal agenesis, …
o Primary amenorrhea, cyclic pain
Vulvitis – Allergic and contact dermatitis, Lichen Sclerous, Infection
o Discomfort, pruritus, discharge
Physiologic discharge
Vulvovaginitis – very common: often non-specific
Genital trauma – sexual abuse?
Ovarian Tumors – Most ovarian masses are benign ovarian cysts
Breast development/disease: witches milk: nipple discharge (maternal hormones)
Vaginal bleeding: in neonates – due to withdrawal of maternal estrogen – resolves. Other:
careful evaluation!
Precocious puberty: (CHECK 2/a)
Delayed puberty: (CHECK 2/a)
Contraception – most common prescribed: Combined oral contraceptive pills (COCP)
Information on HPV vaccination

239
44.B Hormones of the Placenta
Human chorionic gonadotropin (hCG)

Produced by syncytiotrophoblast as soon as implantation occurs (6-9th day)


Functions:
o Stops atrophy of Corpus Luteum & stimulates it’s release of progesterone and
estrogen
o Prevents abortion
o Stimulates production of testosterone in male fetus
Indicator in pregnancy test: urine/blood; +ve around 6-12 days after ovulation (blood test
more sensitive)
hCG levels double every 48-72 hours viability. Failure to increase:
o Early pregnancy loss
o Ectopic pregnancy
Unexpected high hCG: gestational trophoblastic disease/neoplasm, chorioca,

Progesterone

Luteal-placental shift: at 7-9 wks: Placenta takes over with production


Functions:
o Prevents abortion
o Prevents uterine contractions
o Supports endometrium
o Suppresses FSH, LH production
< 20nmol/L suggestive of failing pregnancy/ectopic pregnancy

Estrogen

Functions:
o Stimulates growth of myometrium
o Stimulates mammary gland developments
o Suppresses FSH, LH production

Human Placental Lactogen (hPL)

Function
o Development of fetal metabolism
o Stimulates production of insulin-like growth factor (IGF), insulin, surfactant, and
adrenocortical hormones
Decreased in toxemia, chorioca., placental insufficiency

44.C Critical conditions in obstetrics


Sudden maternal collapse

General investigations: History, physical examinations, bloods

240
ECG, CXR, arterial blood gas

Some causes of sudden maternal collapse:

Obstetric
o Massive obstetric hemorrhage (possibly concealed)
o Pre-eclampsia (intracranial hemorrhage)
o Eclampsia
o Amniotic fluid embolism
o Neurogenic shock
o Surgical complications
o Cardiac failure

Medical/Surgical
o Pulmonary embolism
o Cardiac failure
o Shock
o Cerebral haemorrhage
o Substance abuse
o Cerebral infection
o Metabolic

Shoulder dystocia

Def.: Failure to deliver shoulders when applying gentle traction on head.


Cause: usually anterior shoulder is compressed against the pubic symphysis rapid fetal
deterioration due to cord compression
Risk factors:
o Antenatal: macrosomia, previous shoulder dystocia, BMI > 30, DM, post-term
o Intrapartum: lack of progress in 2nd stage, instrumental vaginal delivery
Complications:
Fetal
o Hypoxia, cerebral palsy
o Brachial plexus palsy
o Fracture of clavicle or humerus
o Intracranial hemorrhage
o Cervical spine injury
o Death (rare)
Maternal
o PPH
o Genital tract trauma
Management:
Episitomy
McRoberts position (hyperflexed hips, abducted thighs)
Suprapubic pressure
Internal manoeuvres

241
Replacing head into vagina CS
Symphysiotomy
Paediatric resuscitation team, expect PPH, expect medico-legal issues

Massive obstetric haemorrhage:

Loss of 30-40% (around 2L) of patient’s blood volume. Cave! Blood loss can be concealed!

Causes:

Antepartum:
Placental abruption (Check 23/c)
Placenta praevia (Check 9/c)
Septicaemia
Intrapartum
Intrapartum abruption
Uterine rupture
Amniotic fluid embolism
Complications of CS, angular or broad ligament tears
Placenta accreta/percreta
Vasa praevia
o Fetal vessels run below presenting part
o Triad: Membrane rupture + painless vaginal bleeding + fetal bradycardia
o CS
Post-partum (Check 1/c)
Atonic uterus (“Tone”)
Genital tract trauma (“Trauma”)
Coagulopathy (“Thrombin”)
Retained products of conception (“Tissue”)

Acute hypovolemia, CV-decompensation, DIC, Hypoxia, Lactic+metabolic acidosis,


Multiorgan dysfunction.

General management:

ABC – O2, intubation


Cross match blood, coag. panel
Fluid resuscitation
Catheterize, measure urine output
Blood transfusion
Replace clotting factors

Specific management depends on cause (eg Check 1/c) – But some crucial steps are:

242
Empty uterus (fetus, placenta, tissue) – massage uterus/compression – uterotonics – repair genital
tract injuries – uterine tamponade – laparotomy

Venous thromboembolism (VTE)

Can lead to deep vein thrombosis (DVT) and pulmonary embolism (PE)
Pregnancy = hypercoaguable state
Other risk factors include: previous VTE, thrombophilias, varicositas, BMI > 30,…
Clinical:
o DVT: Leg pain, edema, tenderness, erythema, pyrexia,
o PE: Dyspnea, collapse, chest pain, haemoptysis, raised JVP, fainty,
Dg.: USG, MRI, ECG, Ventilation/perfusion scan, arterial blood gas
Treatment: LMWH, therapeutic dose
Prevention: LMWH, antenatal/postpartum – depends on risk factors

Amniotic fluid embolism

Rare, often fatal – unpredictable/unpreventable – rapidly progressive


Risk factors: Multiple pregnancy, Older maternal age, CS, instrumental vaginal delivery,
Eclampsia, polyhydramnios, placenta praevia, placental abruption, cervical laceration,
uterine rupture, medical induction of labour
Clinical: classic triad
Hypoxia, respiratory arrest
Haemodynamic collapse
DIC
Shock, Fetal distress
Dg./DD.: per exclusionem: PE, anaphylaxis, sepsis, eclampsia, MI
Management:
CPR ICU
O2
Fluid resuscitation
Vassopressors
Management of DIC: FFP, platelets
Deliver fetus by CS

Uterine inversion

Cause/Risk factors: Strong traction on umbilical cord, abnormal adherence of placenta,


uterine anomalies
Clinical:
Haemorrhage + Shock
Sever abd pain in 3rd stage
Uterus not palpable + mass in vagina

243
Management:
Fundus pushed back through cervix or hydrostatic repositioning or laparotomy
IV access CBC&coag. – fluids
Placental left in situ until uterus is replaced

Cord prolapse

Def.: Cord protrudes below presenting part after ROM (rupture of membranes).

May cause cord compression

Compromise of fetal circulation

Perinatal hypoxia, fetal death

Risk factors:
Transverse line, breech
Multiple pregnancy
Polyhydramnios
Prematurity
Long umbilical cord
Management:
- Deliver ASAP
- Prevent further prolapse (manually, filling the bladder)

Fetal distress of second twin

Common causes of distress in second twin:

- Placental abruption
- Cord prolapse
- Excessive uterine contractions

Management: deliver by fastest, safest way.

244

Вам также может понравиться